DTRS Explanation January 2024

You might also like

Download as pdf or txt
Download as pdf or txt
You are on page 1of 167

DTRS Expalanations

CSE 2024 January 2024 1


DTRS Expalanations

CSE 2024 January 2024 2


DTRS Explanations GENERAL STUDIES
January 2024 (Paper-I)
1.
Answer: D
Context- Evaluation of Project Tiger and it being in conflict with the provisions of the Forest Rights Act 2006 (affecting the
rights and livelihood of the tribal and forest dwellers).
Explanation:
Statement 1 is incorrect: Forest Rights Act 2006 defines Critical Wildlife Habitats (CWHs) as ‘areas of national parks and
sanctuaries where it has been specifically and clearly established, case by case, on the basis of scientific and objective crite-
ria, that such areas are required to be kept as inviolate for the purposes of wildlife conservation.
The power to notify the rules to designate a CWH rests with the Ministry of Environment and Forests. The State Govern-
ments are needed to initiate the process for notification of a critical wildlife habitat by submitting an application on a case-
by-case basis, to the Ministry of Environment and Forests, which is the nodal agency under the said Act. Critical Wildlife
Habitats are thus declared by the Central Government ONLY.
Before a critical wildlife area is notified, not only do the rights of the tribals and forest dwellers have to be settled, but also
scientific evidence has to be provided to establish that people’s presence would adversely impact the wildlife in the area.
Statement 2 is incorrect: Section 38V of the Wildlife (Protection) Act, 1972 (as amended in 2006) explains the core or
critical tiger habitat as well as the buffer or peripheral area of a tiger reserve.
A tiger reserve includes two parts:
y Core or critical tiger habitat (National Park or Sanctuary status)
y Buffer or peripheral area.
The phrase ‘core or critical tiger habitat’ is mentioned only in the Wildlife (Protection) Act, 1972, as a sequel to amendment
made to the said Act in 2006. It is NOT defined in the Scheduled Tribes and Other Traditional Forest Dwellers (Recognition
of Forest Rights) Act, 2006.
Source: The government has trapped Project Tiger, now 50, in a tough spot | Explained - The Hindu; Clarification by Minis-
ter of State for Environment and Forests on Tiger Reserves, Critical Wildlife Habitats and Forest Right Act, 2006 (pib.gov.in)

2.
Answer: C
Context: The question is based on a write up that is featured in the Down to Earth magazine. The write up talks about the
strategic importance of the Sahel region and how global powers are engaging with the region in greater strategic interests
of each other.
Explanation: Libya is not a part of Sahel region
Sahel region: Sahel is a semi-arid region of western and north-central Africa extending from Senegal eastward to Sudan.
It forms a transitional zone between the arid Sahara (desert) to the north and the belt of humid savannas to the south.
The Sahel part of Africa includes from west to east parts of northern Senegal, southern Mauritania, central Mali, northern
Burkina Faso, the extreme south of Algeria, Niger, the extreme north of Nigeria, the extreme north of Cameroon and

CSE 2024 January 2024 3


DTRS Expalanations
Central African Republic, central Chad, central and southern Sudan, the extreme north of South Sudan, Eritrea, and the
extreme north of Ethiopia.

Source: https://www.downtoearth.org.in/blog/africa/scramble-for-the-sahel-why-france-russia-china-and-the-united-
states-are-interested-in-the-region-93799

3.
Answer: D
Context: Seven products from Odisha, ranging from the Similipal Kai chutney made with red weaver ants to the embroi-
dered Kapdaganda shawl, have bagged the coveted Geographical Indication (GI) tag in recognition of their exclusivity to the
state.
Given below is correctly matched pairs:
GI Products Related states
1. Kapdaganda shawl Odisha
2. Lanjia Saura Painting Odisha
3. Dhenkanal Magji Odisha
Kapdaganda shawl
y Woven and embroidered by the women of the Dongria Kondh tribe, a particularly vulnerable tribal group (PVTG)
in the Niyamgiri hills in Odisha’s Rayagada and Kalahandi districts, the shawl reflects the rich tribal heritage of the
Dongria Kondhs.
y It is embroidered on an off-white coarse cloth with red, yellow and green coloured threads, with each colour hold-
ing significance. Green symbolises the mountains and hills, and yellow stands for peace and happiness. Red stands
as the symbol of blood.

CSE 2024 January 2024 4


DTRS Expalanations

Lanjia Saura Painting


y The painting, one of the oldest tribal art forms, is also known as Idital. The artworks are famous for their beauty,
aesthetics, ritualistic association and iconography.
y The art form belongs to the Lanjia Saura community, a PVTG largely residing in the Rayagada district. These paint-
ings are in the form of exterior murals painted on the mud walls of homes. White paintings figure over a crim-
son-maroon background.
y It is believed that the Lanjia Sauras paint their walls with Idital artworks to show gratitude to their deities and fore-
fathers, and also for the well-being of their community. Reflecting the love and affection of the primitive tribes for
nature, they feature subjects like tribal humans, trees, animals, birds, the Sun and the Moon.

Dhenkanal Magji
y Dhenkanal Magji is a type of sweet made from cheese from buffalo milk, with distinct characteristics in terms of
appearance, taste, flavour, shape, and size.
y It also has unique nutritional values that distinguish it from other cheese-based sweets. The sweet is prepared by
draining moisture from the cheese and then frying it, finally forming balls from the mixture.

CSE 2024 January 2024 5


DTRS Expalanations

Source: https://indianexpress.com/article/explained/explained-culture/odisha-gi-tag-products-significance-9098672/

4.
Answer: B
Explanation:
Context: Recently, the NITI Aayog organized a national workshop on Harnessing the Potential of Fisheries in Marine
States at Indian council of Agricultural Research - Central Marine Fisheries Research Institute (CMFRI).
Statement 1 is correct. India is the third largest fish producing country and ranks second in aquaculture production.
Statement 2 is correct. Fisheries are state subject under Seventh Schedule of Constitution while Fishing and fisheries
beyond territorial waters (Exclusive Economic Zone - beyond 12 nautical miles) are on the Union list.
Key Policy Suggestions for sustainable marine fisheries management include
y Introduce central legislation (Marine Fishery Act) for regulation of fishing in areas beyond territorial waters to ad-
dress policy and legislative vacuum.
y Institutionalize regular stock assessments of marine fishery resources (MFR) and mechanism for overseeing deep-
sea fishing.
y Establish national guidelines on eco-labeling of MFR.
y Implementation of frontier technologies like Artificial Intelligence for tracking fishing vessel through Vessel Moni-
toring Systems.
y Initiatives taken to harness fisheries potential
y Pradhan Mantri Matsya Sampada Yojana (PMMSY) 2020.
y National Policy on Marine Fisheries Policy (NPMF) 2017 for promoting blue revolution through sustainable utiliza-
tion of MFR.
y Fisheries and Aquaculture Infrastructure Development Fund (FIDF), 2015 for infrastructure creation.
CSE 2024 January 2024 6
DTRS Expalanations
Central Marine Fisheries Research Institute (CMFRI):
y The Central Marine Fisheries Research Institute was established by the Government of India in 1947 under the
Ministry of Agriculture and Farmers Welfare.
y Later it joined the Indian Council of Agricultural Research (ICAR) in 1967.
y ICAR is the largest network of agricultural research and education institutes in the world functioning under the
Department of Agricultural Research and Education, Ministry of Agriculture & Farmers› Welfare.
Source: https://www.en.krishakjagat.org/india-region/niti-aayog-organises-national-workshop-on-harnessing-the-po-
tential-of-fisheries-in-marine-states-at-icar-cmfri/

5.
Answer: A
Explanation:
Context: The global economy has recovered from last year after the impact of global recession fears was factored in.
However, a World Bank report said that 2020s will go down as a decade of wasted opportunity on January 9, 2023. The
global economy is set to “rack up a sorry record by the end of 2024 —the slowest half-decade of GDP growth in 30 years,»
according to the World Bank’s latest Global Economic Prospects report.
y The Global Economic Prospects Report 2024 presents a challenging economic outlook.
y The World Bank forecasts a deceleration in global GDP growth, from an estimated 2.7% in 2023 to 2.4% in 2024,
signaling a continuation of sluggish growth.
y The report also highlights that by the end of 2024, people in one out of every four developing economies will still
be poorer than they were before the pandemic.
y The World Bank forecasts 2024 global growth to slow for the third consecutive year, with growth weaker than in
previous periods.
y The International Monetary Fund (IMF) projects global growth to fall from an estimated 3.5 percent in 2022 to 3.0
percent in both 2023 and 2024.
y These reports collectively indicate a challenging economic environment with slowing growth prospects for 2024.
Source: https://www.livemint.com/economy/world-bank-indermit-gill-has-said-2020s-will-go-down-as-decade-of-wast-
ed-opportunity-talking-on-global-economy-slowdown-11704864632767.html

6.
Answer: D
Context: Scientists have found a natural remedy, a fungi biopesticide, to protect eucalyptus forest plantations from a pest,
a snout beetle, which is known to cause serious damage to eucalyptus.
Explanation:
Statement 1 is correct: Biopesticides are pesticides derived from natural materials, including animals, plants, fungi, mi-
crobes and certain types of minerals.
Biopesticides fall into three major classes:
1. Biochemical pesticides are naturally occurring substances that control pests by non-toxic mechanisms. Biochem-
ical pesticides include substances that interfere with mating, such as insect sex pheromones, as well as various
scented plant extracts that attract insect pests to traps.

CSE 2024 January 2024 7


DTRS Expalanations
2. Microbial pesticides consist of a microorganism (e.g., a bacterium, fungus, virus or protozoan) as the active ingre-
dient. Microbial pesticides can control many different kinds of pests, although each separate active ingredient is
relatively specific for its target pest[s].
3. Plant-Incorporated-Protectants (PIPs) are pesticidal substances that plants produce from genetic material that
has been added to the plant. For example, scientists can take the gene for the Bt pesticidal protein and introduce
the gene into the plant›s own genetic material.
Statement 2 is correct: Biopesticides are usually inherently less toxic than conventional pesticides.
Biopesticides generally affect only the target pest and closely related organisms, in contrast to broad spectrum, conven-
tional pesticides that may affect organisms as different as birds, insects and mammals.
Statement 3 is correct: Canola oil and baking soda have pesticidal applications and are considered biopesticides.
Biopesticides often are effective in very small quantities and often decompose quickly, resulting in lower exposures and
largely avoiding the pollution problems caused by conventional pesticides.
About the new research:
Eucalyptus snout beetle (Gonipterus platensis) is a leaf-feeding beetle that is a major defoliator of eucalypts, according to
the Food and Agriculture Organization of the United Nations. The pest is indigenous to Australia but occurs in many coun-
tries throughout the world where eucalypts are grown.
The identification of the pathogenic fungi to control the beetle is already known, according to a recently published study.
Now, the characterisation could help protect the 115,570 hectares of eucalyptus plantation in the country.
Source: https://www.downtoearth.org.in/news/science-technology/scientists-characterise-a-natural-pathogenic-fun-
gi-to-help-save-eucalyptus-forests-from-devastating-pest-93788
https://www.epa.gov/ingredients-used-pesticide-products/what-are-biopesticides

7.
Answer: B
Context: In a first, India is set to chair and host UNESCO›s World Heritage Committee in New Delhi from 21st to 31st July
2024, the 46th session of the committee.
Explanation:
Statement 1 is correct: The Committee is responsible for the implementation of the World Heritage Convention, defines
the use of the World Heritage Fund and allocates financial assistance upon requests from States Parties. It has the final say
on whether a property is inscribed on the World Heritage List.
Statement 2 is correct: The World Heritage Committee meets once a year, and consists of representatives from 21 of the
States Parties to the Convention elected by their General Assembly.
Statement 3 is incorrect: India has been a member of the World Heritage Committee thrice before (1985-91, 2001-07 &
2011-15) and is serving a fourth term (2021-25). This is the first time that India will chair and host the committee summit.

8.
Answer: A
Explanation:
Context:
In continued violation of Madras high court›s three-year-old order to remove encroachments in Chitlapakkam lake in south
Chennai, the water resources department has left nearly 330 encroachers untouched.

CSE 2024 January 2024 8


DTRS Expalanations
1. Sissu Lake: Sissu Lake is a man-made lake located in the Sissu Valley, Lahaul and Spiti district of Himachal Pradesh,
India. It is one of the highest lakes in India, located at an altitude of 4,270 meters (14,010 feet). The lake is sur-
rounded by snow-capped peaks and pine forests, and is a popular tourist destination.
2. Chitlapakkam Lake: Chitlapakkam Lake is a lake located in the Chitlapakkam area of Chennai, Tamil Nadu, India. It
is one of the largest lakes in Chennai, with a surface area of 160 hectares (395 acres). The lake is surrounded by a
park, which is a popular spot for picnics and recreation.
3. Madiwala Lake: Madiwala Lake is a lake located in the Madiwala area of Bengaluru, Karnataka, India. It is one of
the largest lakes in Bengaluru, with a surface area of 100 hectares (250 acres). The lake is surrounded by a park,
which is a popular spot for boating, fishing, and bird watching.
4. Vibhutipura Lake: Vibhutipura Lake is a lake located in the Vibhutipura area of Hyderabad, Telangana, India. It is
one of the largest lakes in Hyderabad, with a surface area of 80 hectares (200 acres). The lake is surrounded by a
park, which is a popular spot for picnics and recreation.
Source:
1. https://timesofindia.indiatimes.com/city/chennai/encroachers-remain-on-chitlapakkam-lake-despite-ma-
dras-high-court-order/articleshow/106713024.cms
2. https://www.livemint.com/news/himachal-pradesh-in-grip-of-peak-winter-spell-sissu-lake-freezes-at-15-de-
grees-watch-video/amp-11704776625080.html#amp_tf=From%20%251%24s&aoh=17048103150068&c-
si=0&referrer=https%3A%2F%2Fwww.google.com
3. https://www.deccanherald.com/india/karnataka/bengaluru/fencing-gaps-threaten-rejuvenated-lakes-finds-sur-
vey-2843859
9.
Answer: A
Context: India established the «Global Alliance for Global Good - Gender Equity and Equality» at the 54th annual World
Economic Forum (WEF) in Davos for promoting women›s empowerment and gender equality.
Explanation:
y The WEF has come on board as a ‘Network Partner’ and Invest India as an ‘Institutional Partner’.
y The alliance is a response to the G20 Summit 2023 Leaders› Declaration and India›s commitment to women-led
development.
y It seeks to build on the initiatives of the Engagement Group and frameworks like Business 20, Women 20, and G20
EMPOWER.
y The G20 Alliance for the Empowerment and Progression of Women›s Economic Representation (G20 EMPOWER)
is an initiative that aims to increase women›s leadership and empowerment in the private sector.
y The primary objective of this new Alliance is to bring together global best practices, knowledge sharing and invest-
ments in the identified areas of women’s health, education, and enterprise.
y Supported by the Bill and Melinda Gates Foundation, the alliance will be housed and anchored by the Confedera-
tion of Indian Industry (CII) Centre for Women Leadership.
Additional information:
y The World Economic Forum is the International Organization for Public-Private Cooperation.
y The Forum engages the foremost political, business, cultural and other leaders of society to shape global, regional
and industry agendas.
y The WEF was founded on 24th January, 1971 by German engineer Klaus Schwab. headquartered in Geneva.
CSE 2024 January 2024 9
DTRS Expalanations
y He introduced the concept of “stakeholder capitalism.”
y The WEF is best known for its annual meeting at Davos, Switzerland. The event attracts political and business lead-
ers from around the world for discussions about global issues.
Source: https://newsonair.gov.in/Main-News-Details.aspx?id=475571

10.
Answer: D
Context: India and UAE have initiated ‹Desert Cyclone 2024› joint military exercise in Rajasthan from 2nd January to 15th
January, 2024.
Explanation:
Given below is correctly matched pairs:
Military exercises Participating countries
1. Desert Cyclone India-UAE
2. Sampriti India-Bangladesh
3. KAZIND India-Kazakhstan
Additional information:
Desert Cyclone
y The exercise aims to enhance interoperability by learning & sharing best practices in Urban Operations.
y It is also expected to not only strengthen the defense ties between India and the UAE but also contribute to the
broader goal of fostering regional peace and security.
y India has also been a regular participant at the biennial International Defence Exhibition (IDEX) in Abu Dhabi.
Sampriti Exercise
y It is a bilateral defence cooperation exercise between the armies of India and Bangladesh. It is seen as a part of
Indo-Bangladesh defence cooperation.
y The eleventh edition of Sampriti is being held in October 2023 in Meghalaya.
y The aim of Sampriti is to enhance military cooperation between two countries in the wake of Indo-Bangladesh
defence cooperation.
y The exercise usually culminates into a final validation exercise where both the troops practice a Counter-Terrorism
operation in a controlled environment.
Kazind exercise
y It is a joint military exercise conducted between the armed forces of India and Kazakhstan.
y The exercise involves personnel from both the Indian Army and Indian Air Force, as well as members of the Kazakh-
stan Army and Air Force.
y It aims to strengthen mutual cooperation, enhance combat readiness, and promote interoperability between the
participating armed forces.
y The exercise spans over a specified period, involving various tactical and strategic military operations, including
counter-terrorism drills, aiming to strengthen the defense capabilities and collaboration between the two nations.
Source: https://www.hindustantimes.com/india-news/desert-cyclone-5-things-to-know-about-india-uae-joint-military-ex-
ercise-underway-in-rajasthan-101704424757820.html

CSE 2024 January 2024 10


DTRS Expalanations
11.
Answer: D
Explanation:
Context: Recently, the Similipal kai chutney made with red weaver ants by the tribal people of Odisha’s Mayurbhanj district
(Odisha) received the geographical identity tag. Hence, Statement 3 is correct.
Statement 2 and 3 are correct: They are also recognised as bio-control agents because they are aggressive and will prey on
most arthropods entering their territory. They protect a variety of tropical crops against insect pests, acting as an alternative
to chemical insecticides.
About the Similipal kai chutney special:
Ingredients: The key ingredient is the red weaver ant, also known as «kai pimpudi» in Odia. These ants are found in the
forests of Similipal, including the Similipal Tiger Reserve, Asia›s second-largest biosphere reserve.
Preparation: The tribal communities have traditionally prepared this chutney for generations. They collect the ants, remove
their wings, and grind them into a paste along with spices like ginger, garlic, green chilies, and salt. The grinding is done
manually on a stone mortar and pestle called a «sil batta.»
Taste and aroma: The chutney has a tangy, spicy, and slightly sour flavor with a distinct aroma. The ant paste adds a unique
umami flavor, while the spices bring in heat and complexity.
Nutritional value: Red weaver ants are surprisingly rich in protein, calcium, zinc, vitamin B-12, iron, magnesium, and po-
tassium. This makes the chutney a valuable source of nutrients, especially for tribal communities who rely on it as part of
their diet.
Medicinal properties: The chutney is also believed to have medicinal properties and is used to treat ailments like rheuma-
tism, skin issues, and stomach problems.
Source: https://www.downtoearth.org.in/news/food/mayurbhanj-s-red-ant-chutney-receives-gi-tag-why-this-is-import-
ant-for-nutritional-security-of-tribals-93707

12.
Answer: D
Explanation:
Context:
Warming oceans forced women in Zanzibar to switch from seaweed to climate-resilient sponge farming to stay afloat.
Statement 1 is correct: A sponge is a living animal which is made of loosely arranged cells that surround a skeleton of fibres.
Statement 2 is correct: Sponges provide homes for many other animals, plants, and microorganisms. In many cases, they
all work together in a mutual symbiotic relationship. Sea sponges exist in all oceans around the world and make up 20% of
the global silicon biological sink.
Statement 3 is correct: Sponges, unlike seaweed, possess remarkable resilience to climate change, require minimal main-
tenance, and command premium market prices,
Source: https://www.downtoearth.org.in/news/africa/saved-by-sponge-93521

13.
Answer: B
Context: The Defence Research and Development Organisation has launched Ugram, a state-of-the-art assault rifle of the
7.62 x 51 mm calibre.
CSE 2024 January 2024 11
DTRS Expalanations
Explanation: Statement 1 is incorrect and statement 2 is correct
About the rifle: It has been indigenously designed, developed and manufactured in collaboration with a private industry
partner, a Hyderabad-based Dvipa Armour India Private Limited.
The assault rifle has been designed for the operational requirements of armed forces, paramilitary and state police entities.
The rifle has an effective range of 500 metres and weighs less than four kilograms.
It will have to go through several internal tests, acceptance trials and user trials before it can be considered for induction.
There is a major shortfall of assault rifles in India.
As part of its trial, a set number of rounds will be fired from Ugram without stoppage, and accuracy and consistency checks
will be conducted.
Source: https://indianexpress.com/article/cities/pune/drdo-launches-indigenous-assault-rifle-for-armed-forces-9100894/

14.
Answer: B
Context: A recent study has revealed that bottled water of popular brands contain up to 100 times more nanoplastics than
previously estimated and could pose serious health risks.
Explanation:
Statement 1 is incorrect: Nanoplastics have a diameter that is less than 1μm (micrometer). On the other hand, microplas-
tics are small plastic pieces less than five millimeters long.
Statement 2 is correct: Due to its very small size, nanoparticles have been found to have health implications.
Nanoplastics pose a greater threat to human health than microplastics because they’re small enough to penetrate human
cells, enter the bloodstream and impact organs.
Nanoplastics can also pass through the placenta to the bodies of unborn babies.
About the study: Scientists have long suspected nanoplastics presence in bottled water, but lacked the technology to iden-
tify individual nanoparticles.
The researchers targeted seven common plastic types, including polyethylene terephthalate (PET), which many water bot-
tles are made from, and polyamide, often used in filters to purify water before it’s bottled.
They found 110,000 to 370,000 tiny plastic particles in each liter, 90% of them nanoplastics.

Source: https://www.hindustantimes.com/lifestyle/health/health-concerns-soar-as-new-research-exposes-high-levels-of-
nanoplastics-in-bottled-water-101704777394759.html
https://www.hindawi.com/journals/jnm/2022/6707819/fig2/

CSE 2024 January 2024 12


DTRS Expalanations
15.
Answer: D
Explanation:
Context: Recently, the “Transforming Energy Demand” White Paper has been released by the World Economic Forum
(WEF)
y Its goal is to find practical ways for companies and governments to speed up the global energy transition by taking
action on energy demand.
y At COP28, governments committed to tripling the world›s renewable energy capacity and doubling energy efficien-
cy improvement by 2030.
Key Findings
y Global Savings:
¾ Taking steps to manage energy demand can cut energy consumption by as much as 31%, resulting in annual
savings of up to $2 trillion.
y Action on energy consumption alongside supply:
¾ It›s crucial to tackle both energy demand and supply to lower the energy intensity of current and future
activities.
y Energy Needs:
¾ By 2050, the global population will increase by 2 billion, and the GDP is expected to double.
¾ Developing economies need sufficient and affordable energy to fuel their growth and accomplish development
goals.
Source: https://www.weforum.org/publications/transforming-energy-demand/

16.
Answer: D
Explanation:
Context: Hog deer are spotted for the first time in Rajaji Tiger Reserve (Uttrakhand).
Statement 1 is incorrect: Hog Deer populations are found in South and Southeast Asia, not in the arid regions of Central
Asia.
Statement 2 is incorrect: It has been categorised as the endangered as per the conservation status of the International
Union for Conservation of Nature (IUCN).
Statement 3 is incorrect: It has been provided protection under the Schedule I of Wildlife Protection Act, 1972.
Hog deer:
y Hog Deer is a small to medium-sized deer species native to South and Southeast Asia.
y It is known for its distinctive appearance and behavior, the hog deer holds ecological significance and has cultur-
al importance in the regions it inhabits.
y It is a small to medium-sized deer native to South and Southeast Asia.
y It is a distinctive reddish-brown coat with white spots, known for lowering heads like hogs when running.
y They are found in grasslands, swamps, and deciduous forests.

CSE 2024 January 2024 13


DTRS Expalanations
y They are distributed across South and Southeast Asia, including India, Nepal, Bangladesh, Myanmar, Thailand,
and Vietnam.
y Behavior and Diet:
¾ Crepuscular, active during dawn and dusk.
¾ browsers, feeding on grasses, herbs, and shrubs.
y Threats: Hunting, loss of habitat due to agriculture

Source: https://timesofindia.indiatimes.com/city/dehradun/in-a-first-hog-deer-spotted-inside-rajaji-tiger-reserve/arti-
cleshow/106588427.cms

17.
Answer: B
Context- A study has highlighted the potential for catastrophic disasters in future due to the melting of over 100 active per-
mafrost structures identified in the Jhelum basin. It has also argued that the active rock glaciers in Kashmir Himalayas also
hold significant volumes of water, which underlines the need to explore their hydrological potential further.
The Kashmir Himalayas are dotted with permafrost structures called ‘rock glaciers’, with significant ice volumes within.
Explanation:
Statement 1 is incorrect: River Jhelum, a major tributary out of five major tributaries of river Indus, ultimately merges with
river Indus in Pakistan. It is a west flowing river. The Jhelum (Vyeth in Kashmiri, Vetesta in Sanskrit and Hydaspes in Greek)
is the main waterway of the Kashmir valley.
Jhelum river originates from a magnificent spring called “Chashma Verinag” in the Kashmir Himalayas. Cheshma Verinag is
situated at the foot hills accommodating Banihal pass in the south eastern corner of the Kashmir Valley and flows towards
west through the Kashmir Valley.

CSE 2024 January 2024 14


DTRS Expalanations
Statement 2 is correct: Some of the major tributaries of the river Jhelum include Dudhganga, Rambiara, Lidder, Sandran
river, Bringi river and Arapath.
About Dudhganga river : It rises in the central part of Pir Panchal around mountains “Tala Kosi”. Their main sources are two
mountain Streams “Sungsafad” and “Yachera”. The total catchment of this stream is 165.8 Sq. Km. Dudhganga joins river
Jhelum on its left flank.
Statement 3 is correct: The Wular lake is a tectonic lake which is fed by river Jhelum.

Source: Over 100 active permafrost structures identified in Jhelum basin, can cause catastrophic disasters in future: Study
(downtoearth.org.in); WRIS website

18.
Answer: B
Context: Government of India has given a Call for Nominations under Rashtriya Vigyan Puraskar 2024 in the field of Science,
Technology, and Innovation.
Explanation: The Government of India has come out with a new set of National Awards in the field of Science, Technology
and Innovation known as “Rashtriya Vigyan Puraskar’’.

CSE 2024 January 2024 15


DTRS Expalanations
The objective of the Rashtriya Vigyan Puraskar (RVP) is to recognize the notable and inspiring contribution made by the
scientists, technologists, and innovators individually or in teams in various fields of science, technology and technology­led
innovation.
The Rashtriya Vigyan Puraskar shall be one of the highest recognitions in the field of science, technology, and innovation in
India. Scientists/ technologists/innovators working in government, private sector organizations or any individual working
outside any organization, who have made distinguished contributions in terms of path-breaking research or innovation or
discovery in any field of science, technology, or technology- led innovation shall be eligible for the awards.
Statement 1 is correct: The awards shall be given in following four categories:-
Vigyan Ratna (VR) award will recognize lifetime achievements & contributions made in any field of science and technology.
Vigyan Shri (VS) award will recognize distinguished contributions in any field of science and technology.
Vigyan Yuva-Shanti Swarup Bhatnagar (VY-SSB) award will recognize & encourage young scientists up to the age of 45 years
who made an exceptional contribution in any field of science and technology.
Vigyan Team (VT) award to be given to a team comprising of three or more scientists/researchers/innovators who have
made an exceptional contribution working in a team in any field of science and technology.
The Rashtriya Vigyan Puraskar shall be given in the following 13 domains, namely:
Physics, Chemistry, Biological Sciences, Mathematics & Computer Science, Earth Science, Medicine, Engineering Sciences,
Agricultural Science, Environmental Science, Technology & Innovation, Atomic Energy, Space Science and Technology, and
Others.
Statement 2 is correct: People of Indian Origin staying abroad with exceptional contributions benefiting the Indian commu-
nities or society shall also be eligible for the awards.
Statement 3 is incorrect: The awards shall be announced on 11th May 2024 (National Technology Day).
The Award Ceremony for all categories of awards will be held on 23rd August 2024 (National Space Day).
Source:
1. https://pib.gov.in/PressReleasePage.aspx?PRID=1995945
2. https://pib.gov.in/PressReleasePage.aspx?PRID=1959262

19.
Answer: C
Context: Recently, the Financial Intelligence Unit India (FIU IND) issued show-cause notices to 9 offshore virtual digital asset
service providers (VDA SPs). This was for “operating illegally” without complying with the provisions of the Prevention of
Money Laundering Act, 2002 (PMLA). It has also been written to the Secretary at the Ministry of Electronics and Informa-
tion Technology to block URLs of the mentioned entities.
Explanation:
Statement 1 is correct: It refers to any information or code or number or token. India’s Union Budget for 2022-23 for the
first time defined VDA in the newly-inserted clause (47A) under Section 2 of the Income Tax Act, 1961.
Statement 2 is correct: It can be transferred, stored or traded electronically.
Statement 3 is correct: Indian currency and foreign currency as defined under the Foreign Exchange Management Act,
1999, have been excluded from the ambit of VDAs.
Additional information:
y VDA provides a digital representation of value exchanged with the promise of having inherent value or functions
as a store of value or a unit of account. Thus it includes cryptocurrencies, DeFi (decentralised finance) and non-fun-
gible tokens (NFTs).

CSE 2024 January 2024 16


DTRS Expalanations
y According to the Financial Action Task Force (FATF), a virtual asset service provider (VASp) can be any natural or
legal person who conducts activities like carrying out transfer of virtual assets, administer them, or oversee their
sale by an issuer’s office.
y Tax in India – 30% income tax on gains made from cryptocurrencies and also rules regarding 1% tax deducted at
source on cryptocurrency came into effect.
y Compliance – Entities dealing in VDAs, crypto exchanges and intermediaries are considered as ‘reporting entity’
under PMLA and thus brought under anti-money laundering/counter financing of terrorism regulations.
y Obligations – To verify the identities of on boarded clients, and maintain records of their financial positions and
potentially suspicious transactions.
y Applicability – To all VDA SPs operating in India irrespective of physical presence.
y The entities though catering to a substantial part of Indian users were not getting registered and coming under the
Anti Money Laundering (AML) and Counter Financing of Terrorism Network (CFT) framework’.
y Currently, 31 VDA SPs have registered with FIU IND.
y Significance – It would ensure lack of anonymity and businesses not encountering multiple hurdles.
Source: https://www.thehindu.com/business/Economy/why-did-fiu-ind-act-against-virtual-asset-providers/arti-
cle67695217.ece

20.
Answer: D
Context: India’s GDP will grow by 7.3% in the current financial year (2023-24), slightly faster than the 7.2% growth in 2022-
23, As per the First Advance Estimates (FAEs) released by the Government in January 2024.
Explanation:
Statement 1 is incorrect: The FAE is presented at the end of the first week of January every year. They are only the first
estimates of growth for that financial year. The FAE are based on the performance of the economy over the first seven-odd
months, and the data are extrapolated to arrive at an annual picture. Moreover, the first 3 quarters mean the timeline till
December of any financial year for which data are released at the end of January or February first week. So, the statement
is incorrect. Only actual data of 6-7 months are used and extrapolated.
Statement 2 is incorrect: Second Advance Estimates (SAEs) data are generally released in the month of February of the cur-
rent financial year. It is the Revised Estimates (REs) data which is released in the month of May of the next financial year.
Statement 3 is incorrect: All the estimates, including Advance Estimates, Revised Estimates, Provisional Estimates and the
Actual figures are released by the Ministry of Statistics and Programme Implementation (MoSPI).
Source:https://indianexpress.com/article/explained/explained-economics/gdp-data-advance-estimates-9099092/

21.
Answer: D
Explanation:
Context: Recently, seven products from Odisha, ranging from the Similipal Kai chutney made with red weaver ants to
the embroidered Kapdaganda shawl, have bagged the coveted Geographical Indication (GI) tag in recognition of their
exclusivity to the state.
Statement 1 is correct:
y Kapdaganda Shawl:
¾ Woven and embroidered by the women of the Dongria Kondh tribe, the shawl reflects the rich tribal heritage
of the Dongria Kondhs.
CSE 2024 January 2024 17
DTRS Expalanations
y Dongria Kondh tribe is a particularly vulnerable tribal group (PVTG) in the Niyamgiri hills in Odisha’s Rayagada and
Kalahandi districts.
y It is embroidered on an off-white coarse cloth with red, yellow and green coloured threads, with each colour hold-
ing significance.
y Green symbolises the mountains and hills, and yellow stands for peace and happiness. Red stands as the symbol
of blood.
y The motifs in the shawls are mostly lines and triangles, believed to be a reflection of the importance of mountains
for the community.
y The shawl is worn by both men and women and the Dongrias gift it to their family members as a token of love and
affection.

Statement 2 is correct:
y The Geographical Indication Registry issues this tag under the Department of Industry Promotion and Internal
Trade, Ministry of Commerce and Industry.
Geographical Indications (GI) tags:
y GI tag is the abbreviation of Geographical Indications tags in India. It came into force with effect from 15th Sep-
tember 2003.
y A geographical indication (GI) is a sign used on products with a specific geographical origin and possesses qualities
or a reputation due to that origin.
y This tag is valid for 10 years and can be renewed.
y Geographical Indications protection is granted through the TRIPS Agreement.
y Geographical indications are typically used for agricultural products, foodstuffs, wine and spirit drinks, handicrafts,
and industrial products.
y The Gucchi mushroom, one of the most expensive mushrooms in the world, from the state of Jammu and Kashmir
was recently given the GI tag in India.
Statement 3 is correct:
y Lanjia Saura Painting:
¾ The painting, one of the oldest tribal art forms, is also known as Idital.
¾ The artworks are famous for their beauty, aesthetics, ritualistic association and iconography.
¾ The art form belongs to the Lanjia Saura community, a PVTG largely residing in the Rayagada district.
¾ These paintings are in the form of exterior murals painted on the mud walls of homes.
CSE 2024 January 2024 18
DTRS Expalanations
¾ White paintings figure over a crimson-maroon background.
¾ It is believed that the Lanjia Sauras paint their walls with Idital artworks to show gratitude to their deities and
forefathers, and also for the well-being of their community.
¾ Reflecting the love and affection of the primitive tribes for nature, they feature subjects like tribal humans,
trees, animals, birds, the Sun and the Moon.

Source: https://indianexpress.com/article/explained/explained-culture/odisha-gi-tag-products-significance-9098672/

22.
Answer: C
Explanation:
Context: The question of how life began has invited numerous hypotheses. One that has prevailed since ancient Greece is
panspermia. We will see in detail about panspermia.
Some key aspects of panspermia include:
y The panspermia theory suggests that life exists throughout the universe and can be distributed by space dust,
meteoroids, asteroids, comets, and planetoids.
However, the implications of panspermia for the origin of life are significant:
¾ Alternative origin of life:
◊ Panspermia proposes that life on Earth may have originated elsewhere in the universe, and its evolution
on Earth is due to the delivery of life by cosmic processes
◊ This challenges the widely accepted notion that life originated on Earth through natural processes.
¾ Interplanetary and interstellar life transfer:
◊ The theory implies that life could be transferred between planets, moons, and satellites, as well as
between stars and galaxies.
◊ This suggests that life might be more common in the universe than previously thought, as it would not be
limited to Earth or a specific solar system.
¾ Potential for life on other planets:
◊ Panspermia raises the possibility that life could exist on other planets, such as Mars, Venus, or even
extrasolar planets.
◊ This encourages the search for life beyond Earth and provides a rationale for exploring other celestial
bodies in the universe.
CSE 2024 January 2024 19
DTRS Expalanations
¾ Common origin of life:
◊ If panspermia is true, all life found throughout the solar system would have a common origin, as life
would be distributed by cosmic processes rather than evolving independently on each planet.
Panspermia theory:
y Panspermia is a fringe theory that proposes the existence of life throughout the universe, distributed by space
dust, meteoroids, asteroids, comets, and planetoids, as well as by spacecraft carrying unintended contamination
by microorganisms.
y The theory suggests that life did not originate on Earth but was brought here by cosmic processes.
y Panspermia has been a topic of philosophical debate for millennia, but it remained speculative until recent de-
cades.
Arguments against panspermia:
y Critics argue that panspermia does not answer the question of the origin of life and cannot be tested experimen-
tally.
y Mainstream scientists generally do not support this theory due to its lack of validation and the enormous distance
between stars, making interstellar panspermia an unlikely occurrence.
y However, the concept of panspermia continues to be explored, with some researchers working on mathematical
treatments of how life might migrate naturally.
Source: https://epaper.thehindu.com/reader

23.
Answer: A
Context: The ‘State of the Economy’ report of the Reserve Bank of India (RBI) said that inflation needs to align with the
target by the second quarter of the year and it is key for inclusive growth.
Highlights of the report:
The Indian economy is poised to sustain its momentum in 2024.
Balance sheets of financial institutions need to be strengthened and asset quality improved even further. The ongoing
consolidation of fiscal and external balance needs to continue.
The Indian economy has recorded stronger than expected growth so far in 2023-24, underpinned by a shift from consump-
tion to investment. This highlights the crowding-in effect of the government›s thrust on capex.
Source: https://economictimes.indiatimes.com/news/economy/indicators/rbis-state-of-the-economy-report-inflation-con-
trol-key-for-inclusive-growth/articleshow/106963472.cms?from=mdr

24.
Answer: B
Context: A declining trend has been witnessed on the arrival of birds from Siberia, Russia, Kazakhstan and other cold regions
to Punjab’s protected wetlands, including the Harike, known as Hari-ke-Pattan, in Tarn Taran for the past few years.
Explanation:
Statement 1 is incorrect: The Harike wetland is one of the largest wetlands in northern India. It is situated in Tarn Taran
district in Punjab.
y It was declared a Ramsar site in 1990.

CSE 2024 January 2024 20


DTRS Expalanations
y It is spread over 41 sq km, is an important stopover for migratory birds.

Statement 2 is correct: It stands on the confluence of the Beas and the Sutlej.
For 2023-24, it is estimated that around 40,000 to 50,000 migratory birds are estimated to have arrived at the Harike wet-
land so far.
Punjab has seven protected wetlands, including the Harike Wildlife Sanctuary, Nangal Wildlife Sanctuary, Ropar Conser-
vation Reserve, Kanjli Wetland (Kali Bein Conservation Reserve), Beas River Conservation Reserve and Keshopur-Miani
Community Reserve.
Source: https://www.tribuneindia.com/news/punjab/winged-guests-count-declining-at-harike-579156

25.
Answer: B
Context: In a move to make menstrual hygiene products more environmentally sustainable, scientists at Stanford Univer-
sity have reported a method to produce from sisal leaves a “highly absorbent and retentive material”.
Explanation:
Statement 1 is incorrect: The plant is native to Central America, where its fiber has been used since pre-Columbian times.
It is a xerophytic perennial leaf fiber yielding crop.
Statement 2 is correct: The stiff, green sword-like leaves have been used to make twine, cloth, and carpets. The plant itself
is also used to make mezcal, a distilled alcoholic beverage.
y Its leaves grow up to 2 m long. The lifespan of a sisal plant is about 7-10 years, during which it produces 200-250
usable leaves. Each leaf has about a thousand fibers that can be used to make ropes, paper, and cloth.
Statement 3 is correct: Like all succulents, which are plants with thickened parts to store more water, sisal has an uncanny
ability to store water and thrive in drought-prone areas.
About the research:
y After treating the sisal fibers with peroxyformic acid, the process proceeds by washing them first with a solution
of dilute sodium hydroxide and then water. The result is a wet pulp that is then dried and mechanically blended to
obtain a dry mass called a fluff pulp.

CSE 2024 January 2024 21


DTRS Expalanations

y When the researchers compared the absorption capacity of this fluff pulp to cotton obtained from commercially
manufactured sanitary napkins, or Cotton-CMP, they found that the fluff had the upper hand in standard absorp-
tion tests.
y The cultivation of sisal requires less water and is less environmentally damaging than cotton industries. Thus,
replacing cotton-CMP with sisal fluff pulp could make sanitary napkin production more environmentally sustain-
able in the long run.
Source: https://www.thehindu.com/sci-tech/science/sisal-leaves-sanitary-napkins-sustainable-manu-prakash-explained/
article67715759.ece#:~:text=Its%20leaves%20grow%20up%20to,ropes%2C%20paper%2C%20and%20cloth.

26.
Answer: C
Explanation:
Context: An international team of researchers recently unveiled ground-breaking findings on the fungus Aspergillus fumig-
atus, which can cause deadly disease in humans.
Statement 1 is incorrect: It is a species of fungus that causes diseases in humans.
Statement 2 is incorrect: It can be found throughout the environment, including in soil, plant matter, and household dust.
The fungus can also produce airborne spores called conidia. Most people can inhale many of these spores on a daily basis.
Source: https://phys.org/news/2024-01-secrets-disease-fungus-aspergillus-fumigatus.html

27.
Answer: D
Explanation:
Context: Recently, a new plant species of Polygonum genus has been discovered at Gol Pahadi Island of the Pench Tiger
Reserve (PTR) in Maharashtra during a survey.
A new plant species named Polygonum Chaturbhujanum has been discovered at Gol Pahadi island of PTR. It is a herb. Six
plant species endemic to India have also been found in Pench. They are Aegineta indica, Boerhavia crispa, Habenaria gib-
sonii var foetida, Iphigenia pallida, Petalidium barlerioides and Barleria gibsoniii
Hence option (a) is the correct answer.
Source: https://timesofindia.indiatimes.com/home/environment/new-plant-species-discovered-in-maharash-
tras-pench-tiger-reserve-forest-official/articleshow/106546436.cms?from=mdr
CSE 2024 January 2024 22
DTRS Expalanations
28.
Answer: C
Context: Two recent judgments by the Supreme Court, in crucial cases, have seen the Supreme Court bank on the govern-
ment’s assurances to set things on track.
The first came from a Constitution Bench headed by Chief Justice of India D.Y. Chandrachud, which decided to rest easy on
the back of an assurance given by the Solicitor General, representing the Union government, that Statehood will soon be
restored to Jammu and Kashmir.
In the second judgment, a three-judge Bench also led by the CJI took the word of the Solicitor General, this time repre-
senting the Securities and Exchange Board of India (SEBI), that its investigations against the Adani group will be completed
expeditiously.
However, Article 144 of the Constitution mandates that “all authorities, civil and judicial, in the territory of India shall act in
aid of the Supreme Court”.
Explanation:
Statement 1 is correct: The Constitution of India mandates civil and judicial authorities in the territory of India to act in aid
of the Supreme Court. This duty is enshrined in Article 144 of the Constitution.
All authorities, civil and judicial, in the territory of India shall act in aid of the Supreme Court.
While seemingly straight-forward, this article has various implications and is supported by other constitutional provisions
that give it context and teeth.
y This duty extends to all authorities, civil and judicial, within the territory of India. This includes government offi-
cials, lower courts, tribunals, and other bodies exercising public functions.
y The nature of «aid» can vary depending on the circumstances such as:
¾ Executing orders and decrees passed by the Supreme Court.
¾ Providing assistance in investigations or inquiries ordered by the Supreme Court.
¾ Furnishing information or documents sought by the Supreme Court.
¾ Taking any other action required to support the Supreme Court›s functioning and enforcement of its orders.
¾ Constitutional provisions strengthening Article 144:
Supremacy of the Constitution: Article 144 derives its force from Article 350, which declares the Constitution to be the
supreme law of India. This implies that all other laws and authorities must comply with the Constitution and its provisions,
including Article 144.
Binding nature of Supreme Court judgments: Article 141 states that the judgments of the Supreme Court are binding on all
courts in India. This further solidifies the duty of subordinate authorities to comply with Supreme Court orders, as acting in
aid often involves enforcing these judgments.
Contempt of Court: Disobeying or obstructing the Supreme Court or its processes constitutes contempt of court and can be
punishable with imprisonment or fines. This serves as a deterrent against non-compliance with Article 144.
Significance of Article 144:
y It upholds the Supreme Court's position as the apex judicial body in India, ensuring its effectiveness in carrying out
its constitutional duties.
y It promotes uniformity and consistency in the application of law across the country by ensuring lower authorities
adhere to Supreme Court precedents.
y It safeguards the rights of individuals by enabling them to seek effective remedies through the Supreme Court
against any violation of their fundamental rights or due process.
CSE 2024 January 2024 23
DTRS Expalanations
Statement 2 is correct: The Supreme Court holds the power to impose exemplary costs against a government in default, but
it›s not through statutes, but through its inherent powers and procedural guidelines.
Power to Award Costs: The Supreme Court›s power to award costs is not explicitly mentioned in the Constitution but flows
from its inherent power to regulate its own proceedings and ensure compliance with its orders. - This power is recognized
in various landmark judgments and procedural guidelines like the Supreme Court Rules, 2013.
Imposing Exemplary Costs: Exemplary costs are those exceeding the normal court fees and are awarded in exceptional
cases where the party›s conduct is deemed frivolous, vexatious, or intended to abuse the court process. In the case of a
government in default, exemplary costs can serve as a penalty for disobeying court orders, neglecting responsibilities, or
causing unnecessary delays in proceedings.
Recent Instances: Campaign for Judicial Accountability and Reforms vs. Union of India (2023): The Supreme Court im-
posed exemplary costs of Rs. 1 lakh on the petitioner, an NGO, for filing a «contemptuous» and «scandalizing» petition
against the judiciary.
Statement 3 is correct: In India, any civil or judicial authority that disobeys or obstructs a Supreme Court directive can face
contempt of court proceedings and potential penalties. This principle upholds the Supreme Court›s position as the apex
judicial body and ensures its effectiveness in fulfilling its constitutional duties.
Grounds for Contempt:
y Wilful Disobedience: Failing to comply with a Supreme Court order without any justifiable reason.
y Scandalizing the Court: Publishing or saying anything that undermines the authority, dignity, or impartiality of the
Supreme Court.
y Interfering with Judicial Proceedings: Obstructing or hindering the due course of proceedings in any court, includ-
ing the Supreme Court.
y Abuse of Court Process: Misusing the court›s resources for personal gain or malicious purposes.
Contempt Procedures:
y The Supreme Court can initiate contempt proceedings on its own motion or upon receiving a complaint.
y The accused authority has the right to defend themselves and present their explanation for non-compliance.
y The Court can hold hearings and examine evidence before determining whether contempt has been committed.
Penalties for Contempt:
y If found guilty, the Supreme Court can impose various penalties, including:
y Imprisonment: Up to six months for criminal contempt and indefinite for civil contempt until compliance is achieved.
y Fine: Up to Rs. 2,000 or any higher amount deemed appropriate.
y Public Apology: Ordering the contemnor to express remorse and apologize to the Court.
y Other Directions: The Court may issue additional directions to ensure compliance with its orders.
Significance of Contempt Power:
y Maintains the Supreme Court›s authority and ensures its decisions are respected and implemented.
y Promotes efficient administration of justice by preventing delays and disruptions in judicial proceedings.
y Protects the rights of individuals by deterring authorities from acting contrary to Supreme Court pronouncements
that safeguard fundamental rights.
Source: https://www.thehindu.com/news/national/two-crucial-judgments-saw-sc-take-the-government-at-its-word-on-
setting-things-right/article67716103.ece

CSE 2024 January 2024 24


DTRS Expalanations

CSE 2024 January 2024 25


DTRS Expalanations
29.
Answer: D
Context: The recently concluded COP28 summit in Dubai saw slower progress on the Mitigation Work Programme even as
issues like Loss and Damage, Global Stocktake grabbed the limelight.
Explanation:
Statement 1 is incorrect: The Mitigation Work Programme (MWP) was established at COP26, in Glasgow Summit, to ur-
gently enhance mitigation ambition and implementation in this critical decade.
As a multilateral platform backed by the legitimacy and convening power of the UNFCCC, the MWP could help to raise
awareness of available tools and solutions, build momentum behind relevant ongoing mitigation-related initiatives without
being prescriptive, and deliver more effective, targeted mitigation efforts across all fronts in the near-term.
Statement 2 is incorrect: The programme has seen pushback from developing countries who initially questioned how its
role differed from the Global Stocktake – itself a mechanism written in to scale up ambition.
At the UN’s mid-year climate conference in Bonn in June 2023, discussions on the MWP were stalled as developing countries
demanded financial support to be able to take on more mitigation ambition, as detailed in a proposal by the Like-Minded
Developing Countries bloc.
Source: https://www.downtoearth.org.in/blog/climate-change/cop28-recap-what-was-discussed-at-the-mitigation-work-
programme--93743

30.
Answer: A
Context: There have been allegations that enrollment to the Digi Yatra facility is being made at the designated airports
without the informed consent of the passengers. Although the Digi Yatra initiative is a voluntary option, the Central Gov-
ernment is seen aggressively promoting this facility through various kiosks at the entry of various airport terminals.
Explanation:
Statement 1 is correct: Digi Yatra uses facial recognition technology to provide for contactless, seamless movement of
passengers at various checkpoints at airports. This initiative aims to enhance passenger convenience, reduce waiting times,
and improve overall efficiency in air travel.
Statement 2 is incorrect: DigiYatra is a digital initiative introduced by the Ministry of Civil Aviation in India.
About the DigiYatra facility
y The Digi Yatra Foundation is a not-for-profit company and is the owner of the facility whose shareholders com-
prise the Airports Authority of India and the five private airports at Delhi, Bengaluru, Cochin, Hyderabad, and
Mumbai.
y It was launched in December 2022.
Source:https://www.thehindu.com/news/national/passengers-say-cisf-and-airport-staff-are-collecting-biometric-data-
without-consent/article67710134.ece

31.
Answer: A
Context: ISRO successfully tests fuel cells to generate power in space. Throughout the brief test duration onboard POEM, an
impressive 180 W of power was generated from Hydrogen and Oxygen gases stored in high-pressure vessels.

CSE 2024 January 2024 26


DTRS Expalanations
Explanation:
Statement 1 is correct:
About POEM (PSLV Orbital Experimental Module) Initiative:
y The POEM is a platform that will help perform in-orbit experiments using the final, and otherwise discarded stage
of ISRO’s workhorse rocket, the Polar Satellite Launch Vehicle (PSLV).
y The PSLV is a four-stage rocket where the first three spent stages fall back into the ocean, and the final stage (PS4)
— after launching the satellite into orbit — ends up as space junk.
y However, under POEM initiative, the spent final stage will be utilised as a “stabilised platform” to perform exper-
iments.
Statement 2 is incorrect:
About POEM3 experiment:
y This experiment was launched by ISRO on PSLV-C58/XPoSat in January, 2024.
y ISRO assessed a 100 W class Polymer Electrolyte Membrane Fuel Cell based Power System (FCPS) within its orbital
platform POEM3.
y The objective of the experiment was to assess Polymer Electrolyte Membrane Fuel cell operation in space and to
collect data to facilitate the design of systems for future missions.
y Throughout the brief test duration onboard POEM, an impressive 180 W of power was generated from Hydrogen
and Oxygen gases stored in high-pressure vessels. (not the entire power required to keep the platform floating
in space).
y Notably, Hydrogen fuel cells function by directly converting Hydrogen and Oxygen gases into electricity, water, and
heat, employing electrochemical principles rather than combustion reactions used in traditional generators.
Source:
1. https://www.businesstoday.in/technology/news/story/isro-successfully-tests-fuel-cells-to-generate-power-in-
space-411986-2024-01-05
2. https://pib.gov.in/PressReleasePage.aspx?PRID=1838324
32.
Answer: C
Context- Death of India’s oldest sloth bear in captivity in the Van Vihar national park, Bhopal, Madhya Pradesh.
Explanation:
SLOTH BEAR
Distribution: Native to India, Sri Lanka and Nepal; found in Western Ghats, Chattisgarh, Madhya Pradesh, West Bengal
and North-East India.

CSE 2024 January 2024 27


DTRS Expalanations
Habitat: moist and dry tropical forests, savannahs, scrublands and grasslands below 1,500 m (4,900 ft) on the Indian sub-
continent, and below 300 m (980 ft) in Sri Lanka›s dry forests.
y Not confined to upper altitudes of the Himalayas.
y Found in areas with forest cover and low hills bordering outer ranges of the Himalayas.
Diet: It is myrmecophagous as it feeds exclusively on ants and termites.
Characteristics:
y Does not hibernate.
y Solitary or occasionally seen in groups.
y Honey Bears: Very fond of honey.
y Unlike other bear species, they routinely carry their cubs on their backs.
y Sloth bears› nostrils can close completely, protecting the animals from dust or insects when raiding termite nests
or beehives.
y A cream-colored “V” or “Y” shape usually marks their chests. Their shaggy fur does not have an undercoat, so it
keeps them relatively cool in their native, warm climate and protects them from tropical insects.
Conservation Status:
y IUCN Red List: Vulnerable (VU)
y CITES listing: Appendix I
y Indian Wildlife Protection Act, 1972: Schedule I
Van Vihar National Park is a national park in Bhopal, the capital city of Madhya Pradesh in central India. Declared a
national park in 1979, it covers an area of about 4.45 km2. It has the status of a national park, but is developed and man-
aged as a modern zoological park, following the guidelines of the Central Zoo Authority.
Fauna commonly found in captivity in Van Vihar National Park:
y Sloth bear
y Gharial
y Bengal tiger
y Asiatic lion
y Python
y Mugger
y Crocodiles, etc.
Source: ‹India›s oldest› sloth bear in captivity dies at 36 in Bhopal zoo - The Hindu; LukmaanIAS notes

33.
Answer: B
Context: The greater one-horned rhinoceros has returned to central Assam’s Laokhowa-Burachapori Wildlife Sanctuary
after 40 years.
Explanation:
Pair 1 is correctly matched: Laokhowa Wildlife Sanctuary, in Assam, covers an area of around 70.13 sq km and lies on the
southern bank of the Brahmaputra River. It forms an integral part of the Laokhowa-Burachapori eco-system and is a noti-
fied buffer of the Kazairanga Tiger reserve.The sanctuary is home to the great Indian-one horned rhinoceros, elephants,
royal Bengal tigers, Asiatic water buffaloes and more than 225 species of birds.
CSE 2024 January 2024 28
DTRS Expalanations
y Large:scale encroachment in the 1980s led to the edging out of most animals from the Laokhowa-Burachapori
Wildlife Sanctuary
y The rhinos have returned within 1 year of the successful anti-encroachment operation in the region.

Pair 2 is incorrectly matched: The sanctuary takes its name from the Bhadra River, its lifeline. Bhadra Wildlife Sanctuary is
one of Karnataka’s prominent tiger sanctuaries and is home to over 30 tigers. But apart from the tiger, it is a great place to
sight and observe other mammals, reptiles, and more than 250 species of birds, many of which are endemic to the Western
Ghats.
The sanctuary is classified as a tropical moist forest — a major portion is moist deciduous forest while the rest is dry decid-
uous and semi-evergreen sholas, filled with trees of teak, rosewood, honne, (pterocarpus marsupium) matthi (terminalia
elliptica) and tadasalu (sterculia foetida).
Pair 3 is correctly matched: Kuldiha wildlife sanctuary is an integral part of Baleshwar district in Odisha . Apart from being
known as a coastal district, this wildlife sanctuary gives Baleshwar its identity. If one visits this sanctuary, he finds abode
amidst the hilly terrains and densely populated deciduous forests.
y The wildlife sanctuary was first established in 1984.
y Various animals inhabit the forest, including Tiger, Leopard, Elephant, Gaur, Sambar, Giant Squirrel, Hill Myna, Pea-
fowl, Hornbills, other migratory birds and reptiles.

CSE 2024 January 2024 29


DTRS Expalanations
Source: https://www.thehindu.com/sci-tech/energy-and-environment/rhinos-return-to-assam-wildlife-sanctuary-after-40-
years/article67713125.ece

34.
Answer: A
Explanation:
Context: Remote Bengal village to begin the New Year with a workshop on ancient indigenous art.
Statement 1 is incorrect.
y Sohrai Painting is a folk/tribal painting tradition that is mostly practised in the villages of Jharkhand’s Hazaribagh
area.
Statement 2 is incorrect.
y The colours used in paintings are red, yellow, black and white.
Statement 3 is correct.
y The paintings depict a matriarchal tradition in which the art form is passed down as a legacy to daughters by
their mothers; likewise, one of the key topics of these paintings is the mother-child bond.
Additional information:
y Traditionally, women of the household paint on the mud walls of their homes during the holiday of Sohrai,
which falls just a day after the Hindu celebration of Diwali.

y The colours used in this picture (red, black, yellow, and white) are natural earth colours foraged from the wild
or purchased from local merchants.
y Chewing twigs are used as paint brushes, while cloth rags are used to apply the base coat.
y Sohrai is derived from the Mundari term Soroi, which means “to lash with a stick.”

CSE 2024 January 2024 30


DTRS Expalanations
y The economy of the community relies heavily on domestic animals.
y They assist in agriculture as well as supplying milk and meat.
y Previously, a villager’s wealth was determined by the number of live animals he had.
y Cattle are the most essential domestic animals since they supply milk and are used in agriculture for ploughing
fields, as well as their dung being used as manure and plastering dwellings.
y This is why cows are held in such high regard among Hindu families that their manure is used in religious rites.
Source: https://www.thehindu.com/news/cities/kolkata/remote-bengal-village-to-begin-new-year-with-workshop-
on-ancient-indigenous-art/article67709259.ece

35.
Answer: B
Explanation:
Context: The recent floods in the Mulugu district of Telangana has led to a fresh discovery of paleolithic quartzite tools.

Historical Context and Significance


y Paleontologist sheds light on the historical period to which the stone axe belongs, identifying it as part of the Lower
Paleolithic period, dating back approximately 30 lakh (3 million) years ago.
y This period corresponds to the Early Stone Age, also known as the Old Stone Age, and spans around 33 lakh (3.3
million) years BC, persisting for about 10,000 years.

CSE 2024 January 2024 31


DTRS Expalanations
y The identification of these tools is based on various characteristics, including chipping style, material, and size.
y The heavy quartzite material and the large size of these tools align with the typical implements used by Paleolithic
hunter-gatherers.
y Such hand axes, globally discovered, were primarily utilized for activities such as cutting wood and hunting animals
for sustenance.
y Comparative Analysis with Previous Discoveries
y This discovery in Telangana augments the understanding of human settlements in the region and central India,
significantly pushing back the timeline.
y Reference is made to the Paleolithic site at Attirampakkam near Madras (Chennai), where bifacial hand axes made
of stone by early humans were found in 1863.
y These tools, estimated to be about 15 lakh (1.5 million) years old, are attributed to the Madras Hand-Axe Industry
or Madrasian Culture.
Source: https://www.thehindu.com/news/national/telangana/the-recent-floods-in-mulugu-district-telangana-dried-up-a-
stream-which-exposed-old-stone-age-tools-this-pushes-back-the-understanding-of-human-habitations-in-the-state-and-
central-india/article67709084.ece/amp/

36.
Answer: D
Context: Prime Minister paid tributes to social reformer Savitri Bai Phule on her Jayanti. In a social media post, he said she
inspired society with her compassion and courage and her contribution towards the nation is invaluable.
Explanation:
Savitri Bai Phule became the first female teacher in India (1848) and opened a school for girls with her husband, Jy-
otirao Phule. She went on to establish a shelter (1864) for destitute women and played a crucial role in groom-
ing Jyotirao Phule’s pioneering institution, Satyashodhak Samaj, (1873) that fought for equality of all classes.
Additional information:
Savitribai was born in Naigaon, a small village in the state of Maharashtra, India. As a young girl, Savitribai displayed a strong
sense of curiosity and ambition. Savitribai was married to Jyotirao Phule in 1840 at the age of nine and became a child bride.
She moved to Pune with him soon after.
Abala Bose was the daughter of Durga Mohan Das, the leader of Brahmo Samaj undefined and an advocate of the education
of girls on an equal footing to that of boys. Belonging to a Brahmo family, she received a liberal education at Bethune College
in Kolkata. Ironically, Bose is not known for her educational work or her activism. She is mainly remembered as the wife of
Jagdish Chandra Bose, a famous physicist and scientist. However, a peek into the history of women’s education in Bengal
reveals that she played a very significant role in the education of girls and Purdah women in villages of Bengal. Bose had a
very radical view of women’s education, which was much ahead of her time.
Rani Velu Nachiyar was the first queen to fight against the British colonial power in India. She is known by Tamils as Veera-
mangai. She was the princess of Ramanathapuram and the only child of Raja Chellamuthu vijayaragunatha Sethupathy and
Rani Sakandhimuthal of the Ramnad kingdom.
Rani Velu Nachiyar was trained in war match weapons usage, martial arts like Valari, Silambam (fighting using stick), horse
riding and archery. She was a scholar in many languages and she had proficiency with languages like French, English and
Urdu. She married the king of Sivagangai, with whom she had a daughter.
Kulsum Sayani is another forgotten social activist and educator. Sayani pioneered the adult education programme in Bom-
bay. She was part of the first National Planning Committee which was set up by the Congress government under the chair-
manship of Jawaharlal Nehru in Bombay in 1938.
CSE 2024 January 2024 32
DTRS Expalanations
Sayani travelled extensively to London, Paris, Geneva, Denmark, Baghdad, Cairo, Jerusalem and Karachi in the decade of
the 1950s. During her visits, she studied the educational system of these countries which informed her adult education
programme in India. To keep up the interest of the new literates, Sayani started editing and publishing a fortnightly journal
called Rahber in Hindustani language and three scripts viz., Devanagari, Urdu and Gujarati.
Source: https://newsonair.gov.in/News?title=PM-Modi-pays-tributes-to-social-reformer-Savitri-Bai-Phule-%26-Rani-
Velu-Nachiyar-on-their-Jayanti&id=474434

37.
Answer: A
Explanation:
Context: Sikki artisans suffer livelihood losses amid climate change and failed promise of government in providing financial
help in setting up of Sikki stalls in countrywide trade fairs held round the year.
Statement 1 is correct: It is a rich yellowish variety of reed grass, locally known as ‘Kaincha’ is called Golden Grass due to its
golden luster on drying. Scientifically called Chrysopogon zizanioides, it finds mention in ancient Sanskrit texts such as the
Ramayana, where it is referred to as viran, sugandhimool, ushir and nalad.
Statement 2 is incorrect: This grass grows indigenously in the Tarai regions of Uttar Pradesh and Bihar and has an average
height of 3-4 feet.
Statement 3 is correct: It is most known for is its handicraft. It is used to make traditional items such as multipurpose bas-
kets, ornaments, show pieces and many more utility items that are still considered valuable in rural India.
Source: https://www.downtoearth.org.in/blog/climate-change/-government-failed-us-sikki-artisans-suffer-livelihood-
losses-amid-climate-change-state-apathy-93681

38.
Answer: A
Explanation:
Context: The pangolin conservation project launched to protect pangolins has succeeded at Pench Tiger Reserve and Satpu-
ra Tiger Reserve, as the mammals have started to breed there.
Statement 1 is incorrect: The Indian pangolin, also called the thick-tailed pangolin, is native to the Indian subcontinent. They
are one of the most trafficked mammals in the world, despite an international ban on their trade. It lives in India (south of
the Himalayas), Bangladesh, Southern Nepal, Sri Lanka, and small parts of Pakistan. They are well adapted to desert regions
and prefer barren, hilly areas. Their habitat extends up to 2,500 feet above sea level. They have also been shown to survive
in various types of tropical forests, open land, grasslands, and in close proximity to villages.
Statement 2 is correct: It is an insectivore and a nocturnal mammal.
Statement 3 is incorrect: It is an endangered species under IUCN red list.
Source: https://www.freepressjournal.in/bhopal/mp-wildlife-pangolin-conservation-project-succeeds-in-state

39.
Answer: A
Context: A group of experts have appealed to the International Union for Conservation of Nature (IUCN) to reclassify the
status of the Northeast African Cheetah (Acinonyx jubatus soemmeringii), found in the Horn of Africa, to ‘endangered’
from ‘vulnerable’.

CSE 2024 January 2024 33


DTRS Expalanations
Explanation:
Statement 1 is incorrect: A group of cheetahs is called a coalition. Coalitions are typically made up of two to three male
cheetahs, and are usually established amongst brothers.
y Cheetahs create coalitions to protect their territory from males from other coalitions.
Statement 2 is correct: Cheetah females, on the other hand, are solitary. They spend much of their life alone, wandering
and hunting.
y Female cheetahs typically rear their offspring for two years before leaving them to form coalitions or live on their
own.
y Female cheetahs only interact with other cheetahs when they mate.
Statement 3 is incorrect: The IUCN status of African cheetah is ‘vulnerable’. For Asiatic cheetah, the status is ‘critically
endangered’.
‘Project Cheetah’
Under the project Cheetah, the government decided to reintroduce the cheetah into India, the only large carnivore species
that went extinct in Independent India. The Cheetahs for release into the selected sites in India are sourced from the South
African and Namibian populations, depending on the suitability of animals.
y The African cheetahs have been brought into the Kuno National Park (KNP), Madhya Pradesh.
y However, out of the total 20 cheetahs introduced last year, six adult cheetahs died.
y Wildlife experts associated with Project Cheetah are considering options of importing the next batch of big cats
from countries other than South Africa and Namibia.
y Recently, the environment minister announced the birth of three cheetah cubs on January 3, 2023.
y This is the second litter born at the KNP. The first litter was born to Jwala, in March 2023, who was also brought
from Namibia. Only one of the female cubs survived, the other three died due to dehydration and weakness.
y Till now, there were 15 cheetahs at KNP — seven males and females each and a cub. With the inclusion of the cubs,
the tally now stands at 18.
Source: https://www.downtoearth.org.in/news/africa/upgrade-horn-of-africa-cheetah-status-to-endangered-experts-ap-
peal-to-iucn-93711

40.
Answer: C
Context: The Union government recently approved the overarching scheme “PRITHvi VIgyan (PRITHVI)” of the Ministry of
Earth Sciences, for implementation during the period from 2021-26, at an overall cost of Rs. 4,797 crore.
Explanation:
The scheme encompasses five ongoing sub-schemes namely “Atmosphere & Climate Research-Modelling Observing Sys-
tems & Services (ACROSS)”, “Ocean Services, Modelling Application, Resources and Technology (O-SMART)”, “Polar Science
and Cryosphere Research (PACER)”, “Seismology and Geosciences (SAGE)” and “Research, Education, Training and Outreach
(REACHOUT)”.
YUVIKA scheme: YUVIKA («YUva VIgyani KAryakram,»), also known as the «Young Scientist Program” by the Indian Space
Research Organisation (ISRO), is specifically designed for school-age children who are interested in the newest develop-
ments in the space science and technology. This programme is run by the ISRO to teach younger children the fundamentals
of space technology, science, and applications. This has not been included under the PRITHVI scheme.
Earth System Sciences deal with all the five components of the earth system: atmosphere, hydrosphere, geosphere, cryo-
sphere, and biosphere and their complex interactions.
CSE 2024 January 2024 34
DTRS Expalanations
The major objectives of the PRITHVI scheme are:
y Augmentation and sustainance of long-term observations of the atmosphere, ocean, geosphere, cryosphere and
solid earth to record the vital signs of the Earth System and change.
y Development of the modeling systems for understanding and predicting weather, ocean and climate hazards and
understanding the science of climate change.
y Exploration polar and high seas regions of the Earth towards discovery of new phenomena and resources.
y Development of technology for exploration and sustainable harnessing of oceanic resources for societal applica-
tions.
y Translation of knowledge and insights from Earth systems science into services for societal, environmental and
economic benefit.
Source: https://pib.gov.in/PressReleasePage.aspx?PRID=1993366

41.
Answer: C
Context: The Ministry of Health and Family Welfare released the results of a survey conducted by the National Centre for
Disease Control (NCDC), mapping the patients treated on one to five days each at 20 tertiary care institutes across 15 States
and two Union Territories between November 2021 and April 2022.The survey attributed that main cause of antimicrobial
resistance is the excessive and inappropriate use of antibiotics.
Explanation:
What is antimicrobial resistance?
Antimicrobial Resistance (AMR) occurs when bacteria, viruses, fungi and parasites no longer respond to antimicrobial
medicines. As a result of drug resistance, antibiotics and other antimicrobial medicines become ineffective and infections
become difficult or impossible to treat, increasing the risk of disease spread, severe illness, disability and death.
Statement 1 is correct: Over half of the almost 10,000 hospital patients surveyed recently were given antibiotics to prevent
infection, rather than to treat it, amidst growing concerns about the rise in resistance to antibiotics. The vast majority of
patients surveyed – 94% – were given antibiotics before the confirmation of a definitive medical diagnosis of the precise
cause of infection.
Statement 2 is correct: The World Health Organization (WHO) has identified antimicrobial resistance as one of the top
threats to public health. Tuberculosis (TB) is a major contributor to antimicrobial resistance.
Source:https://www.thehindu.com/sci-tech/health/55-of-patients-were-prescribed-antibiotics-for-preventive-indica-
tions-45-for-therapeutic-indications-ncdc-survey/article67701666.ece

42.
Answer: A
Context: Ministry of Environment, Forest, and Climate Change submits proposals for Wetland City Accreditation under the
Ramsar Convention on Wetlands for cities of Indore, Bhopal and Udaipur.
Explanation:
y Ministry of Environment, Forest, and Climate Change (MoEF&CC) has submitted three nominations from India
for Wetland City Accreditation (WCA) scheme of Indore (Madhya Pradesh), Bhopal (Madhya Pradesh) & Udaipur
(Rajasthan) under the Ramsar Convention on Wetlands.
y These are the first three Indian cities for which nominations have been submitted for WCA based on the proposals
received from respective State Wetlands Authorities in collaboration with the Municipal Corporations.

CSE 2024 January 2024 35


DTRS Expalanations
About the nominated cities:
Indore:
y Founded by Holkars, Indore is the cleanest city in India and the recipient of India’s Smart City Award 2023 for its
best sanitation, water and urban environment.
y Sirpur Lake, a Ramsar Site in the city, has been recognised as an important site for water bird congregation and is
being developed as a Bird Sanctuary.
y A strong network of more than 200 wetland mitras is engaged in bird conservation and sensitizing the local com-
munity to protect Sarus Crane.

Bhopal:
y One of the cleanest cities in India that has proposed conservation zones around the wetlands in its draft City De-
velopment Plan 2031.
y Bhoj Wetland, Ramsar Site is the city’s lifeline, equipped with the world-class wetlands interpretation centre, Jal
Tarang.
y Additionally, the Bhopal Municipal Corporation has a dedicated Lake Conservation Cell. A network of more than
300 wetland mitras is engaged in wetland management and conservation of Sarus Crane.

Udaipur:
y Located in Rajasthan, the city is surrounded by five major wetlands, namely, Pichola, Fateh Sagar, Rang Sagar,
Swaroop Sagar, and Doodh Talai.
CSE 2024 January 2024 36
DTRS Expalanations
y These wetlands are an integral part of the city’s culture and identity, help maintain the city’s microclimate, and
provide a buffer from extreme events.

Wetland City Accreditation (WCA) scheme:


Launched:
y During the Ramsar Convention COP12 held in the year 2015 approved a voluntary Wetland City Accreditation
system under Resolution XII.10 which recognizes cities which have taken exceptional steps to safeguard their
urban wetlands.
Aim:
y The Wetland City Accreditation scheme aims to further promote the conservation and wise use of urban and
peri-urban wetlands, as well as sustainable socio-economic benefits for local populations.
More:
y Additionally, the Accreditation seeks to encourage cities that are close to and dependent on wetlands, primarily
Wetlands of International Importance, but also wetlands with other conservation category status, to develop and
strengthen a positive relationship with these valuable ecosystems.
y To be formally accredited, a candidate for the Wetland City Accreditation should satisfy the standards used to
implement each of the six international criteria mentioned in Operational Guidance for WCA of the Ramsar Con-
vention on Wetlands.
y This voluntary scheme provides an opportunity for cities that value their natural or human-made wetlands to gain
international recognition and positive branding opportunities for their efforts in demonstrating strong positive
relationships with wetlands.
Source: https://pib.gov.in/PressReleasePage.aspx?PRID=1993224

43.
Answer: C
Explanation:
Context: From Odisha’s Lanjia paintings to Arunanchal’s Wancho Wooden Craft to West Bengal’s Kai Chutney, a selection
of 17 products from four States bagged the coveted Geographical Indication tag.
CSE 2024 January 2024 37
DTRS Expalanations
States/UTs GI tagged Products

Odisha Lanjia Saura paintings, Dongaria Kondh shawl, Khajuri Guda, Dhenkanal Magji, Similipal
Kai Chutney, Nayagarh
Kanteimundi Brinjal, Koraput Kalajeera Rice

Arunachal Pradesh Handmade Carpets, Wancho Wooden Craft, Adi Kekir (Ginger)

Jammu and Kashmir Ramban Anardana

West Bengal Tangail Saree, Garad Saree, Korial Saree, Kalonunia Rice, Sundarban Honey

Gujarat Kachchhi Kharek

y A GI is a sign used on products that have a specific geographical origin and possess qualities or a reputation that
are due to that origin.
International Recognition:
y Under the Paris Convention for the Protection of Industrial Property, GI are covered as an element of Intellectual
Property Rights (IPRs).
y Also covered under the WTO’s Trade Related Aspects of Intellectual Property Rights (TRIPS) Agreement.
y Madrid Agreement and Lisbon Agreement
In India, GI registration is administered by the Geographical Indications of Goods (Registration and Protection) Act of
1999.
y It is used to identify agricultural, natural, or manufactured goods.
y GIs are registered by the Registrar of Geographical Indications (RGI), which is the Controller-General of Patents,
Designs, and Trade Marks.
y Nodal Ministry: Department for Promotion of Industry and Internal Trade, Ministry of Commerce and Industry.
Source: https://www.thehindu.com/news/national/gi-tags-for-over-17-products-from-odisha-arunachal-pradesh-
west-bengal-and-jk/article67707062.ece

44.
Answer: A
Context: Buxa Tiger Reserve (BTR) in West Bengal witnessed the return of a tiger for the second time in two years after a
23-year absence, sparking hope for a flourishing ecosystem and a potential resurgence of tiger populations.
Explanation:
Buxa National Park is a “low density” reserve and part of a larger tiger territory that stretches to Bhutan. The rivers Sankosh,
Raidak, Jayanti, Churnia, Turturi, Phashkhawa, Dima, and Nonani flow through the National Park.
Additional information:
y According to the National Tiger Conservation Authority (NTCA), the Reserve has corridor connectivity across the
border with the forests of Bhutan in the North; linkages with the Kochugaon forests (Assam) and Manas Tiger Re-
serve (Assam) in the East; and with the Jaldapara National Park in the West.

CSE 2024 January 2024 38


DTRS Expalanations
y The reserve›s connectivity serves as a vital link in promoting the migration and genetic diversity of Bengal tigers.
y Flora: Prominent tree species include Sal, Champ, Gamar, Simul, and Chikrasi, contributing to the reserve›s diverse
and vibrant ecosystem.
y Fauna: The primary wildlife species include the Asian Elephant, Tiger, gaur (Indian bison), Wild boar, Sambar, and
Wild dog (Dhole).
y Endangered species in Buxa Tiger Reserve encompass the Leopard cat, Bengal florican, Regal python, Chinese
Pangolin, Hispid hare, and Hog deer.
Source: https://indianexpress.com/article/cities/kolkata/tiger-zinda-hai-buxa-reserve-bengal-count-9093718/

45.
Answer: A
Context: The global nuclear order is under strain given the fact that the GNO created in the shadow of the Cold War has held
reasonably well so far, but is facing pressures under changing geopolitics.
Explanation: The International Atomic Energy Agency (IAEA) is the principal international organization responsible for pro-
moting safe, secure, and peaceful nuclear technologies. It plays a crucial role in monitoring and regulating nuclear activities
to ensure compliance with international agreements and the Global Nuclear Order.
The IAEA›s mandate includes verifying that countries are complying with their non-proliferation commitments, promoting
nuclear safety and security, and facilitating the peaceful use of nuclear energy.
Source: https://www.thehindu.com/opinion/lead/the-global-nuclear-order-is-under-strain/article67699339.ece

46.
Answer: B
Context: January 5 is observed as National Birds Day in some parts of the world. It is aimed at raising awareness for the
conservation of avian species that have been adversely affected by habitat destruction, reducing food choices and climate
change.
Explanation:
Recognised as the most argumentative bird, the Jungle Babbler couldn’t be named more aptly. Even its genus name ‘argya’
means to argue in Latin. Known as saath bhai (seven brothers) in Hindi, Odia, Bengali, Gujarati and many other Indian re-
gional languages; the Jungle Babbler groups are called Seven Sisters in English.
As some stealth army commandos, they help farmers combat all the notorious pests by feeding upon them — like gram pod
borers, grasshoppers, locusts, crickets and their larvae. Hence, rather than being just babblers, when in action, these birds
protect us from heavy crop losses.
Further, not just their diet but their appearance also proves to be helpful for our agro-ecosystem. For instance, their dull
brown-greyish plumage perfectly camouflages them with the soil where they pick up insects and grains. Both the male and
female partners display this identical dress.
Their yellow bill, pale-cream eyes, rounded wings, and long tail are other notable features which blend them with their
terrestrial background. In fact, the babblers remain low-flight birds and prefer hopping because their flight is weak. That is
also partly why they are nonmigratory; but are, however, found across the Indian Subcontinent.
It is surprising why they are categorised as ‘jungle’ birds when they are well-adapted to our cities and villages. Interestingly,
these socialites often fall prey to brood parasitism (laying one›s eggs in another bird’s nest) by Common Hawk Cuckoo and
Pied Crested Cuckoo.
CSE 2024 January 2024 39
DTRS Expalanations

Source: https://www.downtoearth.org.in/news/wildlife-biodiversity/national-birds-day-the-various-moods-colours-of-in-
dian-city-birds-through-the-eyes-of-an-amateur-ornithologist-93751

47.
Answer: B
Explanation:
Context:
As a first, the Forest Department in Odisha has planned a ‘nocturnal trail’ for people, especially students and researchers,
in the Chandaka-Dampara Wildlife Sanctuary.
About Chandaka-Dampara Wildlife Sanctuary:
y It lies partly within Khurda and partly in Cuttack Districts of Odisha State, and is in close proximity to the state
capital, Bhubaneswar.
y It is nestled in the Khordha uplands of the ‘North-Eastern Ghats’ biotic region.
Habitat: The sanctuary is home to a variety of habitats, including dry deciduous forests, sal forests, mixed bamboo forests,
and grasslands. These habitats provide a home for a wide variety of animals, including elephants, tigers, leopards, sloth
bears, deer, monkeys, and birds.
Vegetation: The floral diversity of the sanctuary is distributed in six types, which include secondary moist miscellaneous
semi-evergreen forests, moist Kangada (Xyliaxylocarpa) forests, Coastal Sal forests, thorny bamboo brakes, planted Teak,
and Eupatorium scrub.
Hence, option (b) is the correct answer.
Source: https://www.newindianexpress.com/states/odisha/2024/jan/01/forest-department-in-odishaplans-noctur-
nal-trail-in--dampara-wildlife-sanctuary-2646736.html

CSE 2024 January 2024 40


DTRS Expalanations
48.
Answer: A
Context: A recent CRISIL report has appreciated the strength of the gold-loan NBFCs which can survive competition from
the banks.
From the UPSC perspective, it becomes important to understand the basics of the gold-loan NBFCs/FIs.
Explanation:
Statement 1 is incorrect: A secured loan known as “gold loan” uses gold jewellery as collateral. In India, nationalized public
banks, private banks and some NBFCs are allowed to offer gold loans at competitive rates. These loans give the borrowers
more freedom to utilise the money for any purpose, say education, wedding expenses, etc. as compared to other loans
that have defined terms and restrictions on their usage.
Statement 2 is incorrect: While the gold-loan NBFCs and banks are regulated by the Reserve Bank of India, determination
of interest rate for gold loans is left to the discretion of the financial institutions (NBFCs/banks) itself. However, they must
adhere to the RBI-determined Loan to Value Ratio (LVR) for risk evaluation. LVR represents the maximum amount that can
be granted against the value of the jewellery pledged as security/collateral.
The RBI has raised the LVR from 75% to 90% in 2020 in the backdrop of the COVID-19 pandemic for loans against pledge of
gold ornaments and jewellery for non-agricultural purposes up to March 31, 2021.
Statement 3 is correct: There are certain rules regarding gold loans in India:
y If the loan is not repaid, the gold pledged as collateral will be sold at auction.
y Only the value of gold in jewellery is taken into consideration for assessing collateral security (not the value of
precious metals or stones etc. used in it).
y An individual or MSME can only use gold loan facility against physical collateral security, including specially pro-
duced gold coins.
y No NBFC or bank is permitted to offer gold loans secured by the sovereign gold bonds or exchange-traded funds
(ETFs).
y A bank will only offer a gold loan if the applicant has an account there. An NBFC is, however, not subject to the
same restrictions.
Source: Gold loans help NBFCs hold their own against traditional banks - The Economic Times (indiatimes.com); Rules Re-
garding Loans against Security of gold jewellery for NBFCs (enterslice.com)

49.
Answer: B
Context: NHAI signs MoU with NRSC for Development and Reporting of “Green Cover Index” for National Highways of India.
Explanation:
About Green Cover Index:
y The National Highways Authority of India (NHAI) has entered into a Memorandum of Understanding with the
National Remote Sensing Centre (NRSC) under the Indian Space Research Organization (ISRO) for a period of three
years to develop and report a «Green Cover Index» for the extensive network of National Highways in India.

CSE 2024 January 2024 41


DTRS Expalanations

Overall Objective:
y To capture the Green Cover Index region-wise for National Highways in the first assessment cycle.
y Subsequent annual cycles will focus on estimating the growth patterns of green cover for National Highways using
scientific techniques.
y The findings of the index will facilitate comparison and ranking of various National Highways for timely and pe-
riodic intervention.
The MoU:
y The prioritization of greening Highway Corridors has been a focal point for the Ministry of Road Transport and
Highways (MoRTH), since the initiation of the Green Highways Policy in 2015.
y Using the capabilities of emerging technologies to enhance in-situ data collection and supplement plantation man-
agement and monitoring, including performance audits conducted by NHAI, the NRSC will undertake a compre-
hensive pan-India estimation of green cover, referred to as the «Green Cover Index» for National Highways using
high-resolution satellite imagery.
y This innovative approach promises to be a robust and reliable mechanism, providing a time-saving and cost-effec-
tive solution to generate a macro-level estimate of the degree of greenness along National Highways.
y It will also facilitate targeted interventions in regions that are reported to lack adequate green cover.
Source: https://pib.gov.in/PressReleasePage.aspx?PRID=1992882

50.
Answer: A
Context: In a paper published in the Proceedings of the National Academy of Sciences, a team of scientists demonstrated
that kelp forests can do more than supply food to tiny, hungry crustaceans living in the sand. They can also influence the
dynamics of the sandy beach food web.
Explanation:
Statement 1 is incorrect: Kelp forests are underwater ecosystems formed in shallow water by the dense growth of several
different species known as kelp. Though they look very much like plants, kelps are actually extremely large brown algae.

CSE 2024 January 2024 42


DTRS Expalanations
Statement 2 is incorrect: Kelp thrives in cold, nutrient-rich water.
y Because kelp attaches to the seafloor and eventually grows to the water’s surface and relies on sunlight to generate
food and energy, kelp forests are always coastal and require shallow, relatively clear water.
y Kelps live further from the tropics than coral reefs, mangrove forests, and warm-water seagrass beds, so kelp for-
ests do not overlap with those systems.
Statement 3 is correct: Giant kelp is harvested from kelp forests and used as a binding agent in products like ice cream,
cereal, ranch dressing, yogurt, toothpaste, lotion and more.
y Some kelp species can measure up to 150 feet (45 m) long. If living in ideal physical conditions, kelp can grow 18
inches (45 cm) a day.
Source:
1. https://phys.org/news/2024-01-uncovers-synchrony-kelp-forests-beach.html
2. https://oceana.org/marine-life/kelp-forest/#:~:text=Kelp%20forests%20are%20underwater%20ecosys-
tems,actually%20extremely%20large%20brown%20algae.
51.
Answer: D
Context: The Government of India has launched ‘Prerana: An Experiential Learning program’, aiming to offer a meaningful,
unique, and inspiring experience to all participants, thereby empowering them with leadership qualities.
Statement 1 is incorrect: Prerana is a week-long residential program for selected students of class IX to XII.
y It is an experiential and inspirational learning program for students with the best-in-class technology where heri-
tage meets innovation.
y A batch of 20 selected students (10 boys and 10 girls) will attend the program, every week from various parts of the
country. Hence, it is not restricted to students belonging to the SC/ST community.
Statement 2 is incorrect: Department of School Education & Literacy, Ministry of Education has launched this programme.
y The curriculum of Prerana School prepared by IIT Gandhi Nagar is rooted in nine value based themes: Swabhiman
and Vinay, Shaurya and Sahas, Parishram and Samarpan, Karuna and Sewa, Vividhta and Ekta, Satyanishtha and
Shuchita, Navachar and Jigyasa, Shraddha aur Vishwas, and Swatantrata and Kartavya.
Source: https://pib.gov.in/PressReleseDetail.aspx?PRID=1993086

52.
Answer: B
Context: More than 1 crore people have been screened for Sickle Cell Disease (SCD) under the National Sickle Cell Anaemia
Elimination Mission. The National Sickle Cell Anemia Elimination Mission launched in 2023, aims to eliminate sickle cell
anemia from India by 2047.
Explanation:
Statement 1 is incorrect: SCD is a group of inherited red blood cell (RBC) disorders. RBCs contain he-
moglobin, a protein that carries oxygen and healthy RBCs are round. In SCD, the hemoglobin is abnor-
mal, which causes the RBCs to become hard and sticky and look like a C-shaped farm tool called a “sickle.
Statement 2 is correct: It can be treated by bone marrow or stem cell transplantation.
Additional information:
Symptoms:
y Symptoms of sickle cell disease can vary, but some common symptoms include:
CSE 2024 January 2024 43
DTRS Expalanations
y Chronic Anaemia: leading to fatigue, weakness, and paleness.
y Painful episodes (also known as sickle cell crisis): these can cause sudden and intense pain in the bones, chest,
back, arms, and legs.
y Delayed growth and puberty
Treatment:
y Blood Transfusions: These can help relieve anemia and reduce the risk of pain crises.
y Hydroxyurea: This is a medication that can help reduce the frequency of painful episodes and prevent some of the
long-term complications of the disease.
Government Initiatives to Tackle SCD:
y The government released technical operational guidelines for the prevention and control of sickle cell anaemia in
2016.
y The State Haemoglobinopathy Mission has been established in Madhya Pradesh to address the challenges in
screening and management of the disease.
Anaemin Mukt Bharat Strategy.
Source: https://pib.gov.in/PressReleasePage.aspx?PRID=1992340#:~:text=Sickle%20cell%20disease%20is%20a,2023%20
at%20Shahdol%2C%20Madhya%20Pradesh.

53.
Answer: D
Explanation:
Context: Recently, the Quality Council of India (QCI) and Khadi and Village Industries Commission (KVIC) signed an initial
pact to enhance the quality of the latter›s products, train artisans, and introduce ‹Made in India› label for Khadi.
The following are the schemes covered under the KVIC:
y Prime Minister’s Employment Generation Programme (PMEGP)
y Market Promotion Development Assistance (MPDA)
y Interest Subsidy Eligibility Certificate (ISEC)
y Workshed Scheme for Khadi Artisans
y Strengthening the infrastructure of existing weak Khadi institutions and assistance for marketing infrastructure
y Khadi Reform and Development Programme (KRDP)
y Scheme of Fund for Regeneration of Traditional Industries (SFURTI)
y Honey Mission
Khadi and Village Industries Commission (KVIC):
y The Khadi and Village Industries Commission (KVIC) is a statutory body established by an Act of Parliament.
y In April 1957, it took over the work of former All India Khadi and Village Industries Board.
The broad objectives that the KVIC has set before it are:
y The social objective of providing employment.
y The economic objective of producing saleable articles.
y The wider objective of creating self-reliance amongst the poor and building up of a strong rural community spirit.

CSE 2024 January 2024 44


DTRS Expalanations
The functions of the KVIC are as follows:
y To build a strategic reserve of raw materials and implements for supply to producers.
y To create common service facilities for processing raw materials as semi-finished goods and provisions for facilities
for marketing KVI products.
y To enhance the sale and marketing of Khadi and other products of village industries or handicrafts.
y To be responsible for encouraging and promoting research in the production techniques and equipment employed
in the Khadi and Village Industries sectors.
y To provide financial assistance to institutions and individuals for developing and guiding the Khadi and Village in-
dustries through the supply of designs, prototypes, and other technical information.
y To assure the genuineness of the products and to set standards for the quality of products from Khadi and Village
Industries.
Source: https://timesofindia.indiatimes.com/city/ahmedabad/qgi-kvic-mou-to-promote-khadi/article-
show/106529403.cms

54.
Answer: A
Explanation:
Context:
Statement 1 is correct. They have a thick layer of body fat and a water-repellent coat that insulates them from the cold
air and water.
Statement 2 is incorrect. They are found in the Arctic region only.

CSE 2024 January 2024 45


DTRS Expalanations
Statement 3 is incorrect.
They are categorised as Vulnerable as per the conservation status of International Union for Conservation of Nature
(IUCN).
Additional information:
y The largest bear in the world and the Arctic›s top predator, polar bears are a powerful symbol of the strength and
endurance of the Arctic.
y In the United States, Alaska is home to two polar bear subpopulations.
y Considered talented swimmers, polar bears can sustain a pace of six miles per hour by paddling with their front
paws and holding their hind legs flat like a rudder.
y Polar bears spend over 50% of their time hunting for food.
y A polar bear might catch only one or two out of 10 seals it hunts, depending on the time of year and other vari-
ables.
y Their diet mainly consists of ringed and bearded seals because they need large amounts of fat to survive.
y Polar bears rely heavily on sea ice for traveling, hunting, resting, mating and, in some areas, maternal dens.
y But because of ongoing and potential loss of their sea ice habitat resulting from climate change–the primary threat
to polar bears Arctic-wide–polar bears were listed as a threatened species in the US under the Endangered Species
Act in May 2008.
y As their sea ice habitat recedes earlier in the spring and forms later in the fall, polar bears are increasingly spending
longer periods on land, where they are often attracted to areas where humans live.
y The survival and the protection of the polar bear habitat are urgent issues for WWF.
y In October 2019, the International Union for the Conservation of Nature (IUCN) Polar Bear Specialist Group re-
leased a new assessment of polar bear populations showing that the number of polar bear subpopulations experi-
ence recent declines has increased to four, with eight populations still being data-deficient.
y The good news is that five populations are stable while two have been experiencing an upward trend.

Source: https://www.downtoearth.org.in/video/wildlife-biodiversity/in-a-first-a-polar-bear-has-died-of-avian-flu-in-
the-arctic-93723#:~:text=A%20death%20of%20a%20polar,the%20presence%20of%20a%20virus.

55.
Answer: C
Context: The IMF has raised concerns about the long-term sustainability of India’s debts. It also reclassified India’s exchange
rate regime, terming it a “stabilised arrangement” instead of “floating”. Both these observations by the International Mon-
etary Fund (IMF) sparked reactions from the Indian Government.

CSE 2024 January 2024 46


DTRS Expalanations
Explanation:
The Global Financial Stability Report provides an assessment of the global financial system and markets, and addresses
emerging market financing in a global context. It focuses on current market conditions, highlighting systemic issues that
could pose a risk to financial stability and sustained market access by emerging market borrowers. The Report draws out the
financial ramifications of economic imbalances highlighted by the IMF›s World Economic Outlook.
Additional Information:
y The IMF was set up along with the World Bank after the Second World War to assist in the reconstruction of
war-ravaged countries. The two organizations agreed to be set up at a conference in Bretton Woods in the US.
Hence, they are known as the Bretton Woods twins.
y Created in 1945, the IMF is governed by and accountable to the 189 countries that make up its near-global mem-
bership. India joined on 27th December 1945.
y The IMF›s primary purpose is to ensure the stability of the international monetary system — the system of ex-
change rates and international payments that enables countries (and their citizens) to transact with each other.
y The Fund›s mandate was updated in 2012 to include all macroeconomic and financial sector issues that bear on
global stability.
Reports by IMF:
y Global Financial Stability Report.
y World Economic Outlook.
Source: https://www.thehindu.com/opinion/op-ed/the-dispute-on-indias-debt-burden/article67701846.ece

56.
Answer: B
Explanation:
Context:
India and Pakistan exchanged the list of nuclear installations and facilities through diplomatic channels in New Delhi and
Islamabad on January 01, 2024.
Statement 1 is correct & 2 is incorrect: Agreement on the Prohibition of Attack against Nuclear Installations and Facilities:
It was signed on December 31, 1988, by the then Pakistani Prime Minister Benazir Bhutto and Indian PM Rajiv Gandhi. The
treaty came into force on January 27, 1991, and has two copies each in Urdu and Hindi. In 1986, the Indian army carried out
a massive exercise ‘Brasstacks’, raising fears of an attack on nuclear facilities. Since then, both countries have been negoti-
ating to reach an understanding towards the control of nuclear weapons, which culminated in the treaty.
Statement 3 is correct: The agreement mandates both countries to inform each other about any nuclear installations and
facilities to be covered under the agreement on the first of January of every calendar year, providing a confidence-building
security measure environment.
Source: https://economictimes.indiatimes.com/news/india/india-pakistan-exchange-list-of-nuclear-installations/video-
show/106457836.cms?from=mdr

57.
Answer: C
Explanation:
Context: A two-month-old girl with bubble baby syndrome underwent bone marrow transplant (BMT) in Mumbai recently.
Statement 1 is correct: Severe Combined Immunodeficiency (SCID) is very rare genetic disorder that causes life-threatening
problems with the immune system. It is a type of primary immune deficiency.

CSE 2024 January 2024 47


DTRS Expalanations
Statement 2 is correct: SCID is a «combined» immunodeficiency because it affects both of these infection-fighting white
blood cells.
Statement 3 is correct: SCID is a pediatric emergency. Without treatment, babies are not likely to survive past their first
birthday. The most common treatment is a stem cell transplant (also called a bone marrow transplant). This means the child
receives stem cells from a donor. The hope is that these new cells will rebuild the child›s immune system.
Source: https://www.deccanherald.com/india/two-month-old-becomes-the-youngest-to-get-bone-marrow-transplant-in-
mumbai-2830602

58.
Answer: B
Context: The Union Cabinet in January 2024 approved Rs 1,250 crore to the multinational Square Kilometer Array (SKA)
project, whose telescope arrays or groups of telescopes will be built in Australia and South Africa.
Explanation:
Statement 1 is incorrect: The Square Kilometre Array (SKA) is an intergovernmental international radio telescope project
being built in Australia (low-frequency) and South Africa (mid-frequency). This is because in the southern hemisphere, the
view of the Milky Way galaxy is the best and radio interference at its least.
Statement 2 is correct: It will have a total collecting area of approximately one square kilometre. It will operate over
a wide range of frequencies and its size will make it 50 times more sensitive than any other radio instrument. If built as
planned, it should be able to survey the sky more than ten thousand times faster than before. With receiving stations ex-
tending out to a distance of at least 3,000 km from a concentrated central core, it will exploit radio astronomy›s ability to
provide the highest-resolution images in all astronomy.
Statement 3 is correct: The USA is not one of the members of the SKA project.

Member countries of the SKA Project:


y Australia: Department of Industry and Science
y Canada: National Research Council
y China: National Astronomical Observatories of the Chinese Academy of Sciences
CSE 2024 January 2024 48
DTRS Expalanations
y France: French National Centre for Scientific Research
y Germany: Max-Planck-Gesellschaft
y India: National Centre for Radio Astrophysics
y Italy: National Institute for Astrophysics
y Portugal: Portugal Space
y South Africa: National Research Foundation
y Spain: Institute of Astrophysics of Andalusia
y Sweden: Onsala Space Observatory
y Switzerland: École Polytechnique Fédérale de Lausanne
y The Netherlands: Netherlands Organisation for Scientific Research
y United Kingdom: Science and Technology Facilities Council
More information on the SKA project
The SKA will combine the signals received from thousands of small antennas spread over a distance of several thousand
kilometres to simulate a single giant radio telescope capable of extremely high sensitivity.
The frequency range from 50 MHz to 14 GHz cannot be realised using one design of antenna and so the SKA will comprise
separate sub-arrays of different types of antenna elements that will make up the SKA-low, SKA-mid and survey arrays.
Source: https://indianexpress.com/article/india/another-eye-in-sky-on-ground-india-is-now-part-of-worlds-largest-ra-
dio-telescope-project-9092577/

59.
Answer: A
Explanation: Only the second condition is correct.
Context:
India is one of the fastest-growing economies in the world, driven by a young population, rising disposable incomes, and
increasing urbanization. India increased investment opportunities across various sectors like infrastructure, technology, and
consumer goods and benign monetary policy has kept interest rates stable, fostering investment and growth.
The RBI›s six-member MPC hiked the repo rate to 6.50 percent, as expected, and maintained the stance to «withdrawal of
accommodation». This comes as retail inflation fell below the RBI›s 6 percent threshold for three months in a row and the
US Fed is slowing down its increase in interest rates.
y RBI Monetary Policy refers to the policy of the Reserve Bank of India (RBI), the central bank of India, which deals
with the control and regulation of the money supply in the Indian economy. This policy is aimed at maintaining
price stability, employment and economic growth of the country.The RBI uses various monetary policy instruments
to achieve these objectives. The primary instruments used by the RBI to control money supply are open market
operations, cash reserve ratio, statutory liquidity ratio and repo rate.
y A repo rate is a fundamental tool used by the Reserve Bank of India (RBI) to control and regulate the money
supply in India’s economy. It is the rate at which commercial banks borrow funds from the central bank of
India for short-term financial needs.This rate is primarily used to control inflation and ensure economic sta-
bility. As it is a benchmark rate, it also affects other interest rates like those on fixed deposits, home loans,
car loans and personal loans.
y When the repo rate goes up, borrowing from RBI becomes more expensive and vice versa. The general public may
not feel the impact of a rise or fall in the repo rate directly, but they will feel the effect through the changes in the
interest rate charged by banks on their products. Banks usually increase their lending rates when the repo rate
increases and reduce them when the repo rate decreases.

CSE 2024 January 2024 49


DTRS Expalanations
y For real estate developers, the repo rate hike will increase the cost of borrowing, making it more expensive to fund
new projects or refinance existing debt. This, in turn, could affect project costs. Developers may also be forced to
raise property prices to cover the higher borrowing costs, which can negatively impact demand for housing. For
homebuyers, the repo rate hike will increase the cost of home loans, making it more expensive to buy a property.
This can lead to a decrease in demand for housing, particularly in markets where property prices are already high.
Homebuyers may also be forced to postpone their purchase plans, which can lead to a slowdown in the real estate
market.
y For the common man, a hike in the repo rate has a direct effect on his pocket. It increases the cost of borrowing
from banks and financial institutions. The loan EMIs, credit card payments and other loan payments become ex-
pensive. However, in contrast, fixed deposits become appealing in a rate increase situation since banks often raise
interest rates on this type of investment vehicle. Hence option (b) is the correct answer.
y A hike in the repo rate also affects inflation. It decreases the money supply in the market and reduces demand
for goods and services. This acts as a deflationary pressure and can lead to lower inflation. Lower inflation means
lower prices and more savings for the common man.Thus, a hike in the repo rate has both positive and negative
effects for the common man. On one hand, it leads to higher loan payments, whereas, on the other hand, it helps
in controlling inflation and encourages saving, leading to more money in the pocket.
y Source: https://www.livemint.com/money/personal-finance/investors-need-to-have-a-balanced-approach-this-
year-11704124640923.html
60.
Answer: A
Context: Government plans to end the Free Movement Regime along the international border with Myanmar.
Explanation:
Statement 1 is correct:
y Under the FMR, all the hill tribes, whether they are citizens of India or Myanmar, can travel within 16 km on
either side of the Indo-Myanmar Border (IMB).
y The FMR was implemented in 2018 as part of the Central government’s Act East policy.
y FMR is implemented by both governments for the people living along the IMB.
y This helps locals to get more culturally assimilated with trans-border villages through weddings, celebrating com-
mon festivals together and trans-border trade.
y It is a reflection of the physical, ethnic, linguistic, cultural and fraternal linkages among the trans-border villagers.
y Statement 2 is incorrect:
y The FMR is a mutually agreed arrangement between the two countries that allows tribes living along the border to
travel up to 16 km inside the other country without a visa.
y They can cross the border by producing a border pass with a one-year validity issued by the competent authority
and can stay up to two weeks per visit.
Statement 3 is incorrect:
Indo-Myanmar Border (IMB):
y It runs for 1,643 km in the four states of Mizoram, Manipur, Nagaland, and Arunachal Pradesh.
y It runs from the tripoint with China in the north to the tripoint with Bangladesh in the south.
y Assam Rifles is tasked with guarding the IMB.
Source: https://indianexpress.com/article/india/govt-plans-to-end-free-movement-regime-along-international-border-
with-myanmar-9092589/#:~:text=The%20border%20between%20India%20and,other%20country%20without%20a%20
visa.
CSE 2024 January 2024 50
DTRS Expalanations

CSE 2024 January 2024 51


DTRS Expalanations
61.
Answer: C
Context: The upcoming BRICS summit in Russia this October is set to undergo a significant transformation as it expands its
membership to include five new nations alongside the original founding five.
Explanation:
About BRICS
The leaders of BRIC (Brazil, Russia, India, and China) countries met for the first time in St. Petersburg, Russia, on the margins
of the G8 Outreach Summit in July 2006.
South Africa joined in 2010, making it «Brics».
Over a period of time, BRICS countries have come together to deliberate on important issues under the three pillars of
political and security, economic and financial and cultural and people to people exchanges.
The New Development Bank (NDB) is a multilateral development bank established by Brazil, Russia, India, China and South
Africa (BRICS) with the purpose of mobilizing resources for infrastructure and sustainable development projects in emerging
markets and developing countries (EMDCs).
Egypt, Ethiopia, Iran, Saudi Arabia and the United Arab Emirates have joined it recently, making it a 10-member bloc.

Source:
1. https://ddnews.gov.in/international/brics-welcomes-five-new-members-signaling-global-shift#:~:text=This%20
expansion%2C%20featuring%20Egypt%2C%20Ethiopia,the%20established%20Western%2Dled%20order.
2. https://www.bbc.com/news/world-66525474

CSE 2024 January 2024 52


DTRS Expalanations
62.
Answer: C
Context: The Central Council for Research in Ayurvedic Sciences (CCRAS) along with National Commission for Indian System
of Medicine (NCISM) has launched ‹SMART 2.0› (Scope for Mainstreaming Ayurveda Research among Teaching profession-
als) program.
Explanation:
Statement C is correct: The program is aimed at promoting robust clinical studies in priority areas of Ayurveda with Ayurve-
da academic institutions/hospitals across the country through mutual collaboration.
The objective of ‘SMART 2.0’ is to generate tangible evidence to demonstrate efficacy and safety of Ayurveda interventions
using interdisciplinary research methods and translating it into public health care.
The study would ensure safety, tolerability and adherence to Ayurveda formulations in the priority research areas of Bal
Kasa, malnutrition, insufficient lactation, Abnormal Uterine Bleeding, Osteoporosis in post-menopausal women and Diabe-
tes Mellitus (DM) II.
Under the earlier program launched, ‘SMART 1.0’, around 10 diseases were covered with the active participation of teaching
professionals from 38 colleges.
Source: https://www.thehindu.com/news/national/smart-20-launched-for-ayurveda-teaching-professionals/arti-
cle67701814.ece

63.
Answer: A
Context: The Pegasus spyware has once again ignited a debate on privacy and security. Recent reports by Amnesty Interna-
tional point to its utilization in targeting the phones of prominent Indian journalists. Security Lab concluded that a message
to facilitate a “zero-click exploit” had been sent to their phones.
Explanation:
y A zero-click exploit is malicious software that allows spyware to be installed on a device without the device own-
er’s consent. It doesn’t require the device owner to perform any actions to initiate or complete the installation
unlike the regular apps that require a user to click ‘install’, ‘confirm’, etc. to complete an installation.
y The specific exploit allegedly in use on the devices is called “BLASTPAST” (previously identified as “BLASTPASS”).
y It plays out in two phases. In the first, the attack attempts to establish a link with the Apple HomeKit which gives
users a way to control multiple smart devices on the target’s device.In the second, some malicious content is sent
via the iMessage app to the target.
y According to Amnesty, the purpose of the first phase – the ‘outreach’ – could be to determine how the device can
be exploited or to keep it in sight for further exploitation in the future. The second phase is the one that delivers
the full spyware “payload”.
Additional information:
About Pegasus Spyware:
y Pegasus spyware is a highly invasive mobile surveillance tool that can secretly infiltrate and monitor smartphones,
collecting data and information from various apps and sources.
y It was developed by the Israeli cyber-intelligence firm NSO Group, which claims to sell it only to government agen-
cies for fighting crime and terrorism.
y NSO emphasizes mechanisms in place to avoid targeting journalists, lawyers, and human rights defenders not in-
volved in terror or serious crimes.

CSE 2024 January 2024 53


DTRS Expalanations
Operating Procedure:
y Pegasus uses “zero-click” methods to infect devices and it is a malicious software that allows spyware to be in-
stalled on a device without the device owner’s consent.
y The spyware doesn›t necessitate any user actions for installation, distinguishing it from regular apps that require
explicit user confirmation.
y It can exploit vulnerabilities in apps such as WhatsApp, iMessage, or FaceTime, and send a message or a call that
triggers the installation of the spyware, even if the user does not open or answer it.
y Pegasus is a spyware that can exploit zero-day vulnerabilities to deploy spyware on Apple products.
y A zero-day vulnerability is an undiscovered flaw or bug in an operating system that the mobile phone’s manufac-
turer does not yet know about and so has not been able to fix.
Targets:
y Several investigations and reports have revealed that Pegasus spyware has been used to spy on journalists, human
rights activists, lawyers, opposition leaders, and heads of state.
y Some of the countries that have been accused of using Pegasus spyware to target their critics and enemies include
Saudi Arabia, Mexico, India, Morocco, Hungary, Azerbaijan, and Rwanda.
Source: https://www.thehindu.com/news/national/is-pegasus-spyware-targeting-journalists-in-india/article67683399.
ece

64.
Answer: A
Context: 7.5 magnitude earthquake in Japan affected the Ishikawa prefecture and Noto Peninsula.
Explanation:
Noto peninsula—Japan

It makes up the northern half of Ishikawa Prefecture, extending about 100 kilometers into the Sea of Japan. The peninsula is
known for its coastal scenery, particularly along the Okunoto Coast and the Kongo Coast, as well as for its rural atmosphere.
It was designated as a part of Globally Important Agricultural Heritage Systems by the FAO.
Shakotan peninsula—Japan
Located in the western Hokkaido, the Shakotan peninsula extends into the Sea of Japan.

CSE 2024 January 2024 54


DTRS Expalanations

Sinai peninsula—Egypt
It acts as a land bridge between Asia and Africa. It is a triangle-shaped peninsula located in northeastern Egypt. It is the
sovereign territory of Egypt.
The peninsula is bordered to the north by the Mediterranean Sea and to the east by Israel and the Gaza Strip.
To the west of the Sinai Peninsula is the Suez Canal, across which lies the African part of Egypt.
The Sinai is bordered to the southwest by the Gulf of Suez and to the immediate south by the Red Sea.
The Gulf of Aqaba borders the Sinai in the southeast.

CSE 2024 January 2024 55


DTRS Expalanations
Egypt shares maritime borders in the Sinai with Jordan and Saudi Arabia.
History of Sinai peninsula:
In the late 19th century, Egypt, including the Sinai Peninsula, became part of the British Empire.
British rule in Egypt would last until 1922, when the country was granted independence.
The peninsula was occupied by Israeli forces during the Six-Day War of June 1967.
It was returned to Egypt in 1982 under the terms of the peace treaty concluded between those countries in 1979.
Today, the Sinai is mostly inhabited by Arab Egyptians and Bedouins.

Source: Japan quake toll rises to 62 as weather hampers rescuers - The Hindu; Japan Earthquake: Race Against Time for
Rescue Amid Aftershocks and Harsh Conditions - The Hindu

65.
Answer: D
Explanation:
Context: Recently, India, the world›s third-largest energy consumer, made its first-ever payment in rupees for crude oil
purchased from the United Arab Emirates (UAE), signalling what could be a strategic push to promote the local currency
globally. The move is part of India›s broader efforts to diversify oil suppliers, cut transaction costs, and establish the rupee
as a viable trade settlement currency. This initiative aligns with the Reserve Bank of India›s move on July 11, 2022, allowing
importers to pay in rupees and exporters to receive payments in the local currency.
Currency Internationalization:
y Currency internationalization is the widespread use of a currency outside the borders of its original country of
issue.
y The level of currency internationalization for a currency is determined by the demand that users in other countries
have for that currency.
y This demand can be driven by the use of the currency to settle international trade, to be held as a reserve currency
or a safe-haven currency, or in general use as a medium of indirect exchange in other countries› domestic econo-
mies via currency substitution.

CSE 2024 January 2024 56


DTRS Expalanations
Benefits of internationalization of Rupee:
y Reduced Dependency on Foreign Currencies:
¾ Internationalisation of the rupee would reduce India›s reliance on foreign currencies, such as the US dollar,
for international trade and financial transactions.
¾ This would enhance India›s economic sovereignty and reduce exposure to currency fluctuations.
y Increased Global Trade:
¾ Internationalisation of the rupee can facilitate smoother international trade by allowing parties to transact
directly in rupees.
¾ It would eliminate the need for currency conversions, reducing transaction costs and simplifying cross-border
trade.
y Enhanced Financial Integration:
¾ A globally recognised and widely used rupee can lead to increased financial integration.
¾ It would attract foreign investors and promote capital inflows, leading to greater investment opportunities
and liquidity in the Indian financial markets.
y Improved Monetary Policy Effectiveness:
¾ Internationalisation can enhance the effectiveness of India›s monetary policy.
¾ With a broader international reach, the Reserve Bank of India (RBI) can use the exchange rate as a tool to
manage inflation and stimulate economic growth.
¾ It provides greater flexibility in managing monetary conditions and responding to economic challenges.
y Strengthened Regional Influence:
¾ A globally accepted rupee can strengthen India›s regional influence and position it as a major economic player
in Asia.
¾ It would promote trade and investment within the region, bolstering economic partnerships and collaborations.
Challenges in Internationalisation of rupee
y Reduced autonomy over Monetary Policies.
y May result in increased volatility in rupee’s exchange rate in initial stages.
y India operates a partially convertible capital account, which allows limited rupee exchange.
Initiatives taken for Internationalisation of Rupee
y In July 2022, RBI allowed trade settlement in rupees through Special Vostro Accounts.
y Allowing issuance of offshore Rupee denominated ‘masala’ bonds.
Source: https://www.business-standard.com/economy/news/india-makes-first-ever-rupee-payment-for-the-purchase-
of-crude-oil-from-uae-123122600124_1.html

66.
Answer: B
Explanation:
Context: Kyrgyzstan, a Central Asian country, has designated the snow leopard as its national symbol, emphasizing its
cultural significance, ecological role, and the importance of preserving its habitat.
Statement 1 is correct.
y Smoky-coloured coats tinted with cream and yellow shades and patterned with black spots.

CSE 2024 January 2024 57


DTRS Expalanations
y The spots are called rosettes which are unique to every snow leopard.
Statement 2 is incorrect.
y It is categorised as Vulnerable as per the conservation status of International Union for Conservation of Nature
(IUCN).
y It comes under the ambit of Schedule I as per the Wildlife (Protection) Act, 1972.
Statement 3 is correct.
y Recently, Kyrgyzstan has acknowledged the snow leopard as a national symbol.
Cultural and Symbolic Importance:
y In Kyrgyz culture, the snow leopard holds historical significance, symbolizing greatness, nobility, courage, bravery,
and resilience.
y It is tied to the story of Manas, a Kyrgyz folk hero, as depicted in the Epic of Manas.
y Legend attributes the snow leopard as the totem animal of the great Manas, symbolizing harmony between hu-
mans and nature.
y A globally acclaimed Kyrgyz writer portrayed the symbiotic relationship between humans and nature, embodied by
the snow leopard, in his novel «When Mountains Fall.»
Presidential Decree and Objectives:
y President of Kyrgyzstan signed a decree recognizing the snow leopard as the national symbol of Kyrgyzstan on
December 30, 2023.
y The decree emphasizes the snow leopard›s importance as a symbol of natural wealth, cultural prosperity, and as
an indicator of ecosystem stability in the mountainous regions that cover 1/3 of the global territory.
y It directs the Cabinet of Ministers to safeguard the snow leopard›s population and ecosystem, create a logo, define
its usage at various levels, and establish its ideological content.
Conservation Efforts and Collaboration:
y The decree calls for collaboration among ministries and agencies to promote snow leopard conservation, imple-
ment protective measures, attract green investments, and leverage the symbol for responsible tourism.
y The Global Snow Leopard and Ecosystem Protection Programme (GSLEP), consisting of 12 snow leopard range
countries, institutions, NGOs, scientists, and local communities, is actively involved in efforts to protect the species
and its habitat.
y snow leopard›s significance extends beyond cultural symbolism.
y It plays a crucial role in maintaining the fragile ecological balance of the High Asia region, known as the Third Pole.
y High Asia, comprising various mountain ranges like the Altai, Tian Shan, Himalayas, and others, is a vital water
source, supplying clean water to one-third of the world›s population.
y Climate change impacts this region profoundly, with warming occurring at nearly twice the average rate in the
Northern Hemisphere.
Global Conservation Initiatives:
y Snow leopard conservation gained momentum with the Bishkek Declaration on Snow Leopard Protection, adopted
unanimously at the World Snow Leopard Conservation Forum in 2013.

CSE 2024 January 2024 58


DTRS Expalanations
y The Global Snow Leopard and Ecosystem Protection Programme (GSLEP) was established to garner support from
snow leopard range countries and the international environmental community.
Source: https://www.downtoearth.org.in/news/wildlife-biodiversity/snow-leopard-enigmatic-big-cat-of-high-asia-now-
national-symbol-of-kyrgyzstan-93664

67.
Answer: A
Context: The Governor of the state and the state governments have been at odds in the recent past. Most recently
during the visit to the Calicut University campus, the Governor instructed the police to remove posters put up against
him. He termed the activists of the Students’ Federation of India “criminals” and accused the Chief Minister of “spon-
soring” them.
Explanation:
Statement 1 is correct: The Governor of a state in India holds office during the pleasure of the President. This is defined by
Article 156 of the Indian Constitution.
What it means:
y The Governor›s term of office is not fixed and can be terminated by the President at any time without giving any
reason.
y However, there is a normal term of five years from the date of assuming office.
y This ensures the President can remove a Governor without having to wait for the full term to end if circumstances
require it.
Constitutional provisions:
y Article 156(1): The Governor shall hold office during the pleasure of the President.
y Article 156(2): The Governor may, by writing under his hand addressed to the President, resign his office.
y Article 156(3): Subject to the foregoing provisions of this article, a Governor shall hold office for a term of five years
from the date on which he enters his office.
Statement 2 is incorrect: Under Article 200 of the Constitution, the Governor has three options when presented with a Bill
passed by the state legislature:
y Assent: Grant approval and make the Bill an Act.
y Withhold assent: Refuse to sign the Bill, essentially vetoing it.
y Reserve for President›s consideration: This applies to non-Money Bills only. In such cases, the Governor sends the
Bill to the President for their decision.
Statement 3 is incorrect: The Indian Constitution does not explicitly list any specific grounds upon which a Governor can be
removed by the President.
At the President›s Pleasure: Article 156(1) states that the Governor holds office «during the pleasure of the President.» This
means the President has the absolute power to remove a Governor at any time without giving any reason. The absence of
specified grounds reinforces this idea of discretionary power.
Flexibility and Adaptability: Specifying grounds could restrict the President›s ability to address unforeseen situations or
misconduct that doesn›t fall neatly into predefined categories. Leaving the grounds open allows for flexibility and adapting
to a diverse range of potential scenarios.

CSE 2024 January 2024 59


DTRS Expalanations
Political Considerations: The Governor acts as a bridge between the Union and the state, often navigating complex political
dynamics. Listing specific grounds for removal could introduce unintended consequences, making the Governor›s position
too vulnerable to political pressure from various players.
Source: https://www.thehindu.com/opinion/op-ed/raj-bhavan-needs-radical-reforms/article67698598.ece

68.
Answer: B
Context: Radiocarbon dating brought the first verifiable way to keep time to many fields of science, significantly transform-
ing them. Radiocarbon dating has revolutionized the study of time and its effects in various branches of science, sociology,
and psychology.
Explanation:
What is radiocarbon dating?
Radiocarbon dating (also referred to as carbon dating or carbon-14 dating) is a method for determining the age of an ob-
ject containing organic material by using the properties of radiocarbon, a radioactive isotope of carbon. The method was
developed in the late 1940s at the University of Chicago by Willard Libby.
Concept of radiocarbon dating
It is based on the fact that radiocarbon (14C) is constantly being created in the Earth›s atmosphere by the interaction of
cosmic rays with atmospheric nitrogen. The resulting 14C combines with atmospheric oxygen to form radioactive carbon
dioxide, which is incorporated into plants by photosynthesis; animals then acquire 14C by eating the plants. When the ani-
mal or plant dies, it stops exchanging carbon with its environment, and thereafter the amount of 14C it contains begins to
decrease as the 14C undergoes radioactive decay.
Statement 1 is correct: This technique is used to determine the age of objects which are generally of archaeological im-
portance.
Statement 2 is correct: This technique can be used for only those objects which have organic content (or in simpler words
objects which once were living). Radiometric dating is a technique which is used to find the age of rocks and other miner-
alised objects.
Statement 3 is incorrect: This technique is used for entities which are dead. When an entity dies, it stops exchanging ra-
diocarbon with the environment. Thus, as the fossil ages, the radiocarbon decays in its concentration. The proportion of
C14 determine the age of organic fossil.
Source: https://www.thehindu.com/sci-tech/science/radiocarbon-dating-history-science-revolution-politics-exp-
lained/article67692713.ece

69.
Answer: B
Context: Agreement between the landlocked Ethiopia and Somalia to use the Red Sea port of Berbera (Somalia) for trade.
Also, the earthquake in Japan hit the port of Wamija.
Explanation:
1. Wajima—Japan
A series of powerful earthquakes with magnitude 7.6 hit the island nation of Japan, bringing much damage in the
port city of Wajima.

CSE 2024 January 2024 60


DTRS Expalanations

2. Sylhet—Bangladesh
Bangladesh Prime Minister in March 2023 has offered India to utilise the country’s ports in Chattogram and
Sylhet, asserting that it would boost connectivity and enhance people-to-people contacts in the region. The Ch-
hattogram port is an important seaport in Bangladesh, which is strategically located due to its proximity to India’s
north-eastern States.

3. Berbera—Somalia
Ethiopia has historically depended on Djibouti for maritime trade support but the ongoing impasse due to Houth-
is’ hijacking Red Sea trade, has led to rebooting of Ethiopia’s strategy to use the port of Berbera for its trade and
transit.

CSE 2024 January 2024 61


DTRS Expalanations

Source: Major Japan quake kills 30; damage ‘extensive’, says PM Kishida - The Hindu; Ethiopia signs pact to use Somaliland›s
Red Sea port | Reuters

70.
Answer: C
Context: Recently, a study has been published titled-Balancing climate goals and biodiversity protection: legal implica-
tions of the 30x30 target for land-based carbon removal. It highlights the conflicts between land-based Carbon Diox-
ide Removal (CDR) Strategies and the establishment of protected areas, focusing on international environmental law.
Explanation:
y A global target to protect 30% of the planet for nature by 2030 (known as ‹30x30›) is included in the Kunming-Mon-
treal Global Biodiversity Framework, and was agreed at the Convention on Biological Diversity (CBD) at COP15.
y Nations have committed to the “30x30” biodiversity target to safeguard 30% of the world’s terrestrial and marine
areas by 2030. However, as of 2023, protected areas cover only around 16% of terrestrial areas and 8% of marine
areas, falling short of the 30x30 goal.
y The 30×30 target implies protection of at least 30 % of the world’s land and ocean by 2030.
y The 30×30 target is a global target that aims to halt the accelerating loss of species and protect vital ecosystems
that are the source of our economic security.
Additional information:
y At the 15th Conference of Parties (COP15) to the UN Convention on Biological Diversity “Kunming-Montreal Global
Biodiversity Framework” (GBF) was adopted.
y GBF includes 4 goals and 23 targets for achievement by 2030.
Key Targets of the GBF:
y Stop the extinction of known species, and by 2050 reduce tenfold the extinction risk and rate of all species (includ-
ing unknown)
CSE 2024 January 2024 62
DTRS Expalanations
y Reduce risk from pesticides by at least 50% by 2030
y Reduce nutrients lost to the environment by at least 50% by 2030
y Reduce pollution risks and negative impacts of pollution from all sources by 2030 to levels that are not harmful to
biodiversity and ecosystem functions
y Reduce global footprint of consumption by 2030, including through significantly reducing overconsumption and
waste generation and halving food waste
y Sustainably manage areas under agriculture, aquaculture, fisheries, and forestry and substantially increase agro-
ecology and other biodiversity-friendly practices
y Tackle climate change through nature-based solutions
y Reduce the rate of introduction and establishment of invasive alien species by at least 50% by 2030
y Secure the safe, legal and sustainable use and trade of wild species by 2030
y Green up urban spaces
Source: https://www.downtoearth.org.in/news/climate-change/study-finds-land-availability-limited-to-achieve-biodiver-
sity-targets-and-apply-climate-mitigation-strategy-93590

71.
Answer: A
Explanation:
Context: Recently, IIM-V concluded the central vigilance commission (CVC) programme on public procurement. At least 22
officers from various central public enterprises from various parts of the country participated in this programme.
y Central Vigilance Commission is an apex Indian governmental body created in 1964 to addres governmental cor-
ruption. In 2003, the Parliament enacted a law conferring statutory status on the CVC.
y The CVC was set up by the Government in February 1964 on the recommendations of the Committee on Preven-
tion of Corruption, headed by Shri K. Santhanam. In 2003, the Parliament enacted CVC Act conferring statutory
status on the CVC. Hence, statement 3 is not correct.
y The CVC is not controlled by any Ministry/Department. It is an independent body which is only responsible to the
Parliament. Hence, statement 1 is correct.
y Its annual report gives the details of the work done by the Commission and points to systemic failures which lead to
corruption in government departments. It presents this report annually to the President. The President then places
this report before each House of Parliament. Hence, statement 2 is not correct.
Source: https://timesofindia.indiatimes.com/city/visakhapatnam/cvc-programme-at-iimv-visakhapatnam-explore-pub-
lic-procurement/articleshow/106463823.cms

72.
Answer: D
Explanation:
Context: The Securities and Exchange Board of India (Sebi) recently approved a framework for Application Supported by
Blocked Amount (ASBA) like facility being made available to investors for secondary market trading.
y ASBA is an application containing an authorization to block the application money in the bank account, for sub-
scribing to an initial public offering (IPO). Hence option (a) is the correct answer.

CSE 2024 January 2024 63


DTRS Expalanations
y The facility is based on blocking of funds for trading in the secondary market through UPI (unified payments inter-
face) and will be optional for investors as well as stock brokers
y Under the framework client will continue to earn interest on his blocked funds in his savings account till the time
amount is debited. There will be a direct settlement with clearing corporation (CC), without passing through pool
accounts of the intermediaries, thereby providing client level settlement visibility to CC and thus avoiding the risk
of co-mingling of clients’ funds and securities.
y The Sebi board introduced an alternative route to enable a diverse set of entities to become sponsors of mutual
fund (MF). Such entities, who otherwise may not have been eligible to be sponsors, include private equity funds,
with requisite safeguards included in the proposal.
y The board approved a framework to provide for an institutional mechanism for prevention and detection of fraud
or market abuse by stock brokers. For this, the stock broker regulations will be amended to provide systems for
surveillance of trading activities and internal controls and whistle blower policy.
y The amendment will be made with regard to escalation and reporting mechanisms and obligations of the stock
broker and its employees.
y These amendments will come into effect from October 1, 2023.
y In order to bring more transparency and to ensure timely disclosure of material events or information by listed
entities,the board approved a few amendments to Listing Obligations and Disclosure Requirements (LODR) Reg-
ulations.
y It introduced a quantitative threshold for determining materiality of events and information.
y The board approved a stricter timeline for disclosure of material events / information for which a decision has been
taken in the meeting of the board of directors (within 30 minutes) and which are emanating from within the listed
entity (within 12 hours).
y The regulator also announced to set up a Corporate Debt Market Development Fund (CDMDF) in the form of an
Alternative Investment Fund to act as a backstop facility for purchase of investment grade corporate debt securities
during times of stress to instill confidence amongst the participants in the corporate bond market. The fund will
enhance secondary market liquidity.
Hence, Option (d) is the correct answer.
Source: https://www.newindianexpress.com/business/2024/jan/02/hdfc-bank-upi-goes-live-in-secondary-mkt-2647055.
html

73.
Answer: D
Context: The Tamil Nadu government has started work to breathe life into the Kolavai Lake and plans to open it for tour-
ism.
Explanation:
Pair 1 is incorrectly matched: Bellandur is a lake in Bengaluru city, Karnataka. It remains in headlines for the toxic foam
formation, which often catches fire. The lake receives 550 MLD of sewage.
y In his research in 2017, Dr TV Ramachandra, wetland specialist at the Centre for Ecological Sciences of the
Indian Institute of Science (IISc), highlighted that the wetland of Agara-Bellandur Lake, which earlier aided in
retaining rainwater and groundwater recharge, has been ‘abused’ with construction activities by Karnataka
Industrial Areas Development Board (KIADB).

CSE 2024 January 2024 64


DTRS Expalanations

Pair 2 is incorrectly matched: Kolavai Lake is in Tamil Nadu. Once a huge lake, Kolavai Lake has now been reduced to half
the size because of Mahindra World City, which is built on the bank of the lake.
y The lake is home to the South Asian carp, stinging catfish, and conger eel. Most of the surrounding land is now
overrun by invasive prosopis juliflora.
Pair 3 is incorrectly matched: Kolleru lake is in Andhra Pradesh.
y It is the largest freshwater lake of the country.
y It lies between the Godavari and Krishna river deltas near the city of Eluru.
y The lake is fed directly by the seasonal Budameru and Tammileru rivers and is connected to the Krishna and Goda-
vari systems by 67 inflowing drains and channels.
y It is also a Ramsar site.
y The lake was declared as a wildlife sanctuary in November 1999.
Source: https://timesofindia.indiatimes.com/city/chennai/explore-the-beauty-of-kolavai-lake-a-new-ecological-tourist-
spot/articleshow/106463530.cms

74.
Answer: B
Context: Members of the Warli tribal community in suburban Mumbai’s Gorai village are worried about the Shivaji museum
project announced by the Maharashtra government, as it may involve the destruction of their homes.
Explanation:
Pair 1 is incorrectly matched: Warli Tribes are an Adivasi indigenous tribe who live in the mountainous, coastal, and border-
ing regions of Gujarat and Maharashtra.
y The Warli speak their own language which is a mixture of Khandeshi Bhili dialect and Marathi. They speak Gujarati
and use Gujarati script. They also make use of Marathi words in their conversation.
y A Warli woman wears choli, (a type of blouse), a four yard sari called lugda. A man wears a bush-shirt, or a half
sleeve shirt, or sleeveless jacket and a half-pant or a striped under-wear.
Pair 2 is incorrectly matched: The Todas are a pastoral tribe inhabiting the higher elevations of the Nilgiri Hills of southern
India, mainly Tamil Nadu.
y The Toda traditionally live in settlements, consisting of three to seven small thatched houses, constructed in the
shape of half barrels and spread across the slopes of the pasture.

CSE 2024 January 2024 65


DTRS Expalanations
y They traditionally trade dairy products with the neighbouring people of Nilgiri. The religious and funerary rites
provide the social context in which complex poetic songs about the cult of the buffalo are composed and chanted.
Fraternal polyandry in traditional Toda society was fairly common, however, this has now largely been abandoned.
y During the last quarter of the twentieth Century some Toda pasture land was lost due to agriculture by outsiders
or forestation by the state government of Tamil Nadu.
y They are recognized among the particularly vulnerable tribal group. (PVTGs).
Pair 3 is correctly matched: Reangs are the second largest tribal community of Tripura.
y They are recognized as one of the 75 primitive tribes in India.
y Numerically as per 2001 Census there are 1,65,103 persons in this State.
y Reangs are said to have come first from Shan State of upper Burma (now Myanmar) in different weaves to the
Chittagong Hill tracts and then the Southern part of Tripura.
Pair 4 is correctly matched: Onges are one of the most primitive tribes in India.
y The Onges inhabit Little Andaman Island.
y This Hunting and Gathering tribe has also been settled by the Andaman & Nicobar Administration at Dugong Creek
and South Bay on Little Andaman Island.
y The present population of Onges is 96 as per the census 2001.
y They are a part of PVTGs.
Source: https://thewire.in/rights/warli-tribe-in-suburban-mumbai-fear-displacement-due-to-shivaji-museum-report

75.
Answer: A
Context: President Droupadi Murmu presented the Pradhan Mantri Rashtriya Bal Puraskar, 2024 to 19 children for their
exceptional achievements. The awards were presented to nine boys and 10 girls in six categories — art and culture, bravery,
innovation, science and technology, social service, and sports.
Explanation: Options 1 is correct only
Statement 1 is correct:
Pradhan Mantri Rashtriya Bal Puraskar
y The Pradhan Mantri Rashtriya Bal Puraskar (PMRBP) is organized to celebrate the energy, determination, ability,
zeal and enthusiasm of our children.
Eligibility:
y A child who is an Indian Citizen and is resident of India.
y Age Limit: A Child above the age of 5 years and not exceeding 18 years (as on 31st July of respective year).
y The act/incident/achievement should have been within 2 years of the last date of receipt of application/nomina-
tion for the year of consideration.
Statement 2 is incorrect:
y The Awards are announced on ‘Veer Bal Diwas’ every year which is marked on December 26th.
Statement 3 is incorrect:
y The awards are conferred by the President of India at New Delhi at a special ceremony to be held in January every
year.

CSE 2024 January 2024 66


DTRS Expalanations
Statement 4 is incorrect:
y The Pradhan Mantri Rashtriya Bal Puraskar was formerly called the National Child Award for Exceptional Achieve-
ment.
y It is India’s highest civilian honour for children, awarded annually by the Ministry of Women and Child Develop-
ment.
Source:
1. https://pib.gov.in/PressReleasePage.aspx?PRID=1997883
2. https://awards.gov.in/

76.
Answer: A
Explanation:
Context: Recently, the Tsunami waves hit several parts of Japan’s coastal areas and urgent evacuation warnings were issued
after a 7.6-magnitude earthquake shook the country’s north-central region.
Statement 1 is correct:
y The mid oceanic ridges are subjected to frequent shallow earthquakes due to up thrust of the lava. However, at the
convergent plate boundaries, deep focus earthquakes are found. Therefore at deep trenches deep seated earth-
quakes are found as deep trenches are generally formed at the convergent plate boundaries.
Statement 2 is incorrect:
y It is not currently possible to predict exactly when and where an earthquake will occur, nor how large it will be.
y However, seismologists can estimate where earthquakes may be likely to strike by calculating probabilities and
forecasts.
Additional information:
y An earthquake is a phenomenon that occurs without warning and involves violent shaking of the ground and ev-
erything over it.
y It results from the release of accumulated stress of the moving lithospheric or crustal plates.
y The earth›s crust is divided into seven major plates, that are about 50 miles thick, which move slowly and contin-
uously over the earth›s interior and several minor plates.
y Earthquakes are tectonic in origin; that is the moving plates are responsible for the occurrence of violent shakes.
y The occurrence of an earthquake in a populated area may cause numerous casualties and injuries as well as exten-
sive damage to property.
y Between 1998-2017, earthquakes caused nearly 750 000 deaths globally, more than half of all deaths related to
natural disasters.
y More than 125 million people were affected by earthquakes during this time period, meaning they were injured,
made homeless, displaced or evacuated during the emergency phase of the disaster.
Source: https://indianexpress.com/article/explained/everyday-explainers/japan-tsunami-earthquake-9090621/
CSE 2024 January 2024 67
DTRS Expalanations
77.
Answer: C
Context: The Mines Ministry, through the state-owned Khanij Bidesh India Ltd (KABIL), has entered into a draft exploration
and development agreement with Argentinan miner CAMYEN for possible acquisition and development of five-odd lithium
blocks.
Explanation:
The “Lithium Triangle” refers to a region in South America that contains some of the world’s largest lithium reserves. This
triangular-shaped region encompasses parts of Argentina, Bolivia, and Chile. Latin America nations, primarily Chile and
Argentina account for 30–35 percent of the world’s supplies. Chile, which has 11 per cent of the world’s lithium reserves,
supplies 26 per cent of the requirements; while Argentina with nearly one-fifth of the global resources supplies about 6 per
cent. Hence Option C is correct.
Additional Information:
y The alkaline mineral, also called ‘white gold’, is a cornerstone in India’s switch to green energy thereby reducing
its carbon footprints.
y India has put up just lithium blocks for auction – one in J&K and another in Chhattisgarh - with most of its domestic
requirements, across categories like EVs, lithium-ion battery making, and other energy storage solutions, being
met completely through imports.
Source: https://www.thehindubusinessline.com/companies/india-closing-in-on-lithium-deal-with-argentina-tapping-aus-
tralia-and-chile-too/article676866ece

78.
Answer: A
Context- Senegal’s pink lake (lake Retba) is on the verge of disappearing, threatened by the excessive artisanal salt mining,
dilution of its waters with a low salinity that is incompatible with the development of the green algae and accumulation of
extremely high levels of nitrates in its waters.

CSE 2024 January 2024 68


DTRS Expalanations
Explanation:
Statement 1 is correct: Lake Retba, better known as Lac Rose (the Pink Lake), is located around 35km from the city of Da-
kar, Senegal. It sits in a depression with a shoreline 6.5 metres below sea level.The lake is isolated from the sea by about 1
km of sand dunes. Its fresh water comes from the seasonal water table in the dunes, which are higher than the lake. Thus,
the sea provides most of the lake’s water and all of its salt.
Statement 2 is incorrect: The Pink Lake is one of the main tourist destinations in the Dakar region, primarily because of
the pink colour of its waters. The pink coloration is due to the proliferation of halophilic green algae (living in a salty
environment), Dunaliella salina, which contain red pigments. The alga is associated with halophilic bacteria of the genus
Halobacterium. This microscopic alga’s resistance to salt comes from its high concentration of carotenoid pigments, which
protect it from light, and its high glycerol content.
Source: Senegal’s pink lake is on the verge of disappearing — how to protect it (downtoearth.org.in)

79.
Answer: B
Explanation:
Context: The government has recently proposed repealing the Indian Stamp Act, 1899 and bringing in a new legislation for
the stamp duty regime in the country. The Department of Revenue under the Ministry of Finance has invited suggestions on
the draft ‘Indian Stamp Bill, 2023’ from the public within a period of 30 days.
Statement 1 is correct: The Indian Stamp Act, 1899 is a fiscal statute laying down the law relating to tax levied in the form
of stamps on instruments recording transactions.
Statement 2 is incorrect: Stamp duties are levied by the Central government, but within the states are collected and appro-
priated by the concerned states in terms of provisions of Article 268 of the Constitution.
Statement 3 is correct: The Union has the power to levy stamp duties on certain specified documents listed in Entry 91 of
the Union List of the Seventh Schedule of the Constitution of India. However, it doesn›t levy stamp duties on all documents.
y Union List: Entry 91 of the Union List empowers the Union government to set rates of stamp duty on the following
documents:
¾ Bills of exchange
¾ Cheques (except for those used for withdrawing money from banks)
¾ Promissory notes
¾ Bills of lading
¾ Letters of credit
¾ Policies of insurance
¾ Transfer of shares
¾ Debentures
¾ Proxies
¾ Receipts
y State List: The State List (Entry 63) allows states to levy stamp duties on documents not specifically mentioned in
the Union List. This includes documents like:
¾ Sale deeds

CSE 2024 January 2024 69


DTRS Expalanations
¾ Mortgage deeds
¾ Lease agreements
¾ Agreements
¾ Power of attorney
¾ Affidavits
¾ Certificates
¾ Collection and Revenue Sharing:
y While the Union government sets the rates for the documents in Entry 91, the actual collection of stamp duties is
usually done by the states.
y The revenue from these duties is generally retained by the states, except in the case of Union Territories, where it
goes to the Consolidated Fund of India.
y The power to levy stamp duties is shared between the Union and state governments, with the Union having au-
thority over specific documents and states having authority over others.
Source: https://indianexpress.com/article/business/economy/govt-proposes-repealing-indian-stamp-act-seeks-public-in-
put-on-draft-bill-9114509/
https://dor.gov.in/sites/default/files/stamp%20duty%202023.pdf

80.
Answer: B
Context: The new second generation Distress Alert Transmitter has been launched with new features.
Statement 1 is incorrect: It has been developed by the Indian Space Research Organisation (ISRO) with advanced capabil-
ities and features for the fishermen at sea to send emergency messages from fishing boats.
y The first version of DAT has been in operation since 2010, using which messages were sent through a communi-
cation satellite and received at a central control station (INMCC: Indian Mission Control Centre), where the alert
signals are decoded for the identity and location of the fishing boat.
y The extracted information is then forwarded to Maritime Rescue Coordination Centres (MRCCs) under the Indian
Coast Guard (ICG).
y Using this information, the MRCC coordinates to undertake search and rescue operations to save the fishermen in
distress. Till now, more than 20,000 DATs are being used.
Statement 2 is correct: The new features on DAT-SG include the facility to send back acknowledgement to the fishermen
who activate the distress alert from sea.
y The information about potential fishing zones can also be transmitted to fishermen using DAT-SG on regular
intervals.
y DAT-SG can be connected to mobile phones using bluetooth interface and the messages can be read in native
language using an app.
y The central control centre (INMCC) has a web based network management system called Sagarmitra, which main-
tains a database of registered DAT-SGs and helps MRCCs to access the information about boat, coordinate the boat
in distress in real time.
Source: https://www.thehindu.com/news/national/karnataka/isro-develops-second-generation-distress-alert-transmit-
ter/article67752250.ece/amp/

CSE 2024 January 2024 70


DTRS Expalanations
81.
Answer: A
Context: A total of 14 people, including 12 children and two teachers, who were on a picnic, died after a boat capsized in
Harni Lake in Gujarat›s Vadodara. The boat was ferrying 27 students of a private school, none of whom were reportedly
wearing life jackets.
Explanation:
y Pair 1 is incorrectly matched: Harni lake is situated in Vadodara, Gujarat. . Harni lake is still outside the city and
has a sparse human population in the surrounding area. It is a perennial pond and is the main source for the water
supply for drinking, bathing, sewage dumping and for many other human activities.
y Pair 2 is correctly matched: Vembanad is the largest brackish water lagoon in Kerala and the longest lake in India.
¾ It is also known as Vembanad Kayal, Vembanad Kol, Punnamada Lake (in Kuttanad) and Kochi Lake (in Kochi).
¾ It has its source in four rivers, Meenachil, Achankovil, Pampa and Manimala
¾ It is separated from the Arabian Sea by a narrow barrier island and is a popular backwater stretch in Kerala.
¾ The famous Vallam Kali, or Nehru Trophy Boat Race, is a Snake Boat Race held every year in August here.
¾ It was recognised as a Ramsar site in 2002.

y Pair 3 is incorrectly matched: Karyali lake, also called Karari lake, is located in Shimla, Himachal Pradesh. There is
a hill known as Chhota Shali Hillock, and the lake is located on the other side of this Shali peak.
Source: https://www.indiatoday.in/india/story/boat-capsizes-in-lake-in-gujarat-vadodara-death-toll-
kids-2490445-2024-01-18

82.
Answer: A
Explanation:
Context: Recently, tata steel has rejected a trade union proposal to keep the blast furnaces operational at Port Talbot Steel-
works in Wales, the UK, potentially jeopardising nearly 3,000 jobs, according to a report by the Guardian.
y Sparrows Point, Morisville, Scun Thorpe and Port Talbot are important industrial centres best known for the Iron
and Steel Industry.

CSE 2024 January 2024 71


DTRS Expalanations
y The iron and steel industry forms the base of all other industries and, therefore, is called a basic industry. Iron is
extracted from iron ore by smelting in a blast furnace with carbon (coke) and limestone. The molten iron is cooled
and moulded to form pig iron which is used for converting into steel by adding strengthening materials like man-
ganese.
Factors for Location of Iron and Steel Industry.
y The large integrated steel industry is traditionally located close to the sources of raw materials – iron ore, coal,
manganese and limestone – or at places where these could be easily brought, e.g. near ports. But in mini steel mills
access to markets is more important than inputs.
y Traditionally, most of the steel was produced at large integrated plants, but mini mills are limited to just one-step
process – steel making – and are gaining ground.
Distribution:
y The industry is one of the most complex and capital-intensive industries and is concentrated in the advanced coun-
tries of North America, Europe and Asia.
y In the U.S.A, most of the production comes from the northern Appalachian region (Pittsburgh), Great Lake
y region (Chicago-Gary, Erie, Cleveland, Lorain, Buffalo and Duluth) and the Atlantic Coast (Sparrows Point and
Morisville).
y In Europe, the U.K., Germany, France, Belgium, Luxembourgh, the Netherlands and Russia are the leading pro-
ducers. The important steel centres are Scun Thorpe, Port Talbot, Birmingham and Sheffield in the U.K.; Duisburg,
Dortmund, Dusseldorf and Essen in Germany; Le Creusot and St. Ettienne in France; and Moscow, St. Petersburgh,
Lipetsk, Tula, in Russia and Krivoi Rog, and Donetsk in Ukraine.
y In Asia, the important centres include Nagasaki and Tokyo-Yokohama in Japan; Shanghai, Tienstin and Wuhan in
China; and Jamshedpur, Kulti-Burnpur, Durgapur, Rourkela, Bhilai, Bokaro, Salem, Visakhapatnam and Bhadravati
in India.
Hence, option (a) is the correct answer.
Source: https://www.business-standard.com/companies/news/tata-steel-to-shut-blast-furnaces-at-port-talbot-plant-
3-000-jobs-at-risk-124011801135_1.html
https://www.cbsnews.com/baltimore/news/deadly-sparrows-point-stabbing-sparks-homicide-investigation/

83.
Answer: B
Context: Quishing or the use of QR codes to steal money and personal information has been on a rise in India.
Explanation:
y In quishing, the scammer generally asks the victim to scan a QR code using the camera, which then redirects
the user to a website created by the scammer. This website is generally an imitation of a genuine website, be it
an e-commerce platform, a bank or other platforms that deal with important personal and financial information.
y Then, the website asks the user to enter their personal information or account details. When the user enters this
information, the information is sent to the scammer, who can use this information to steal money or commit iden-
tity theft.
y According to a report, complaints related to UPI frauds have increased from 15,000 cases in 2022 to over 30,000
in 2023. Of this, nearly half of the cases reportedly involved QR codes.
y In some instances, these QR codes also prompt the victim to enter their UPI PIN to send money. When the user
enters their UPI PIN, the scammer receives it and can use it to steal money.
Source: https://www.businessinsider.in/tech/news/quishing-scams-on-the-rise-what-is-it-and-how-to-protect-yourself/ar-
ticleshow/106953014.cms

CSE 2024 January 2024 72


DTRS Expalanations
84.
Answer: D
Explanation:
Context: Ministry of Electronics and Information Technology (MeitY), Government of India, launched two flagship pro-
grams – “Centre of Excellence (CoE) in Intelligent Internet of Things (IIoT) Sensors” and India’s first Graphene Centre
“India Innovation Centre for Graphene (IICG)” today at Maker Village, Kochi, Kerala.
y The India’s first Graphene Centre IICG also has been established at Makers Village Kochi by MeitY, GoI and Govt.
of Kerala alongwith Tata Steel Limited.
y It aims to foster R&D, product innovation and capacity building in the area of Graphene and 2D material systems.
y It will also support the Graphene-Aurora program of the MeitY.
y This programme aims to fill the gap between R&D and commercialization by providing a complete facility to
startup and industry.
y Along with IICG, the Centre of Excellence in Intelligent Internet of Things Sensors was also launched.
y It aims at creating incubation facilities, build capacity in intelligent IoT sensors, promote research and innova-
tion, IoT is the interconnection of computing devices in everyday lives with the internet.
y Intelligent IOT is integrated with Artificial Intelligence.
Source: https://pib.gov.in/PressReleaseIframePage.aspx?PRID=1997096#:~:text=The%20India›s%20first%20
Graphene%20Centre,Graphene%20and%202D%20material%20systems.

85.
Answer: C
Context: Melvalai and similar sites at Alampadi and Sethavarai are a treasure trove of the art. The Tamil Nadu gov-
ernment has planned to declare the entire hillocks of Alampadi, Melvalai, and Sethavarai a protected monument.
Explanation:
y Alampadi, Melvalai, and Sethavarai, are examples of site of prehistoric rock paintings.
y All 3 sites are located in Villupuram district of Tamil Nadu.
y Melvalai is a nondescript village whose paintings dates back to 3000 B.C. and majority of etchings was in red ochre.
The word, Rattapparai, has become synonymous with the rock art painted in red ochre. There are 4 sets of paint-
ings that includes human figurines, animals, and symbols.
y The site at Sethavarai has prominent paintings of animals, especially of a deer and a fish. The outlines for these two
figures are drawn in red ochre, while the inner portion is filled with white ochre.
y The site at Alampadi has a combination of prehistoric rock paintings in red and white ochre that are similar to those
found at Sethavarai.
Additional information:
y The Ancient Monuments and Archaeological Sites and Remains Act (AMASR Act) 1958 regulates the preservation
of monuments and archaeological sites of national importance.
y The Act protects monuments and sites that are more than 100 years old, including temples, cemeteries, inscrip-
tions, tombs, forts, palaces, step-wells, rock-cut caves, and even objects like cannons and mile pillars that may be
of historical significance.
y The Archaeological Survey of India (ASI), which is under the aegis of the Union Ministry of Culture, functions under
this Act.
Source: https://www.thehindu.com/news/national/tamil-nadu/three-hillocks-that-boast-prehistoric-rock-paintings-in-vil-
lupuram-district/article67753841.ece
CSE 2024 January 2024 73
DTRS Expalanations
86.
Answer: A
Context: The recent study highlighted that the bastion for India’s democracy is being targeted, with the anti-communal and
progressive civic space under the most serious attack by the state
Explanation:
Statement 1 is correct: Civil society functions as a dynamic «bridge» between the state and the individual, advocating for
citizen rights and holding power accountable while simultaneously providing services and filling gaps in state provision. This
statement captures the dual role of Indian civil society. It acts as a watchdog, holding the state accountable for its policies
and ensuring they don›t infringe on citizen rights. Additionally, it fills in gaps in state provision, delivering crucial services like
education, healthcare, and environmental protection, particularly in marginalized communities. This «bridging» function is
essential for a healthy democracy and social progress.
Statement 2 is incorrect: Unlike a monolithic entity, Indian civil society is a vibrant tapestry of diverse organizations and
individuals holding a multitude of viewpoints and agendas. These can range from environmental groups advocating for sus-
tainable development to religious organizations focusing on community welfare, and everything in between. This diversity
enriches the public discourse but also presents challenges in building consensus and collective action.
Statement 3 is incorrect: Though grassroots activism remains a critical component, Indian civil society›s influence extends
far beyond. Many NGOs effectively collaborate with the government, influencing policy formulation and implementation
through research, advocacy, and public campaigns. They contribute to drafting legislation, participate in expert committees,
and raise public awareness on critical issues, thereby impacting the national discourse and policymaking processes. Under-
standing the diverse roles and complexities of Indian civil society is crucial for appreciating its significant contributions to
democracy, social progress, and development in the country.
Source: https://www.thehindu.com/opinion/lead/civil-society-under-siege-in-india/article67706998.ec

87.
Answer. B
Context: Places in News due to conflicts/protests.
Explanation- Only Pairs 2 and 3 are correctly matched.
Bashkortostan Region- Russia

CSE 2024 January 2024 74


DTRS Expalanations
Places in News and their location Reason for being in news
Bashkortostan Region- Russia Hundreds of protesters have clashed with police in the Russian
republic of Bashkortostan in a rare display of public outrage
against the prison sentence given to a local eco-activist Faiz
Alsynov.
Sistan Baluchistan- Iran Terrorist group Jaish-al-Adl (A sunni Islamist group) claimed an
attack on the police station in Sistan Baluchistan region recent-
ly.
This terrorist group backs the separatist movement in Sistan
Baluchistan region which has a majority of Sunni population.

Kurdistan- Iraq, Turkey, Iran, Syria Iran’s Revolutionary Guards claimed the recent attacks on the
spy headquarters of Israel in Iraq’s semi-autonomous Kurdis-
tan region.
Islamic State claimed responsibility for two explosions in Iran
this month that killed nearly 100 people and wounded scores
at a memorial for top commander Qassem Soleimani.

Idlib - Syria Iran has also attacked the military bases of terror groups based
in Idlib, Syria. This region is controlled by the Jihadists and reb-
els.

CSE 2024 January 2024 75


DTRS Expalanations
Source: https://www.indiatvnews.com/news/world/iran-pakistan-airstrike-what-is-baloch-liberation-front-which-paki-
stan-targeted-in-airstrikes-in-sistan-baluchestan-province-2024-01-18-912584
https://zeenews.india.com/world/explained-pakistan-iran-border-tensions-and-story-of-balochistan-2711242.html

88.
Answer: C
Explanation:
Context:
The Prime Minister of India recently visited the Kalaram Temple on the banks of the Godavari in the Panchavati area of the
city.
y In March 1930, Dr. B.R. Ambedkar launched the famous Kalaram Temple Entry Satyagraha at Nasik, which became
a landmark in the Dalit liberation movement in the history of modern Maharashtra.
Kalaram Temple Entry Satyagraha was a movement from the masses and involved active participation of the people. Hence
statement 1 is correct.
y From 3rd March 1930, the squatting started around the temple. The Bombay Government imposed Article 144 of
Civil Procedure Code around the temple. On April 7th, the day of Ram Navami Dr. Ambedkar led procession of two
hundred volunteers.
¾ On 9th April, the caste Hindus made an attack on the satyagrahis There were several attempts made to
conciliate the issue. In the beginning, B.G Kher, Swami Anand the Congress leaders from Bombay, supported
the cause of Ambedkar.
y Meanwhile Gandhiji started the nationwide Civil Disobedience Movement. He did not want any other movement
at the same time. In his article in Young India he mentioned Kalaram Temple Entry Satyagraha and urged depressed
classes to give up the entry movement. The first phase of Satyagraha came to an end when Dr. Ambedkar gave the
permission for its suspension. Hence statement 2 is correct.
y In January 1933 Dr. Subarayan›s temple entry bill was rejected in the Imperial Council. The decisions of the Bombay
High Court were unfavourale to the satyagraha. Dr. Ambedkar did not give permission to restart the satyagraha in
April 1933.
¾ When in 1934 the Satyagraha Committee wanted to start satyagrala, Dr Amedkar in his letter to Bhaurao
Gaikwad expressed that he wanted to stop saryagrah permanently. He came to the decision that he would
not use another temple entries. On the advice of Dr. Ambedkar, the Satyagraha Committee gave up the cause.
Thus the satyagraha was not successful in its objective of gaining temple entry for depressed classes. Hence
statement 3 is correct.
Source: https://indianexpress.com/article/explained/explained-history/nashik-kala-ram-temple-modi-ambed-
kar-9106708/

89.
Answer: B
Explanation:
Context: According to a new study, more droughts brought on by climate change could worsen the impact of wildfires in
United States’ southern Appalachians region
y Rising annual drought patterns and heightened drought variability may amplify the incidence of wildfires, leading
to increased forest burning, a paper published in journal Fire Ecology, January 12, 2024 found.
y The southern Appalachians is a unique section of the mountain range comprising Kentucky, Tennessee, Virginia,
Maryland, West Virginia and North Carolina and features a wealth of forested land.

CSE 2024 January 2024 76


DTRS Expalanations
y Coal is one of the important minerals which is mainly used in the generation of thermal power and smelting of iron
ore. It is one of the most mined minerals from the earth. Of the three fossil fuels (Petroleum, natural gas and coal),
coal has the most widely distributed reserves; coal is mined in over 100 countries, and on all continents except
Antarctica. The largest proved reserves are found in the United States, Russia, China, Australia and India.
Distribution:
Pair 1 is correctly matched:
y USA: Allegheny and Appalachian Mountains have enormous coal deposits. Carboniferous coal of the Great Lakes
and Appalachians region helped USA become a leading industrialized nation. The North Antelope Rochelle Coal
Mine located in the Powder River Basin of Wyoming is the world›s biggest coal mine.
y Russia & Ukraine: Ural region and the Donbass Basin are important mining regions. Most of Russia’s coal in the
Siberian Region is untapped.
y Australia: Australia is a leading producer of coal. Most of its coal is exported to China, Japan etc. Australia has rich
coking coal deposits. India imports coking coal mainly from Australia. Bowen Basin coalfield, Galilee Basin coalfield,
South Maitland coalfield, Sydney Basin coalfield, and Latrobe valley coalfield are some of the important coalfields
of Australia. Hence, pair 2 is not correctly matched.
y China: China is the largest producer and consumer of coal in the world. China’s coal is of poor quality. It imports
metallurgical grade coal from Australia. Shanxi, Fushun, Inner Mongolia, Kansu and most coal is found in the north
and northwest of the country. Hence, pair 3 is correctly matched.
Source: https://www.downtoearth.org.in/news/climate-change/more-droughts-brought-on-by-climate-change-could-
worsen-wildfires-in-us-s-southern-appalachian-forests-93945

90.
Answer: A
Context: The Ministry of Statistics and Programme Implementation (Mospi) has launched a new app called e-Sakshi to track
the fund flow under the Member of Parliament Local Area Development Scheme (MPLADS).
Explanation:
y The mobile app would offer convenience and accessibility, allowing MPs to propose, track, and oversee the proj-
ects at their fingertips.
y The application will streamline the communication between MPs and relevant authorities, facilitating a more effi-
cient exchange of information.
About the MPLAD scheme:
y The Members of Parliament Local Area Development Division is entrusted with the responsibility of implementa-
tion of Members of Parliament Local Area Development Scheme (MPLADS).
y Under the scheme, each MP has the choice to suggest to the District Collector for works to the tune of Rs.5 Crore
per annum to be taken up in his/her constituency.
y The Rajya Sabha Members of Parliament can recommend works in one or more districts in the state from where
he/she has been elected.
y The nominated members of the Lok Sabha and Rajya Sabha may select any one or more districts from any one state
in the country for implementation of their choice of work under the scheme.
Source: https://pib.gov.in/PressReleasePage.aspx?PRID=1996730

CSE 2024 January 2024 77


DTRS Expalanations

CSE 2024 January 2024 78


DTRS Expalanations
91.
Answer: A
Context: CERT-IN has taken over 11 months to plug the vulnerabilities that exposed the personal details of VVIPs, including
top industrialists, celebrities and sports personalities in the country.
Explanation:
Statement 1 is correct: CERT-In or Indian Computer Emergency Response Team is the national incident response center for
major cyber security incidents in India.
y CERT-In›s primary role is to raise security awareness among the Indian cyber community and to provide technical
assistance and advise them to help recover from computer security incidents.
y CERT-In provides technical advice to System Administrators and users to respond to computer security incidents. It
also identifies trends in intruder activity, works with similar institutions & organizations to resolve major security
issues, and disseminates information to the Indian cyber community.
Statement 2 is incorrect: CERT-In comes under the Ministry of Electronics and Information Technology (Meity).
Source: https://www.thehindu.com/news/national/critical-vulnerability-exposing-personal-data-of-vvips-company-direc-
tors-fixed-by-cert-in-months-after-alert/article67744992.ece#:~:text=It%20took%2011%20months%20and,worth%20indi-
viduals%2C%20to%20get%20resolved.

92.
Answer: D
Context: Odisha Chief Minister recently unveiled the Shree Mandir Parikrama Prakalpa (SMPP), a massive peripheral devel-
opment project for developing Puri as a world-class heritage city.
Explanation:
Statement 1 is incorrect: The Project has been developed under Augmentation of Basic Ameni-
ties and Development of Heritage and Architecture at Puri (ABADHA) scheme of Odisha government.
Statement 2 is incorrect: The 75-metre-wide space encircling the temple, known as the heritage corridor, has been named
the SMPP and divided into nine different zones including:
y A 7-meter green buffer zone with terraced landscapes
y 10-meter inner and outer paths for processions and circumambulation
y A 14-meter garden showcasing trees integral to Jagannath culture
y Public facilities like restrooms, drinking fountains, and more
y Designated emergency and service vehicle lanes
y Traffic lanes ensuring access to neighboring areas
Statement 3 is incorrect: Meghanada Pacheri in the corridor is called the boundary of the temple. This is followed by the
10-metre Antar Pradakshina, which allows space for ceremonial processions of the deities throughout the year and it also
doubles as a parikrama path, inviting the general public to undertake a circumambulation of the sacred Shree Mandira
complex.
Additional information:
Features of Jagannath Temple:
y The temple is believed to have been constructed in the 12th century by King Anatavarman Chodaganga Deva of
the Eastern Ganga Dynasty.
y Jagannath Puri temple is called ‘Yamanika Tirtha’ where, according to the Hindu beliefs, the power of ‘Yama’, the
god of death has been nullified in Puri due to the presence of Lord Jagannath.

CSE 2024 January 2024 79


DTRS Expalanations
y This temple was called the “White Pagoda” and is a part of Char Dham pilgrimages (Badrinath, Dwaraka, Puri,
Rameswaram).
y There are four gates to the temple- Eastern ‘Singhdwara’ which is the main gate with two crouching lions, Southern
‘Ashwadwara’, Western ‹Vyaghra Dwara and Northern ‘Hastidwara’. There is a carving of each form at each gate.
y In front of the entrance stands the Aruna stambha or sun pillar, which was originally at the Sun Temple in Konark.
Source: https://www.thehindu.com/news/national/other-states/odisha-cm-patnaik-inaugurates-800-crore-heritage-corri-
dor-project-around-puris-jagannath-temple/article67747926.ece

93.
Answer: A
Explanation:
Context: According to the Annual Education Status Report (ASER) released on January 18, 2023, enrollment of children in
government schools in rural areas of Jharkhand is increasing.
Statement 1 is incorrect: Recently the 18th Annual Education Report (ASER), 2023 was released by NGO Pratham, which
revealed high enrolments of children in schools which is a good performance indicator for the programs.
y According to this report, in the year 2018, the number of children enrolled in government schools of the state was
78 percent. During this period, the number of children enrolled in government schools has increased by 5 percent.
Statement 2 is incorrect: Annual Status of Education Report (ASER) is biennially published by the NGO Pratham, since 2005.
The Annual Status of Education Report (ASER) is a citizen-led household survey that provides nationally representative esti-
mates of children›s schooling status and their foundational reading and arithmetic skills.
Statement 3 is correct: To conduct a yearly household-based assessment on basic learning outcomes, so as to include all
children (in rural areas) of age 6 to 14. To do an assessment of basic reading and arithmetic, by using the tools that are not
based on grade level objectives and competencies rather the tools that are easy to administer and simple to understand.
y Overall, 86.8% of 14-18-year-olds are enrolled in an educational institution.
y There are small gender gaps in enrolments, but notable differences are visible by age.
y The percentage of youth not enrolled is 3.9% for 14-year-old youth and is 32.6% for 18-year-olds.
This report uses the ASER 2024 survey data to explore the following areas:

CSE 2024 January 2024 80


DTRS Expalanations
Source: https://indianexpress.com/article/education/pratham-foundation-aser-2023-school-report-released-janu-
ary-17-9111464/
https://www.thehindubusinessline.com/news/education/aser-report-reveals-25-of-teens-between-14-and-18-age-group-
struggle-to-read-grade-2-text-with-fluency/article67748291.ece

94.
Answer: A
Explanation:
Context: In February, experts anticipate a surge in the number of Olive Ridley turtles choosing the Rushikulya river mouth
in Odisha’s Ganjam district for mass nesting.
Statement 1 is correct. The males and females grow to the same size.
Statement 2 is correct. They are carnivores and feed mainly on jellyfish, shrimp, snails, crabs, molluscs, and a variety of fish
and their eggs.
Statement 3 is incorrect. The species is recognized as Vulnerable by the IUCN Red list.
Statement 4 is incorrect. The coast of Odisha in India is the largest mass nesting site for the Olive-ridley, followed by the
coasts of Mexico and Costa Rica.
Additional information:
Olive Ridley turtles:
y The Olive ridley turtles are the smallest and most abundant of all sea turtles found in the world.
y They are found to be inhabiting the warm waters of the Pacific, Atlantic, and Indian oceans.
y These turtles, along with their cousin the Kemps ridley turtle, are best known for their unique mass nesting called
Arribada, where thousands of females come together on the same beach to lay eggs.
y They lie in Scheduled 1 of the Wildlife Protection Act, 1972
y They are in Appendix I of the CITES.

About Rushikulya river:


y It is one of the major rivers in Odisha and covers the entire catchment area in the districts of Kandhamal and Gan-
jam.
CSE 2024 January 2024 81
DTRS Expalanations
y The Rushikulya originates at an elevation of about 1000 meters from the Daringbadi hills of the Eastern Ghats.
y The tributaries of the Rushikulya River are Dhanei, Badanadi, and Baghua.
y It does not have any delta in its mouth region.
y This river is extremely rich in mineral wealth and some of the prime ones include-Lime stone, sand talc, grinding
materials, black sand, and clay.
y This is one of the remote areas for mass nesting and is regarded as a site of Ridley Olive sea turtles.

Source: https://www.downtoearth.org.in/news/wildlife-biodiversity/conducive-climate-beach-this-year-in-february-
large-number-of-olive-ridley-turtles-expected-to-lay-eggs-in-rushikulya-93936

95.
Answer: B
Explanation:
Context: The Gujarat Forest department›s recent ban on Conocarpus trees, initially introduced as ornamental and ave-
nue trees, has unveiled the environmental and health hazards associated with this exotic species.
Statement 1 is incorrect. It is an invasive mangrove species. It is native to parts of North and South America, as well as parts
of Africa.
Statement 2 is correct. It is very adaptive and could grow even in areas with extreme salinity.
Statement 3 is correct.
Health Concerns
y Respiratory Issues: Conocarpus pollen can cause respiratory illnesses, colds, coughs, asthma, and allergies, partic-
ularly during winter.
y Human Health Priority: The ban addresses the need to safeguard human health, especially in the context of the
recent global health crisis.

CSE 2024 January 2024 82


DTRS Expalanations
Evolution of Conocarpus in India
y Initially introduced as ornamental and avenue trees, Conocarpus species gained popularity for their fast growth
rate and adaptability to harsh conditions.
y Massive plantation drives led to the widespread presence of Conocarpus, notably in significant projects like the
Smritivan memorial in Bhuj and the ‹Mission Million Trees› drive in Vadodara.
y Native vs. Exotic: Conocarpus, native to Africa, raised concerns about its impact on the existing food web and
ecosystem values for native species.
y Exotic Species vs. Indigenous Diversity: Experts highlight the disturbance caused by Conocarpus to the local eco-
system, advocating for the prioritization of native plant species that support local biodiversity.
Additional information:
Conocarpus tree:
y It is a flowering plant belonging to the family of Combretaceae.
y Trees of this species flower in winter and spread pollen in nearby areas.
y It has been used by various public authorities in India as landscaping for road medians, along roads, and in public
gardens.
y Countries in the Arabian Peninsula have used the plant to block sand from desert storms and to control pollution.
y This tree is famous for its dark green leaves color throughout the year and withstands harsh environmental condi-
tions such as high and low temperatures.
y It absorbs more water from soil than other species and is a threat to groundwater.

Source: https://india.mongabay.com/2024/01/gujarat-bans-exotic-conocarpus-tree-amid-health-and-environ-
ment-hazard/?amp=1

96.
Answer: A
Context: Russia repelled air attacks by Ukraine shooting down a drone near Moscow and missiles over the Belgorod region
bordering Ukraine. Earlier in December, 2023 massive shelling by Ukraine killed dozens in the same region.
Explanation:
y Belgorod region is located in Russia.

CSE 2024 January 2024 83


DTRS Expalanations
y Location – European part of Russia
y Situated on the southern and south-eastern slopes of the Central Russian Upland.
y Borders – Kursk and Voronezh regions, the Lugansk People’s Republic, Ukraine
y Major rivers – Psyol, Seversky Donets, Vorskla, Oskol, Tikhaya Sosna and Valui.

Source: https://www.thehindu.com/news/international/russia-downs-drone-near-moscow-missiles-in-border-region/ar-
ticle67752489.ece#:~:text=Russia%20repelled%20air%20attacks%20by,Ukraine%2C%20officials%20said%20on%20Thurs-
day.

97.
Answer: D
Context- Celebration of the 9th edition of the Pakke Paga Hornbill Festival (PPHF), a state festival of Arunachal Pradesh.
Explanation- The Pakke Paga Hornbill Festival focuses on wildlife conservation, with a particular emphasis on hornbills.
Four species of the birds — Wreathed, Great Indian, Oriental Pied and the endangered Rufous-necked — are found in the
Pakke Tiger Reserve (PTR) of Arunachal Pradesh. The area is also home to the Nyishi, the largest tribal group in Arunachal
Pradesh.

CSE 2024 January 2024 84


DTRS Expalanations
The first-ever PPHF was held on January 16-18, 2015. The aim was to recognise the role played by the Nyishi in conserving
hornbills in PTR. The Nyishi had formerly hunted hornbills and used their bills to craft traditional headgear. They had later
turned into hornbill conservationists.
Other objectives were to raise alternative sources of income for the region and to create awareness in the rest of India
about the wonders of PTR and its surrounding areas.

y Pakke Tiger Reserve (declared in 1999 - 2000) lies in the foothills of the eastern Himalaya in the East Kameng dis-
trict of Arunachal Pradesh. It is also known as Pakhui Tiger Reserve.
y It falls within the Eastern Himalaya Biodiversity Hotspot.
y It is home to over 2000 species of plants, 300 species of birds, 40 species of mammals, 30 species of amphibians
and 36 species of reptiles. Many species of the flora and fauna are globally threatened, and PTR is one of the last
remaining strongholds left for these species.
y It is known for its amazing sightings of four resident hornbill species.
About Hornbills:
y India is home to 9 species of hornbills. The northeastern region has the highest diversity of hornbill species within
India. Five hornbill species are found here, with the northeast being the exclusive habitat for three of them, the
wreathed hornbill (Aceros undulatus), the brown hornbill (Anorrhinus austeni) and the Rufous-necked hornbill
(Aceros nipalensis). The other two species, the great hornbill (Buceros bicornis) and the Oriental pied hornbill
(Anthracoceros albirostris) also occur in other parts of India (Western Ghats).
y The Great hornbill is the state bird of Arunachal Pradesh and Kerala. It is ‘vulnerable’ under the IUCN Red list.
y The Hornbill festival celebrated in Nagaland is named after the bird – Hornbill which is the most revered and ad-
mired bird for the Nagas.
Source: Arunachal Pradesh’s Pakke Paga Hornbill Festival gears up for its 9th Edition (downtoearth.org.in)

98.
Answer: C
Explanation:
Context: The government’s immediate plan is to start a pilot project of the D2M technology in 19 cities, with Tejas Net-
works-owned wireless communication and semiconductor solutions company Saankhya Labs.

CSE 2024 January 2024 85


DTRS Expalanations
y Statement 2 is correct: The D2M technology is based on the convergence of broadband and broadcast, using which
mobile phones can receive terrestrial digital TV.
y Statement 1 is correct: This technology allows broadcasting video and other forms of multimedia content directly
to mobile phones, without needing an active internet connection.
y It would be similar to how people listen to FM radio on their phones, where a receiver within the phone can tap
into radio frequencies.
y Governments Initiatives to Facilitate D2M Technology:
y The DoT has set up a committee to study the feasibility of a spectrum band for offering broadcast services directly
to users› smartphones.
y Band 526-582 MHz is envisaged to work in coordination with both mobile and broadcast services.
y Public service broadcaster Prasar Bharati had last year announced a collaboration with IIT Kanpur to test the fea-
sibility of the technology.
Source: https://www.financialexpress.com/life/technology-internet-free-video-stream-on-mobiles-next-year-3366373/

99.
Answer: B
Explanation:
Context: Recently, deep learning-based systems developed by DeepMind (now a subsidiary of Alphabet Inc. – Google’s
parent company) for predicting the three-dimensional (3D) structures of proteins are, arguably, the tangible AI-based inno-
vations which made the remakable “consumer-facing” introduction of OpenAI’s ChatGPT.
y AlphaFold is an Artificial Intelligence-based protein structure prediction tool.
y It is based on a computer system called Deep Neural Network. Neural networks use a large amount of input data
and provide the desired output exactly like how a human brain would. The real work is done by the black box be-
tween the input and the output layers, called the Hidden Networks.
y AlphaFold is fed with protein sequences as input. When protein sequences enter through one end, the predicted
three-dimensional structures come out through the other.
Hence option (b) is the correct answer.
Source: https://www.oodaloop.com/archive/2024/01/12/before-openais-chatgpt-there-was-google-deepminds-alpha-
fold/
https://www.science.org/content/blog-post/alphafold-s-place-world

100.
Answer: B
Context: Snowfall in the Ladakh-Jammu and Kashmir region has been showing a declining trend in recent years. The overall
decreasing trend of snowfall has been attributed to a decline in western disturbance events and gradual rise in tempera-
tures.
Explanation:
Statement 1 is incorrect: Western Disturbances are low-pressure systems that originate in the Mediterranean region and
move eastwards across central Asia.
Statement 2 is correct: Winter precipitation in the Himalayan region is caused mainly by Western Disturbances. These are
large eastward-moving rain-bearing wind systems that originate beyond Afghanistan and Iran, picking up moisture from as
far as the Mediterranean Sea and even the Atlantic Ocean.

CSE 2024 January 2024 86


DTRS Expalanations
Additional information:
y Western Disturbances are the primary source of rainfall over north and northwest India during the post-monsoon
and winter months.
y Along with the south-west monsoon season that runs from June to September, and the north-east monsoon that
brings rains to Tamil Nadu and some other regions, Western Disturbances are the third major contributors to In-
dia’s annual rainfall.
y During winters, about four to six western disturbance events happen every month on an average.
Source: https://indianexpress.com/article/explained/explained-climate/kashmir-ladakh-without-snow-why-implica-
tions-9110841/

101.
Answer: A
Context: Researchers have discovered for the first time that the presence of certain microbes can tell them whether the
coral will survive heat stress or coral bleaching.
Explanation:
Statement 1 is correct: Variation in ocean temperature, due to climate change, have become a major source of coral
bleaching.
When corals are stressed by changes in conditions such as temperature, light, or nutrients, they expel the symbiotic algae
living in their tissues, causing them to turn completely white.
It is to be noted that not all bleaching events are due to warm water. In January 2010, cold water temperatures in the Florida
Keys caused a coral bleaching event that resulted in some coral death.
Statement 2 is incorrect: When a coral bleaches, it is not dead. Corals can survive a bleaching event, but they are under
more stress and are subject to mortality.

Source: https://indianexpress.com/article/technology/science/microbe-saving-coral-climate-change-9111960/
CSE 2024 January 2024 87
DTRS Expalanations
102.
Answer: D
Context: Reserve Bank of India Governor said that monetary policy in India must remain actively disinflationary despite the
recent sharp fall in core inflation.
Explanation:
Statement 1 is incorrect: Headline inflation refers to the change in value of all goods in the basket.
On the other hand, core inflation excludes food and fuel items from headline inflation.
Statement 2 is incorrect: Since the prices of fuel and food items tend to fluctuate and create ‘noise’ in inflation computa-
tion, core inflation is less volatile than headline inflation.
In a developed economy, food & fuel account for 10-15% of the household consumption basket and in developing econo-
mies it forms 30-40% of the basket.
The Consumer Price Index (CPI) is a common measure of headline inflation.
Source: https://www.business-standard.com/finance/news/monetary-policy-has-to-remain-actively-disinflationary-shak-
tikanta-das-124011701023_1.html

103.
Answer: B
Context: The Chabahar port, situated at the mouth of the Gulf of Oman, holds paramount strategic importance for both
Iran and India. The recent discussions between Indian External Affairs Minister and Iranian Minister underscore the on-
going efforts to establish a long-term cooperation framework for this key maritime asset.
Explanation:

y Chabahar Port is a seaport in Chabahar located in southeastern Iran, on the Gulf of Oman.

CSE 2024 January 2024 88


DTRS Expalanations
y It serves as Iran›s only oceanic port, and consists of two separate ports named Shahid Kalantari and Shahid Behesh-
ti, each of which has five berths.
y It is only about 170 kilometres west of the Pakistani port of Gwadar.
Historical Context and Strategic Significance:
y India›s involvement in the Chabahar port dates back to 2002 when discussions initiated between the then National
Security Advisor to Iranian President, and his Indian counterpart.
y As India sought alternative trade routes due to severed land connections with Iran and Central Asia post-Partition,
Chabahar emerged as a crucial element in its geopolitical strategy.
y Chabahar not only provides Iran with a deepwater port on the global oceanic trade route map but also offers India
an alternative route.
¾ It bypasses Pakistan›s restrictions on land access for trade with Afghanistan and Central Asia.
y Furthermore, the port is integral to the proposed International North-South Transport Corridor (INSTC).
¾ It connects the Indian Ocean and Persian Gulf to the Caspian Sea and northern Europe.
Source: https://indianexpress.com/article/explained/explained-global/history-of-iran-chabahar-port-india-9117732/

104.
Answer: D
Explanation:
Context:In the first batch of beneficiaries of the Pradhan Mantri Tribal Adivasi Nyay Maha Abhiyan (PM-JANMAN) package
for the development of Particularly Vulnerable Tribal Groups (PVTGs), the government has got women who are ambassa-
dors to help the administration in availing the benefits. Are being made. packages to the most remote members of their
community.
y PVTGs are more vulnerable among tribal groups. Due to this factor, more developed and vocal tribal groups take a
larger share of tribal development funds, causing PVTGs to require more funds for their development.
y In this context, in 1975, the Government of India took the initiative to identify the most vulnerable tribal groups
as a separate category called PVTG and announced 52 such groups, while in 1993 an additional 23 groups were
added to this category, taking the total It›s 75. PVTGs are among the 705 Scheduled Tribes spread across 18 states
and one Union Territory (UT) in the country (2011 census). In Uttar Pradesh and Uttarakhand, the Baksa and Rajis
tribes come under the PVTGs. Maximum in Odisha (13)
¾ In 1973 the Dhebar Commission created Primitive Tribal Groups (PTGs) as a separate category among the less
developed tribal groups. In 2006, the Government of India renamed PTGs as Particularly Vulnerable Tribal
Groups (PVTGs). Hence option (D) is the correct answer.
Government of India follows the following criteria for identification of PVTGs.
y Pre-agricultural level of technology
y Low level of literacy
y Economic backwardness
y A declining or stagnant population.
Source: https://www.thehindu.com/news/national/these-pm-janman-beneficiaries-make-sure-none-is-left-out/arti-
cle67745315.ece
CSE 2024 January 2024 89
DTRS Expalanations
105.
Answer: A
Context: An Indian delegation under the Department of Science and Technology (DST) visited Mauna Kea, Hawaii, USA and
discussed challenges in the progress of the Thirty Meter Telescope (TMT) project and ways to overcome them.
Explanation:
Statement 1 is correct:
y Thirty Meter Telescope (TMT) is a 30-meter diameter primary mirror optical and infrared telescope.
Statement 2 is incorrect:
y The Thirty-meter telescope (TMT) project is an international partnership between CalTech, Universities of Califor-
nia, Canada, Japan, China, and India through the Department of Science and Technology (DST) and Department
of Atomic Energy (DAE).
y It is being established at Mauna Kea, Hawaii, USA.

y India is a founder-member partner in this project which aims to open new windows to the universe through opti-
cal and infrared astronomy.
y Indian participation in this project was approved by the Union Cabinet in 2014.
Source:
1. https://pib.gov.in/PressReleasePage.aspx?PRID=1995476
2. https://pib.gov.in/PressReleasePage.aspx?PRID=1671685

106.
Answer: A
Explanation:
Context:
Researchers recently created the world’s first functional semiconductor made from graphene.
Statement 1 is correct: Graphene is a one-atom-thick layer of carbon atoms arranged in a hexagonal lattice. It is the build-
ing-block of Graphite (which is used, among others things, in pencil tips). It was first isolated in 2004.
Statement 2 is incorrect: It is an excellent conductor of heat and electricity. It conducts electricity better than copper. It is
almost perfectly transparent, as it absorbs only 2% of light. It is impermeable to gases, even those as light as hydrogen and
helium.

CSE 2024 January 2024 90


DTRS Expalanations
Source: https://lifestyle.livemint.com/smart-living/innovation/semiconductor-graphene-silicon-quantum-computing-sci-
ence-111704368420342.html

107.
Answer: A
Context: Prime Minister commemorated Thiruvalluvar Day on January 16th, honoring the memory of the great Tamil sage,
Saint Thiruvalluvar.
Explanation:
Statement 1 is correct:
y Thiruvalluvar is renowned Tamil poet and philosopher-saint believed to have lived between 3rd Century BC and
1st Century BC.
Statement 2 is incorrect:
y Thiruvalluvar, also known as Valluvar, holds a significant place in Tamil culture, standing as one of its most distin-
guished literary figures.
y Living during the Sangam era of Tamil classical culture, Thiruvalluvar is revered as a poet, philosopher, and a wise
man.
y The Sangam era, known for its eminent poets, witnessed the emergence of the earliest Tamil literature, which has
endured for over 2,000 years.
y Thirukkural, his masterpiece, consists of couplets covering various aspects of life, including morality, politics, the
economy, and love.
y Widely regarded as a unique and highly esteemed piece of Tamil literature, Thirukkural is considered one of the
greatest works on ethics and morality, known for its universality and secular nature.
y Tolkāppiyam, also romanised as Tholkaappiyam was authored by Tolkāppiyar and is the most ancient extant
Tamil grammar text and the oldest extant long work of Tamil literature.
Source: https://ddnews.gov.in/national/pm-modi-pays-tribute-saint-thiruvalluvar-thiruvalluvar-day

108.
Answer: A
Context- The Supreme Court on January 9, 2024 resumed hearing on the genetically modified (GM) mustard. The NGOs and
environmentalists have long protested the environmental release of the genetically modified food crops, alleging them to
be concerning for the health of those who would consume them.
Explanation:
Statement 1 is correct: The indigenously developed first GM food crop, HT Mustard DMH-11, received approval from the
government in October 2022, soon after which the field trials began.
Statement 2 is incorrect: It is a Herbicide Tolerant (HT) mustard variety that has undergone genetic modification. It has two
alien genes —»barnase» and «barstar» -- isolated from a soil bacterium called Bacillus, amyloliquefaciens. It enables the
breeding of high-yielding commercial mustard hybrids.
At present, Cotton is the only GM crop allowed for cultivation in India.
Statement 3 is incorrect: DMH-Dhara transgenic mustard has been developed by the Delhi university.
Source: GM mustard: Hearing starts afresh; SC hears arguments on both sides (downtoearth.org.in); Explained: What Is GM
Mustard, Why It Was Introduced And What Are The Concerns (outlookindia.com)
CSE 2024 January 2024 91
DTRS Expalanations
109.
Answer: A

Context: The share of India’s population living in multidimensional poverty is estimated to have fallen to 11.28 per cent in
2022-23 from 29.17 per cent in 2013-14, according to a discussion paper released by NITI Aayog in January 2024. In abso-
lute numbers, NITI Aayog estimates a total of 24.82 crore people escaped multidimensional poverty in the last nine years.
Explanation:
Statement 1 is incorrect: The three dimensions of the national Multidimensional Poverty Index are - Health, Education and
Standard of Living. It is modeled on the Global Multidimensional Poverty Index with some minor changes in indicators and
relative weights.
Statement 2 is correct: Bank Account is one of the indicators under the Standard of Living dimension. There are 7 indicators
under the standard of living dimension with each indicator having an equal weight of 1/21.
Statement 3 is incorrect: Under the Health dimension, there are 3 indicators - Nutrition, Child & Adolescent Mortality and
Maternal Health. But the weights are not equal. Nutrition has a weight of ⅙ alone whereas the other 2 indicators have
weights of 1/12 respectively.
Source: https://indianexpress.com/article/business/economy/indias-multidimensional-poverty-rate-down-to-11-28-
in-2022-23-from-29-17-in-2013-14-9110918/

110.
Answer: A
Explanation:
Context: The Supreme Court on 11th Jan 2024 questioned the Centre on why reports of the court-appointed Technical
Experts Committee (TEC) on the biosafety of genetically modified (GM) crops were not looked into by the Genetic Engi-
neering Appraisal Committee (GEAC).The Union Environment Ministry on Saturday rebutted the claims made by activists
that statutory regulations were violated during the appraisal and approval of GM mustard in India, saying the product was
conditionally cleared only after stakeholder consultation. The Coalition of GM-free India, a group of NGOs opposing genet-
ically modified crops, had on Friday released a report alleging that no (independent) health expert ever participated in GM
mustard appraisal.
CSE 2024 January 2024 92
DTRS Expalanations
Statement 1 is incorrect: Genetic Engineering Appraisal Committee (GEAC) was established under MoEFCC Rules 1989
which is the apex disposal body It functions under the Ministry of Environment, Forest and Climate Change (MoEF&CC).
y It seeks approval of activities involving the large-scale use of hazardous microorganisms and recombinants in re-
search and industrial production from an eco-mental perspective.
y India›s apex biotech regulatory committee, the Genetic Engineering Approval Committee (GEAC) functions as a
statutory body under the Environment Protection Act 1986 of the Ministry of Environment and Forests (MoEF).
Statement 2 and 4 incorrect:It is chaired by the Special Secretary/Additional Secretary of MoEF&CC and co-chaired by a
representative from the Department of Biotechnology (DBT).
y Presently, it has 24 members and meets every month to review the applications in the areas indicated above.
y It meets every month to review investigations in certain areas from an environmental perspective.
Statement 3 is correct: GEAC is also responsible for the approval of proposals related to the release of genetically engi-
neered organisms and products into the environment. Which includes experimental field trials (Biosafety Research Level
Trials-I and II known as BRL-I and BRL-II).

Source: https://indianexpress.com/article/explained/gm-mustard-supreme-court-9107917/
https://www.thehindu.com/news/national/geac-approves-commercial-cultivation-of-genetically-modified-mus-
tard-yet-again/article66058092.ece
https://timesofindia.indiatimes.com/topic/genetic-engineering-appraisal-committee
https://pib.gov.in/PressReleasePage.aspx?PRID=1897008

111.
Answer: D
Explanation:
Context: Fair trade regulator Competition Commission of India (CCI) on 15th Jan 2024 cleared Seventy Second Investment
Company›s proposed stake acquisition in Manipal Health Enterprises.

CSE 2024 January 2024 93


DTRS Expalanations
Statement 1 is not correct: The Competition Commission of India (CCI) is a statutory body of the Government of India re-
sponsible for enforcing the Competition Act, 2002.
y The Competition Act, 2002 prohibits anti-competitive agreements, abuse of dominant position by
y enterprises and regulates combinations (acquisition, acquiring of control and mergers & acquisitions), which caus-
es or likely to cause an appreciable adverse effect on competition within India.
¾ It is the duty of the Commission to eliminate practices having adverse effect on competition, promote and
sustain competition, protect the interests of consumers and ensure freedom of trade in the markets of India.
¾ The Commission is also required to give opinion on competition issues on a reference received from a statutory
authority established under any law and to undertake competition advocacy, create public awareness and
impart training on competition issues.
Statement 2 is not correct: Under Section 8(2) of the Competition Act, 2002, the Chairperson and every other Member
shall be a person of ability, integrity and standing and who, has been, or is qualified to be a judge of a High Court, or, has
special knowledge of, and professional experience of not less than fifteen years in international trade, economics, business,
commerce, law, finance, accountancy, management, industry, public affairs, administration or in any other matter which, in
the opinion of the Central Government, may be useful to the Commission.
Statement 3 is not correct: Under Section 410 of the Companies Act, 2013 and Section 53A(1) of the Competition Act, 2002,
the government replaced the Competition Appellate Tribunal (COMPAT) with the National Company Law Appellate Tribunal
(NCLAT) in 2017.
Source: https://health.economictimes.indiatimes.com/news/industry/cci-clears-ssis-stake-buy-in-manipal-health-enter-
prises/106879597

112.
Answer: B
Explanation:
Context: Experts said they recently updated the Model Code of Conduct with adapting to new tech-driven for fair election
conduct in the country.
y The Model Code of Conduct (MCC) consists of a set of guidelines expected to be abiding by the political parties,
their campaigners and the candidates. The Election Commission brings the Model Code of Conduct into effect to
ensure free and fair elections.
y The Model Code of Conduct remains in place during the elections in respect of political parties and candidates,
which remains in force from the date the elections are announced by the Commission till the completion of elec-
tions.
y Statement 1 is correct: The provisions of MCC and related instructions of the Commission issued from time to time
also apply to the content being posted on the internet, including social media websites, by candidates and political
parties.
y Statement 3 is not correct: The State of Kerala adopted MCC for the first time in India during the general election
to the State Legislative Assembly in February 1960.
y Statement 2 is correct: Though the Election Commission of India does not have a judicial ability to penalise the
violators of the Model Code of Conduct, it generally issues show-cause notice to the violators and in some cases,
bars them from the poll campaign.
Source: https://economictimes.indiatimes.com/news/politics-and-nation/is-it-time-to-update-model-code-of-conduct-
adapting-to-tech-driven-election-landscape/articleshow/106818159.cms?from=mdr

CSE 2024 January 2024 94


DTRS Expalanations
113.
Answer: B
Context: The Makaravilakku festival, which culminates the two-month-long annual pilgrimage season at Sabarimala, was
held on the hillock recently amid a fervently devotional mood.
Explanation:
Makaravilakku is an annual festival held on Makar Sankranti in Kerala, India at the shrine of Sabarimala. The festival includes
the Thiruvabharanam (sacred ornaments of Lord Ayyappan) procession and a congregation at the hill shrine of Sabarimala.
Additional information:
y Mahamastakabhisheka festival refers to the abhiṣeka (anointment) of the Jain images when held on a large scale.
The most famous of such consecrations is the anointing of the Bahubali Gommateshwara statue located at Shrava-
nabelagola in Karnataka, India. It is an important Jain festival held once every 12 years. It is an integral part of the
ancient and composite Jain tradition.
y Mahamaham festival is one of the festivals celebrated in South India every 12 years in the Mahamaham Tank that’s
located in Kumbakonam. This 20 acre tank surrounded by Lord Shiva’s mandapams is believed to be sacred. It is
considered that on the day of the festival, the river goddess and Lord Shiva rejuvenate its waters. Thus the Hindus
consider taking a dip in the tank as very auspicious. It is often said to be equivalent to attending the Kumba Mela.
This festival is held over a 10 day period.
y Ugadi festival is one of the festivals celebrated in South India, especially in Andhra Pradesh, Karnataka and Telanga-
na. This south Indian festival marks the beginning of a new Hindu calendar year and is celebrated on the first day
of “Chaitra” month, which falls in the months of March-April as per the Gregorian calendar. Ugadi is recognized by
many as the day Kali Yuga officially began, when Krishna ended his time on earth.
Source: https://www.thehindu.com/news/national/kerala/makaravilakku-festival-celebrated-at-sabarimala/arti-
cle67743136.ece

114.
Answer: B
Context: Recently, the Union government proposed that all education records of school students will be integrated into its
‘One Nation, One Student ID’ initiative by 2026-27 under the National Education Policy (NEP) 2020.
Explanation:
Statement 1 is correct: The initiative aims to digitally consolidate the complete academic records, including degrees, schol-
arships, rewards, and other credits into a unified Automated Permanent Academic Account Registry (APAAR) ID.
y The program is open to all students.
Statement 2 is correct: APAAR is a lifelong ID for students to track their educational journey and achievements, from
pre-primary to higher education, and will not replace the Aadhaar but complement it for educational tracking purposes.
Statement 3 is incorrect: Though the government is encouraging all students to actively participate in this initiative, the
participation is voluntary, not mandatory.
y Parents have the choice to provide consent for their child’s APAAR ID.
y The Education Ministry has proposed a deadline of 2026-27 for 100% integration of the education records of all
students through the ID.
Source: https://www.livemint.com/education/news/what-is-apaar-id-how-is-it-different-from-aadhaar-11705255162936.
html
CSE 2024 January 2024 95
DTRS Expalanations
115.
Answer: B
Context: Prime Minister Narendra Modi visited the Veerabhadra Temple in Andhra Pradesh›s Lepakshi - a place that holds
important significance in Hindu epic Ramayana.
Explanation:
Statement 1 is correct: Sri Veerabhadra temple stands as a model of art, architecture and culture of mid- Vijayanagara
Period. Temple has about 70 pillars.
y While sixty-nine other pillars support the ceiling, one corner pillar, or the hanging pillar, does not touch the temple
floor at all.
y There is a small gap between the temple floor and base of the pillar and you can pass thin objects like a piece of
cloth from one side to the other.
Statement 2 is incorrect: It is not an UNESCO site yet. In 2022, it was added in the Tentative list of UNESCO World Heritage
sites.
Statement 3 is correct: The temple bears witness to the only surviving fresco paintings of mid Vijayanagara Period.
y The fresco paintings like Kiratarjuniyam, Girija Kalyana, and Shiva-Parvati marriage illustrate various narratives
from the puranas and epics that contribute to the field of art and ethnology.
y The technique of Vijayanagara paintings formed a base for the art of fresco paintings continued by the successor
and other kingdoms of South India.
Source: https://www.ndtv.com/india-news/ram-temple-lepakshi-ahead-of-ayodhya-event-pm-modi-visits-historic-ra-
mayana-site-in-andhra-4872733

116.
Answer: A
Context: Ministry of Tourism (MoT) in collaboration with Ministry of Environment, Forest & Climate Change
(MoEF&CC) launched the fourth training programme under the Amrit Dharohar Capacity Building Scheme
for local community members, boat owners, and Forest department officials of Bhitarkanika National Park.
Explanation:
Statement 1 is correct: It is a collaborative effort between the Ministry of Tourism (MoT) and the Ministry of Environment,
Forest & Climate Change (MoEFCC).
Statement 2 is incorrect: It aims to promote unique conservation values of the Ramsar Sites in the country while generating
employment opportunities and supporting local livelihoods.
Additional information:
y It was announced as part of the 2023-24 budget by MoEF&CC during June 2023.
y Emphasis On: It emphasize on the importance of wetlands and their preservation, with an outlook that is inclusive
of local communities as caretakers of the ecosystem.
y Implementation By: In convergence with various Central Government ministries and agencies, State Wetland Au-
thorities, and a network of formal and informal institutions and individuals, working together for a common cause.
y Significance: A scheme that will be implemented over the next three years to encourage optimal use of wetlands,
and enhance biodiversity, carbon stock, eco-tourism opportunities, and income generation for local communities.
Source: https://pib.gov.in/PressReleasePage.aspx?PRID=1993555

CSE 2024 January 2024 96


DTRS Expalanations
117.
Answer: A
Context: The near-complete Lower Orr dam as part of the Ken-Betwa interlinking of rivers project has yet not received the
final environmental clearance.
Explanation:
Statement 1 is correct: The construction of Lower Orr Dam is undertaken as part of the Ken-Betwa river-linking project.
The National Perspective Plan (NPP) for Water Resources Development was formulated by the Government of India in
1980. The NPP comprises two components, viz; Peninsular Rivers Development Component and Himalayan Rivers Develop-
ment Component. The National Water Development Agency (NWDA) has been entrusted with the work of Interlinking of
Rivers under the NPP.
The Ken-Betwa Link project (KBLP) is the first link under the NPP for which implementation has been initiated. The Gov-
ernment of India approved the implementation of the project in December, 2021. The KBLP will benefit the Bundelkhand
region of Madhya Pradesh (MP) and Uttar Pradesh (UP), which faces recurrent drought situation. The project will provide
annual irrigation to an area of 10.62 lakh hactare (8.11 lakh hactare in MP and 2.51 lakh hactare in UP) in the Chhattarpur,
Tikamgarh, Panna, Sagar, Damoh and Datia districts of Madhya Pradesh and Banda, Mahoba, Jhansi and Lalitpur districts of
Uttar Pradesh in Bundelkhand region as well as to the Vidisha, Shivpuri and Raisen districts of Madhya Pradesh.

Statement 2 is incorrect: Orr is a tributary of river Betwa and not river Ken. Both Ken and Betwa are tributaries of river
Yamuna.
The Ken River originates near the village Ahirgawan on the north-west slopes of Barner Range in Katni district of Madhya
Pradesh and travels a distance of 427 km, before merging with the Yamuna at Chilla village, district Banda in Uttar Pradesh.
River Betwa originates in the Vindhyan range just north of Hoshangabad in Madhya Pradesh.
The interlinkage of rivers Betwa and Ken is likely to benefit the drought-prone region of Bundelkhand.
Source: A near-complete dam linked to Ken-Betwa project is yet to get environment clearance - The Hindu; WRIS Website ;
pib.gov.in/PressReleaseIframePage.aspx?PRID=1907654

118.
Answer: B
Explanation:
Context: Recently, Kuchipudi dancer Pendyala Lakshmi Priya to receive Pradhan Mantri Rashtriya Bal Puraskar by the Pres-
ident of India.
CSE 2024 January 2024 97
DTRS Expalanations
1. Kathakali- Kerala
y Kathakali is a classical dance-drama form of Kerala, India. It is a highly stylized and dramatic dance form that tells
stories from Hindu mythology and epics. Kathakali dancers wear elaborate costumes and makeup, and they use
facial expressions, hand gestures, and body movements to tell the story.

2. Sattriya- Assam
y Sattriya is a classical dance form of Assam, India. It is a spiritual dance form that is associated with the Vaish-
navite Sattra monasteries of Assam. Sattriya dancers wear simple, flowing costumes, and they use graceful
movements and gestures to express spiritual themes.

3. Kathak- Uttar Pradesh


y Kathak is a classical dance form of Uttar Pradesh, India. It is a vibrant and energetic dance form that is characterized
by its intricate footwork and fluid movements. Kathak dancers wear elaborate costumes and jewelry, and they use
a variety of musical instruments to accompany their performances.

CSE 2024 January 2024 98


DTRS Expalanations

4. Kuchipudi- Andhra Pradesh


y Kuchipudi is a classical dance form of Andhra Pradesh, India. It is a graceful and elegant dance form that is charac-
terized by its intricate footwork and expressive movements. Kuchipudi dancers wear colorful costumes and jewel-
ry, and they use a variety of musical instruments to accompany their performances.

Source: https://newsonair.gov.in/News?title=Kuchipudi-dancer-Pendyala-Lakshmi-Priya-to-receive-Pradhan-Mantri-Rash-
triya-Bal-Puraskar-by-Prez-Murmu&id=475729

CSE 2024 January 2024 99


DTRS Expalanations
119.
Answer: B
Context: Khanij Bidesh India Ltd (KABIL), a joint venture of public sector enterprises NALCO, Hindustan Copper and Mineral
Exploration Company, has signed an agreement with Argentina›s state-owned CAMYEN for lithium exploration and mining.
Explanation:
Statement 1 is incorrect: Lithium is a metal that has the lowest density.
Properties:
y It is a chemical element with the symbol Li.
y It is the lightest of all metals.
y Being an alkali metal, it is a soft, flammable, and highly reactive metal that tends to form hydroxides.
Statement 2 is correct:
Statement 3 is correct. Lithium is a semi rare earth metal. The production of this metal is still quite scarce and the supply
capacity does not meet the world demand. India meets all its lithium demand from imports.
y Lithium is currently produced from hard rock or brine mines.
y Top producers of the metal:

y India meets all its lithium demand from imports. The reserves of lithium in India have been found in Jammu &
Kashmir, Rajasthan and Jharkhand.
y The Lithium Triangle is an area in the Andes, covering Argentina, Bolivia, and Chile, and known for its abundant
lithium deposits.
Source: https://pib.gov.in/PressReleseDetail.aspx?PRID=1996380

120.
Answer: C
Context: The Indian Army is launching Operation Sarvashakti involving security forces in Jammu and Kashmir to flush out
terrorists who have been involved in a series of ambushes on Indian troops carried out in areas of Rajouri and Poonch in
January 2024.

CSE 2024 January 2024 100


DTRS Expalanations
Explanation:
Troops of the Srinagar-based 15 Corps and the Nagrota-based 16 Corps will be operating in tandem, along with other agen-
cies and paramilitary forces, as part of the operation to wipe off terrorists sponsored by Pakistan.
The operation will be on the lines of Operation Sarp Vinash of 2003, which was launched against terrorists south of the Pir
Panjal range. The operation had lasted for roughly three months and nearly 100 terrorists were killed at that time. A large
number of stores and equipment, including some medical ones, were recovered.
Source:https://indianexpress.com/article/india/army-launches-operation-sarvashakti-to-flush-out-terrorists-9108423/

121.
Answer: B
Explanation:
Context: Enforcement Directorate summons BRS leader K Kavitha for Delhi excise policy case inquiry. The 45-year-old daugh-
ter of former Telangana CM K Chandrashekhar Rao has been asked to depose before the agency in Delhi on 15th Jan 2024.
The Directorate of Enforcement or the ED is a multi-disciplinary organization mandated to investigate economic crimes and
violations of foreign exchange laws. The origin of this Directorate goes back to 1st May 1956, when an ‘Enforcement Unit›
was formed in the Department of Economic Affairs for handling Exchange Control Laws violations under Foreign Exchange
Regulation Act, 1947 (FERA ’47).
y In the year 1957, this Unit was renamed as ‘Enforcement Directorate›. Presently, the Directorate is under the ad-
ministrative control of the Department of Revenue, Ministry of Finance, Government of India.
y With the onset of the process of economic liberalization, FERA, 1973, which was a regulatory law, was repealed and
in its place, a new law viz. the Foreign Exchange Management Act, 1999 (FEMA) came into operation w.e.f. 1st June
2000. Further, in tune with the International Anti-Money Laundering regime, the Prevention of Money Laundering
Act, 2002 (PMLA) was enacted, and ED was entrusted with its enforcement w.e.f. 1st July 2005. Recently, with the
increase in the number of cases relating to economic offenders taking shelter in foreign countries, the Government
has passed the Fugitive Economic Offenders Act, 2018 (FEOA), and ED is entrusted with its enforcement with effect
from 21st April 2018. Hence statement 1 is not correct.
y The ED is the only central investigative agency that does not require permission from the government or any au-
thority to summon or prosecute politicians or government officials for inquiry into economic offenses and financial
crimes like money laundering. Hence statement 3 is correct.
The Prevention of Money Laundering Act, 2002 (PMLA):
y It is a criminal law enacted to prevent money laundering and to provide for confiscation of property derived from,
or involved in, money-laundering and for matters connected therewith or incidental thereto.
y ED has been given the responsibility to enforce the provisions of the PMLA by conducting an investigation to trace
the assets derived from proceeds of crime, provisionally attach the property, and ensuring the prosecution of the
offenders and confiscation of the property by the Special court.
y ED can start an investigation against anyone without complaint or application. An ED officer does not even have to
file a First Information Report (FIR).
y The FIR is a mandatory provision under the Criminal Procedure Code and the Indian Penal Code.
y Without an FIR, an investigating officer does not have the authority to proceed with an investigation.
y However, under the PMLA, an officer just needs to file an Enforcement Case Information Report (ECIR) which is for
internal use and need not be shared with the accused.
y The PMLA gives authority to an investigating officer to arrest an accused without prior intimation, even without
sharing the ECIR. An investigating officer can also issue summons without giving any specific reason. Hence state-
ment 2 is correct.

CSE 2024 January 2024 101


DTRS Expalanations
Source: https://www.newindianexpress.com/nation/2024/Jan/15/ed-summons-brs-leader-kavitha-for-questioning-in-del-
hi-excise-policy-case-on-tuesday

122.
Answer: A
Explanation:
Context: The Ministry of Road Transport and Highways (MoRTH) will come out with a modified Build-Operate-Transfer (BOT)
project document in the next one month to attract interest of private companies for investments in the highways sector.
Addressing an event organised by Rural Electrification Capital (REC) Ltd, Road Transport and Highways Secretary Anurag Jain
said the government›s budget on the highways sector has increased three times in the last 10 years.
y Public-Private Partnership (PPP) means an arrangement between a Government / statutory entity / Government
owned entity on one side and a private sector entity on the other, for the provision of public assets and/or public
services, through investments being made and/or management being undertaken by the private sector entity, for
a specified period of time, where there is well-defined allocation of risk between the private sector and the public
entity and the private entity who is chosen on the basis of open competitive bidding, receives performance-linked
payments that conform (or are benchmarked) to specified and pre-determined performance standards, measur-
able by the public entity or its representative.
y The most common form of PPP used is BOT(Build-Operate-Transfer) where the private sector operator designs,
builds, finances, owns and constructs the facility and operates it commercially for the concession period, after
which the facility is transferred to the authority. In this case, legal ownership of the asset vests with the public
sector as the concession period ends. The most common form of a BOT project is a Toll Road project. Hence option
(a) is the correct answer.
y Operations & Maintenance (Service contract): the Government bids out the right to deliver a specific service or
gives part of the undertaking to the private sector for operations and maintenance of the assets.
y Such contracts are normally of a shorter duration than concession contracts.
y Engineering, Procurement and Construction (EPC): This is a PPP model for the development of infrastructure proj-
ects, especially highways. Under this model, the cost is completely borne by the government. Government invites
bids for engineering knowledge from private players. Procurement of raw materials and construction costs are met
by the government. The private sector’s participation is minimum and is limited to the provision of engineering
expertise. A difficulty of the model is that finance is a high financial burden for the government.
Source: https://economictimes.indiatimes.com/industry/transportation/roadways/morth-to-come-out-with-bot-project-
in-the-next-month-to-attract-investments-anurag-jain/articleshow/106644196.cms?from=mdr

123.
Answer: D
Context: A United Nations report World Economic Situation and Prospects (WESP) report revised India’s gross domestic
product (GDP) growth forecast for the 2024 calendar year to 6.2 per cent from its earlier estimate of 6.7 per cent.
Explanation:
The World Economic Situation and Prospects is a report produced by the United Nations Department of Economic and
Social Affairs (UN DESA), in partnership with the United Nations Conference on Trade and Development (UNCTAD) and the
five United Nations regional commissions: Economic Commission for Africa (UNECA), Economic Commission for Europe
(UNECE), Economic Commission for Latin America and the Caribbean (UNECLAC), Economic and Social Commission for Asia
and the Pacific (UNESCAP) and Economic and Social Commission for Western Asia (UNESCWA). The United Nations World
Tourism Organization (UNWTO) also contributed to the report.

CSE 2024 January 2024 102


DTRS Expalanations
Additional information:
Findings of the recent World Economic Situation and Prospects report:
y Global economic growth is projected to slow from an estimated 2.7 per cent in 2023 to 2.4 per cent in 2024, trend-
ing below the pre-pandemic growth rate of 3.0 per cent, according to the United Nations World Economic Situation
and Prospects (WESP) 2024.
y This latest forecast comes on the heels of global economic performance exceeding expectations in 2023. However,
last year’s stronger-than-expected GDP growth masked short-term risks and structural vulnerabilities.
y The UN’s flagship economic report presents a sombre economic outlook for the near term. Persistently high in-
terest rates, further escalation of conflicts, sluggish international trade, and increasing climate disasters, pose
significant challenges to global growth.
y The prospects of a prolonged period of tighter credit conditions and higher borrowing costs present strong head-
winds for a world economy saddled with debt, while in need of more investments to resuscitate growth, fight
climate change and accelerate progress towards the Sustainable Development Goals (SDGs).
y in several large, developed economies, especially the United States, is projected to decelerate in 2024 given high
interest rates, slowing consumer spending and weaker labour markets.
y The short-term growth prospects for many developing countries – particularly in East Asia, Western Asia and Latin
America and the Caribbean – are also deteriorating because of tighter financial conditions, shrinking fiscal space
and sluggish external demand.
y Low-income and vulnerable economies are facing increasing balance-of-payments pressures and debt sustainabil-
ity risks. Economic prospects for small island developing States, in particular, will be constrained by heavy debt
burdens, high interest rates and increasing climate-related vulnerabilities, which threaten to undermine, and in
some cases, even reverse gains made on the SDGs.
Source: https://indianexpress.com/article/business/economy/un-report-revises-indias-2024-gdp-growth-forecast-
downward-to-6-2-9097171/

124.
Answer: D
Explanation:
Context: IIT Delhi team makes first hi-res landslide risk map for India. The map data is available for free; the researchers
have also created an online portal for people to explore the map.
y Map will help government and agencies such as National Disaster Management Authority (NDMA) in managing
landslides.
y It has divided country into five classes (refer to map) on the basis of vulnerability.
y Map revealed previously unknown places with high risk, such as some areas of the Eastern Ghats, north of Andhra
Pradesh and Odisha.
About the Landslide Map:
y Data Collection: Graduate student Nirdesh Sharma collected data on 1.5 lakh landslide events from sources includ-
ing the Geological Survey of India.
y Factors considered: 16 factors influencing landslide susceptibility, like soil cover, tree density,etc
y High-Resolution Map: A high-resolution ‘Indian Landslide Susceptibility Map’ was developed with a 100m resolu-
tion, covering the entire nation.
CSE 2024 January 2024 103
DTRS Expalanations

Contribution:
y The map is hailed as an excellent contribution, providing the first comprehensive national-scale landslide proba-
bilities.
y Identified vulnerable regions include parts of the Himalayan foothills, Assam-Meghalaya, Western Ghats, and pre-
viously unknown risk areas.
y Ensemble machine learning methods.
Future Initiatives:
y Plans to develop a ‘Landslide Early Warning System’ using map data.
y Beneficial for organizations like Geological Survey of India and National Disaster Management Authority
Accessibility:
y The map, available online for free, offers public-friendly exploration through a user interface.
Source: https://www.thehindu.com/sci-tech/science/iit-delhi-team-first-hi-res-landslide-risk-map-india/arti-
cle67701918.ece

CSE 2024 January 2024 104


DTRS Expalanations

CSE 2024 January 2024 105


DTRS Expalanations
125.
Answer: C
Explanation:
Context: World Economic Forum (WEF) released report titled ‘Green Hydrogen: Enabling Measures Roadmap for Adop-
tion in India’.
y This report is released by the World Economic Forum, written in collaboration with Bain & Company.
y It recommended five areas that can benefit from public-private interventions in expediting the adoption of green
hydrogen in India.
y Hydrogen produced through the electrolysis of water, powered by renewable energy, can not only provide energy
for sectors such as fertilizers, chemicals, refining and iron, but can also save carbon emissions compared to fossil
fuels.
y This report provides a detailed analysis of what it will take to make green hydrogen a credible energy pathway in
India.
y The overarching aim is to turn the report›s recommendations into concrete measures and accelerate action on the
path to net-zero.
Green hydrogen:
y Green Hydrogen is defined as hydrogen produced using renewable energy including production through electrol-
ysis or conversion of biomasses.
y Currently, India produces 6.5 million metric tonnes per annum (MMTPA) of hydrogen, predominantly for use in
crude-oil refineries and fertilizer production.
Need for adoption of Green Hydrogen
y Decarbonize sectors like transportation, shipping, and steel, etc. and achieve net zero emission by 2070.
y Achieve high energy demand, which is estimated to grow 35% by 2030.
Concerns with Green Hydrogen
y High production costs of green hydrogen amount to roughly $4-5 per kilogram (kg).
y Renewable energy is not available round-the-clock and non- economical battery storage.
y Trade barriers with renewable resource-deficient countries like Europe, Korea and Japan which are anticipated to
emerge as crucial markets for green hydrogen.
Source: https://www.weforum.org/publications/green-hydrogen-enabling-measures-roadmap-for-adoption-in-in-
dia/#:~:text=Green%20Hydrogen%3A%20Enabling%20Measures%20Roadmap%20for%20Adoption%20in%20India&tex-
t=This%20report%20by%20the%20World,of%20green%20hydrogen%20in%20India.

126.
Answer: C
Context: Recently, the Prime Minister inaugurated the Mumbai Trans Harbour Link (MTHL), also known as Atal Setu, mark-
ing the opening of India’s longest sea bridge to date.
Explanation:
Statement 1 is correct: It is not only the longest sea bridge but it is also the longest bridge in India over water. The total
length of MTHL Bridge is 21.8 km in which length on sea is 16.50 km and on land portion is 5.5 km. Earlier, Bhupen Hazarika
bridge over Brahmaputra river, spanning 9.1km, was the longest bridge in India over water.

CSE 2024 January 2024 106


DTRS Expalanations
Statement 1 is correct: The entire cost of about Rs 17900 crore has been borne by the government through its agency
Mumbai Metropolitan Region Development Authority (MMRDA). The Japan International Cooperation Agency (JICA) has
provided 85% of the loan. The project has been undertaken via the EPC model.
Engineering Procurement and Construction (EPC) Model
y Under this system the entire project is funded by the government.
y The contractor builds the project by designing, installing and procuring necessary labour and land to construct the
infrastructure, either directly or by subcontracting.
y Under EPC model the contractor is legally responsible to complete the project under some fixed predetermined
timeline and may also involve scope for penalty in case of time overrun.
y In EPC as all the clearances, land acquisition and regulatory norms have to be completed by the government itself
and the private players do not have to get themselves involved in these time taking procedures.
Source: https://www.livemint.com/industry/how-mumbai-trans-harbour-link-mthl-road-will-be-a-game-changer-for-
real-estate-explained-11704959350645.html

127.
Answer: B
Recently, the cities in Uttar Pradesh have been developed under River Cities Alliance (RCA), a joint initiative of the Ministry
of Jal Shakti (MoJS) & the Ministry of Housing and Urban Affairs (MoHUA).
About River Cities Alliance (RCA):
Statement 2 is correct: The RCA is a collaborative effort between the National Mission for Clean Ganga (NMCG) and the
National Institute for Urban Affairs (NIUA). It is launched by the Ministry of Jal Shakti along with the Ministry of Housing
and Urban Affairs. River Cities Alliance started with 30 cities in 2021 and currently has 95 cities as members across India.
Statement 1 is not correct:
y Open to all: Any river city in India can join the RCA at any time.
y Focus areas: The Alliance focuses on three broad themes:
¾ Networking: Facilitating knowledge exchange and collaboration among member cities.
¾ Capacity building: Providing training and resources to help cities improve their river management practices.
¾ Technical support: Offering expert guidance and assistance to cities on specific river management challenges.
y Since its launch in 2021, the RCA has grown to include over 110 member cities across India and one international
member city from Denmark.
y The Alliance has organised various workshops, conferences, and knowledge-sharing
y events to help member cities learn from each other and best practices in urban river management.
y The RCA has facilitated the development of several city-specific River Management Plans and provided technical
support for their implementation.
Source: https://timesofindia.indiatimes.com/city/allahabad/development-of-cities-in-up-under-river-cities-alliance/arti-
cleshow/106225384.cms

128.
Answer: B
Indonesia’s Mount Lewotobi Laki-Laki erupted anew 10th jan 2024, spewing ash clouds two kilometres (6,561 feet) above
its peak as authorities raised the volcano’s alert status to the highest level.
CSE 2024 January 2024 107
DTRS Expalanations
A volcano is a place where gases, ashes and/or molten rock material – lava – escape to the ground. A volcano is called an
active volcano if the materials mentioned are being released or have been released in the recent past.
y The layer below the solid crust is mantle. It has a higher density than that of the crust. The mantle contains a
weaker zone called the asthenosphere. It is from this that the molten rock materials find their way to the surface.
Volcanoes are classified on the basis of the nature of eruption and the form developed at the surface.
Major types of volcanoes are as follows
Shield Volcanoes
9 Barring the basalt flows, the shield volcanoes are the largest of all the volcanoes on the earth. The Hawaiian vol-
canoes are the most famous
9 examples. These volcanoes are mostly made up of basalt, a type of lava that is very fluid when erupted.
9 For this reason, these volcanoes are not steep. They become explosive if somehow water gets into the vent; other-
wise, they are characterised by low-explosivity. The upcoming lava moves in the form of a fountain and throws out
the cone at the top of the vent and develops into cinder cone.
Composite Volcanoes
9 These volcanoes are characterised by eruptions of cooler and more viscous lavas than basalt. These volcanoes
often result in explosive eruptions.
9 Along with lava, large quantities of pyroclastic material and ashes find their way to the ground. This material ac-
cumulates in the vicinity of the vent openings leading to formation of layers, and this makes the mounts appear
as composite volcanoes.
y Composite volcanoes, also known as stratovolcanoes, are tall, steep-sided volcanoes formed from alternating
layers of lava flows, ash, and other volcanic debris.
They are characterised by their distinctive conical shape and their explosive eruptions.
y Some of the features of composite volcanoes are Steep-sided cone shape: Composite volcanoes have a tall, conical
shape, with steep sides that can reach up to 30 degrees.
Crater: At the summit of the volcano, there is often a bowl-shaped depression called a crater. This is the vent through which
lava, ash, and other volcanic materials are ejected during an eruption.
Lava flow: Composite volcanoes are typically composed of both explosive eruptions and effusive eruptions, which produce
lava flows. The lava flows from these volcanoes can be quite thick and viscous, meaning that they don›t travel very far from
the vent.
Lahars: Composite volcanoes are prone to producing lahars, which are fast-moving mudflows that are triggered by volcanic
activity. Lahars can be highly destructive, as they can carry large boulders, trees, and other debris, and can travel many
kilometers from the volcano.
Source: https://www.news18.com/world/indonesia-raises-volcano-alert-status-to-highest-level-as-lewotobi-laki-la-
ki-erupts-8734512.html

129.
Answer: B
Context: Prime Minister has paid tributes to Swami Vivekananda on his birth anniversary and National Youth Day on 12th
january.
Explanation:
Statement 1 is incorrect: In 1893, upon the request of Maharaja Ajit Singh of the Khetri State, he took the name ‘Vivekanan-
da.’ Netaji Subhas Chandra Bose had called Vivekananda the “maker of modern India
CSE 2024 January 2024 108
DTRS Expalanations
Statement 2 is correct: He was the chief disciple of the 19th-century mystic Ramakrishna Paramhansa and established the
Ramakrishna Mission in 1897.
Additional information:
Contributions of Vivekananda:
y Introduced the world to the Indian philosophies of Vedanta and Yoga.
y He preached ‘neo-Vedanta’, an interpretation of Hinduism through a Western lens, and believed in combining
spirituality with material progress.
y Laid the greatest emphasis on education for the regeneration of our motherland. Advocated a man-making char-
acter-building education.
y Best known for his speech at the World Parliament of Religion in Chicago in 1893.
y Spelt out the four pathways of attaining moksha from the worldly pleasure and attachment in his books - Raja-yo-
ga, Karma-yoga, Jnana-yoga and Bhakti-yoga.
Source: https://newsonair.gov.in/Main-News-Details.aspx?id=475033

130.
Answer: D
Explanation:
Context: In the Union government’s annual cleanliness rankings for urban areas, Indore and Surat emerged as joint winners
of the cleanest city award, marking Indore›s seventh consecutive year at the top.
Statement 1 is correct. It is conducted under the ambit of the Swachh Bharat Mission (Urban).
Statement 2 is correct. Recently, Indore and Surat emerged as joint winners of the cleanest city in Swachh Survekshan
Awards.
y Additionally, Maharashtra secured the cleanest state accolade.
y The rankings consider various factors, including door-to-door waste collection, segregation at source, cleanliness
of public areas, the state of water bodies, and citizens’ feedback on their cities› cleanliness.
y Indore›s remarkable streak continued with its seventh consecutive win, showcasing 100% door-to-door waste
collection, 98% segregation at source, and 100% remediation of dumpsites.
y Surat, consistently second behind Indore in recent years, shared the top spot this year, boasting similar cleanliness
metrics.
y With 89.24% door-to-door waste collection and 67.76% source segregation, Maharashtra claimed the title of the
cleanest state.
y Madhya Pradesh secured the second position with 90.59% door-to-door collection and 54.1% source segregation.
Additional information:
Swachh Survekshan Awards:
y Swachh Survekshan has been conducted since 2016 and is the world’s largest urban sanitation and cleanliness
survey.
y It has been instrumental in fostering a spirit of healthy competition among towns and cities to improve their ser-
vice delivery to citizens and towards creating cleaner cities.
y The primary goal of Swachh Survekshan is to encourage large-scale citizen participation and create awareness
amongst all sections of society about the importance of working together toward making towns and cities better
places to reside in.
CSE 2024 January 2024 109
DTRS Expalanations
y Nodal Ministry: Ministry of Housing and Urban Affairs (MoHUA).
Source: https://indianexpress.com/article/india/indore-surat-cleanest-cities-maharashtra-cleanest-9105539/

131.
Answer: B
Explanation:
Context: In a significant archaeological find, a hero stone (veera kallu) adorned with a Kannada script from the 15th-16th
century has been unearthed at Ulturu-Kattemane near Gulladi in Kundapur Taluk of Udupi district, Karnataka. This discovery
sheds light on the rich historical and cultural heritage of the region.
Historical Context
y The hero stone, a rectangular slab standing about six feet tall, contains a sculptural panel at the top and an inscrip-
tion in four lines below it.
y The inscription has been written in the Kannada script of the 15th-16th century.
y It unveils details of an event related to the death of Lavamaaluva and Valeya, a servant.
y The hero stone was erected by an unnamed individual in their memory.
y The inscription begins with an invocation to Lord Ganesh, setting the historical tone for the narrative.

y The sculptural panel on the hero stone intricately captures the essence of the historical event.
y At the center, a heroic figure stands in veerasana, brandishing a sword pointed towards the sky, while holding a
shield or Guraani in the left hand.
y This figure, adorned in royal attire with a crown (kirita), is flanked by the moon on the right and the sun on the left.
y The symbolism embedded in the panel narrates a poignant event where a member of a royal family lost his life in
an unknown battle.
y The figure on the right portrays a servant stabbing himself in grief, holding a shield in one hand.
CSE 2024 January 2024 110
DTRS Expalanations
Cultural Significance: Bobbaryana Kallu
y The hero stone, locally known as Bobbaryana Kallu, holds cultural and religious significance among the local pop-
ulace.
y This stone, standing outside the fortified village of Gulladi, is revered and worshipped by the community.
y The historian notes the stone›s role in Bhuta cult (daiva worship), where Bobbarya or Babbu is perceived in a ser-
vitude position, aligning with the social stratification evident in the historical narrative.
Historical Significance and Connections
y The discovery gains added importance in the broader historical context of the region.
y Approximately eight months prior, a Chikkayi Tayi inscription, related to an Alupa queen married to Hoysala Ballala
III, was found in the same area.
y The identification of Gulladi as the potential maternal home of Chikkayi Tayi introduces layers of historical connec-
tions.
y Further analysis suggests that Lava Alupa, likely a royal member of Gulladi, met his demise in an unidentified battle,
leading to the hero stone being erected in his memory.
Insights into Social Stratification
y The hero stone not only provides a glimpse into historical events but also offers insights into social stratification.
y The death of Lava Alupa›s servant, Valeya or VeLevaLi, who belonged to a servitude status, is noted.
y The servant›s cremation outside the fortified village of Gulladi aligns with societal norms of the time.
Source: https://www.thehindu.com/news/cities/Mangalore/hero-stone-from-15th-16th-century-with-inscription-in-
kannada-script-found-near-kundapur/article67729776.ece

132.
Answer: B
Explanation:
Context: Recently, Russia-led Collective Security Treaty Organization (CSTO) is being discussed more and more actively as
differences grow between Yerevan and Moscow.
The Collective Security Treaty Organization originates from the conclusion of the Collective Security Treaty, which was
signed in Tashkent (Uzbekistan) on May 15, 1992. The treaty entered into force upon completion of the national ratification
procedures on April 20, 1994.
y The Organization today includes: the Republic of Armenia, the Republic of Belarus, the Republic of Kazakhstan, the
Kyrgyz Republic, the Russian Federation and the Republic of Tajikistan (in the summer of 2006, the CSTO resumed
its membership, and in 2012 the Republic of Uzbekistan suspended it).
y It is not a European Union organisation. Hence, statement 1 is not correct.
In accordance with Article 3 of the Charter, the objectives of the Organization are the strengthening of peace, international
and regional security and stability, the protection on a collective basis of the independence, territorial integrity and sov-
ereignty of the member States. The document defines the following principles by which the Organization is guided in its
activities: priority of political means over the military, strict respect for independence, voluntary participation, equality of
rights and obligations of the member States, non-interference in affairs falling under the national jurisdiction of the member
States.
Statement 3 is correct: Article 4 of the Treaty states: “If one of the States Parties is subjected to aggression by any state or
group of states, then this will be considered as aggression against all States Parties to this Treaty. In the event of an act of ag-
CSE 2024 January 2024 111
DTRS Expalanations
gression against any of the participating States, all other participating States will provide him with the necessary assistance,
including military, and will also provide support at their disposal in exercising the right to collective defense in accordance
with Article 51 of the UN Charter.”
Statement 2 is correct: The Collective Security Council (CSC) is the highest body of the Organization and consists of heads
of the member States. It considers the fundamental issues of the Organization’s activities and makes decisions aimed at the
realization of its goals and objectives, as well as provides coordination and joint activities of the member States for the re-
alization of these goals. Sessions of the CSC are held alternately in the member States as necessary, but at least once a year.
The chairmanship of the Council is transferred in the order of the Russian alphabet, unless the Council decides otherwise.
Source: https://eurasianet.org/armenia-considers-possible-future-outside-russia-led-military-bloc

133.
Answer: B
Explanation:
Context: The World Health Organization (WHO) has recently prequalified the novel type 2 oral poliomyelitis (polio) vaccine,
nOPV2, to protect children against the disease. The prequalification will ensure broad and long-term accessibility for inter-
national agencies to deploy nOPV2 in developing countries.
y Poliomyelitis (polio) is a highly infectious viral disease. Polio usually affects children under five, sometimes leading
to irreversible paralysis. Death can occur when breathing muscles are affected. Hence, statement 1 is correct.
y It may affect the spinal cord causing muscle weakness and paralysis. Hence, statement 3 is correct.
y It can be transmitted by person-to-person spread mainly through the faecal-oral route or, less frequently, by a
contaminated water or food.
y Recently, Malawi has recorded Africa’s first wild poliovirus (WPV) case in five years. Wild polio is caught from the
environment, but there is another type of polio linked to the oral vaccine (which contains live, weakened virus) that
is equally worrying. It can linger in the gut, mutate and spread in areas where few people are vaccinated. There
have been outbreaks of this form of polio in more than 20 African countries in recent years. There is no cure but
the polio vaccine protects children for life.
There are 3 strains of WPV- type 1, type 2 and type 3.
y Type 2 was eradicated in 1999.
y No case of type 3 has been found since 2012.
y Since February 2022, Pakistan has reported ten WPV1 cases. Also, countries like Israel, Malawi,
Mozambique reported cases of type 1 WPV in recent months. Hence, statement 2 is not correct.
Source: https://www.pmlive.com/pharma_news/world_health_organization_prequalifies_polio_vaccine_to_protect_chil-
dren_1506336

134.
Answer: D
Context: The Union Cabinet recently cleared a ₹4,797-crore programme called ‘PRITHvi Vigyan’ (PRITHVI) that is expected
to subsume five existing schemes of the Ministry of Earth Sciences (MoES).
Explanation:
Statement 1 is incorrect: It aims to holistically address all the five components of earth system to improve the understand-
ing of the Earth System Sciences and to provide reliable services for the country.

CSE 2024 January 2024 112


DTRS Expalanations
Statement 2 is incorrect: It comes under the Ministry of Earth Sciences (MoES).
Additional information:
The major Objectives of the overarching Prithvi Scheme are:
y Augmentation and sustainance of long-term observations of the atmosphere, ocean, geosphere, cryosphere and
solid earth to record the vital signs of the Earth System and change
y Development of modelling systems for understanding and predicting weather, ocean and climate hazards and un-
derstanding the science of climate change
y Exploration polar and high seas regions of the Earth towards discovery of new phenomena and resources;
y Development of technology for exploration and sustainable harnessing of oceanic resources for societal applica-
tions
y Translation of knowledge and insights from Earth systems science into services for societal, environmental and
economic benefit.
Source: https://www.thehindu.com/news/national/union-cabinet-approves-4797-crore-for-prithvi-scheme/arti-
cle67709390.ece

135.
Answer: D
Explanation:
Context: Lead poisoning, a pervasive yet oft-underestimated public health concern, has silently penetrated the landscapes
of Odisha, leaving a detrimental impact on the well-being of its residents. Lead contamination in Odisha is primarily at-
tributed to improper treatment of waste from industries and e-waste as well as the informal battery recycling sector, which
affects the soil, water and subsequently the food chain.
Statement 1 is correct: Lead accumulates in the body and gets deposited in the bones and teeth.
Statement 2 is correct: Lead poisoning is often asymptomatic early stages, leading to delayed detection.
Statement 3 is correct: There is no level of exposure to lead that is known to be without harmful effects.
Additional information:
y Exposure to lead can affect multiple body systems and is particularly harmful to young children and women of
child-bearing age.
y It poses severe health risks including Cognitive impairments, developmental delays, behavioural disorders in chil-
dren.
y According to the WHO, almost one million people die every year due to lead poisoning across the globe.
y Lead in the body is distributed to the brain, liver, kidney and bones. It is stored in the teeth and bones, where it can
accumulate over time. Human exposure is usually assessed through the measurement of lead in blood.
y Lead in bone is released into blood during pregnancy and becomes a source of exposure to the developing fetus.
y There is no level of exposure to lead that is known to be without harmful effects.
y Lead exposure is preventable.
Source: https://www.thehindubusinessline.com/opinion/lead-poisoning-in-odisha-a-silent-scourge/article67727792.
ece
CSE 2024 January 2024 113
DTRS Expalanations
136.
Answer: B
Explanation:
Context: The International Labour Organization (ILO) has released the World Employment and Social Outlook Trends
2024 report and forecasted a slight increase in the global unemployment rate for 2024, with an additional two million
workers seeking employment.
y This projection is based on various factors influencing the labor market, including low labor force participation,
geopolitical tensions, and potential disruptions in global value chains.
ILO's Projections
y The ILO anticipates a rise in the global unemployment rate from 5.1% in 2023 to 5.2% in 2024.
y The World Employment and Social Outlook Trends 2024 report highlights uneven labor force participation recov-
ery, especially in advanced economies, contributing to persistent labor market imbalances.
Post-Pandemic Labor Market Recovery
y 2023 witnessed significant recovery in labor market participation rates, particularly in higher-income and low-
er-middle-income countries.
y It’s disparities among different labor market groups persisted.
y Advanced economies faced imbalances, and average working hours remained below pre-pandemic levels, impact-
ing overall labor input.
Global Job Gap and Economic Challenges
y The global job gap, although narrowing in 2023, remained high at nearly 435 million.
y Factors such as the erosion of real wages, living standards affected by inflation rates, and rising housing costs con-
tribute to ongoing economic challenges.
y The ILO underscores that compensating for these effects might not be swift.
World Bank›s Cautionary Note
y The World Bank has expressed concerns about a potential slowdown in global economic growth, marking it as the
slowest in three decades.
y Geopolitical tensions add near-term risks, and the ILO report aligns with this sentiment, emphasizing the impact of
rising geopolitical tensions on economic, employment, and social dynamics.
Technology and Labor Market Dynamics
y The introduction and proliferation of artificial intelligence poses additional challenges to labor market adjustments.
y Accelerating technological progress, according to the ILO, may further test the adaptability of the workforce.
y Rising geopolitical tensions, exemplified by conflicts like the Israel–Hamas dispute, pose economic, employment,
and social risks.
y The ILO emphasizes the potential scale of the refugee crisis and its impact on neighboring regions and Europe.
y The report identifies potential disruptions in global value chains as a significant risk.
y The interconnectedness of the global economy implies vulnerability to regional developments and disturbances.
Role of High Interest Rates
y High interest rates in developed economies, specifically highlighted for the United States, could have ripple effects
on global growth.

CSE 2024 January 2024 114


DTRS Expalanations
y The ILO emphasizes the interconnected nature of the global economy, where developments in one region can
impact others.
Source: https://www.downtoearth.org.in/news/economy/ilo-projects-hike-in-global-unemployment-rate-2-million-more-
may-seek-jobs-in-2024-93830

137.
Answer: B
Context: Most of North India experienced a dense fog that reduced visibility in many areas to as little as 50 metres for
several days. The specific type of fog, its duration, and its effects are contingent on various environmental conditions.

Explanation:
Factors responsible for the formation of fog in northern plains of India:
1. Low temperatures
2. Low wind speed
3. Moisture availability
4. Plenty of aerosols
5. Western disturbances
Additional information:
y A fog is a collection of small droplets of water produced when evaporated water has cooled down and condensed.
y According to Madhavan Nair Rajeevan, former director of the IMD and current Vice-Chancellor of Atria University,
“Fog is nothing but a thick cloud, but very close to the earth’s surface. For a thick fog to form, temperatures should
be lower and abundant moisture should be available near the surface.”
y Fog materialises whenever there is a temperature disparity between the ground and the air. This happens fre-
quently during Indian winters fog is created when the temperature drops at night and in the early morning, aero-
sols present in the atmosphere condense.
y High humidity, combined with an ample presence of water vapour or moisture, encourages foggy conditions.
y The process by which it cools plays a pivotal role in the formation of fog. One primary mechanism contributing
to fog formation is called infrared cooling. It typically occurs when the weather is transitioning from summer to
winter.
y In the summer, the ground absorbs radiation from the sun, becomes warmer, and moistens the air passing over it.
When cooler weather kicks in, this mass of warm, moist air comes in contact with processes that cool it.
y The ‘collision’ prompts the water vapour in the air to condense rapidly, giving rise to fog. Another type of fog,
known as radiation fog, is prevalent and occurs when an unseasonably warm day with high humidity is followed by
rapidly dropping temperatures.
Source: https://epaper.thehindu.com/ccidist-ws/th/th_international/issues/66957/OPS/G0EC7MK4G.1+GOTC8P45E.1.html

138.
Answer: C
Explanation:
Context: The new arrangement is to be announced in the upcoming monetary policy for the second half of the current fiscal
year scheduled to be announced on 15 January 2024.

CSE 2024 January 2024 115


DTRS Expalanations
Managed Floating Exchange Rate:
Without any formal international agreement, the world has moved on to what can be best described as a managed floating
exchange rate system. India is having this type of exchange rate system. This system is also known as ‹dirty floating›. It is a
mixture of a flexible exchange rate system (the floating part) and a fixed rate system (the managed part).
In this hybrid exchange rate system, the exchange rate is basically determined in the foreign exchange market through the
operation of market forces. Market forces mean the selling and buying activities by various individuals and institutions. So
far, the managed floating exchange rate system is similar to the flexible exchange rate system. But during extreme fluctu-
ations, the central bank under a managed floating exchange rate system (like the RBI) intervenes in the foreign exchange
market.
The objective of this intervention is to minimize the fluctuation in the exchange rate of the rupee. Since, the exchange rate
is basically determined by market forces, the upward and downward movements in the value of the rupee are appreciation
and depreciation.
Hence, Both statements are correct.
Source:
https://www.tbsnews.net/economy/banking/bb-plans-introduce-managed-floating-exchange-rate-mechanism-773610

139.
Answer: B
Explanation:
Context:
Union finance minister will be presenting her sixth Budget in 2024. The Budget will be presented on February 1, 2024.
y Primary deficit is fiscal deficit of the current year minus interest payments on previous borrowings. While fis-
cal deficit represents the government›s total borrowing including interest payments, primary deficit shows the
amount of borrowing excluding interest payments. Hence option (b) is the correct answer.
y It shows the amount of government borrowings specifically to meet the expenses by removing the interest pay-
ments. Therefore, a zero Primary Deficit means the need for borrowing to meet interest payments.
y Fiscal Deficit: The excess of total expenditure over total receipts excluding borrowings is called fiscal deficit. In oth-
er words, the fiscal deficit gives the amount needed by the government to meet its expenses.
Thus a large fiscal deficit means a large amount of borrowings.
y Revenue Deficit: Revenue deficit is the excess of its total revenue expenditure to its total revenue receipts. Reve-
nue deficit is only related to revenue expenditure and revenue receipts of the government.
y Effective Revenue Deficit: It is the difference between revenue deficit and grants for the creation of capital assets.
Source: https://economictimes.indiatimes.com/news/economy/policy/interim-budget-2024-types-of-deficits-and-how-
they-are-calculated/articleshow/106258664.cms?from=mdr

140.
Answer: D
Context: The Union ministry has created a draft policy to declare Pong Dam eco-sensitive zone.
Explanation:
Pair 1 is incorrectly matched: Pong dam has been constructed on the river Beas in the wetland of Shivalik hills of Kangra
district of Himachal Pradesh. It is known as the Maharana Pratap Sagar.
CSE 2024 January 2024 116
DTRS Expalanations
y It was built in 1975 and is a famous wildlife sanctuary.
y It is one of the Ramsar sites in India.
Pair 2 is incorrectly matched: Matatila dam is built on Betwa river.
y It is entirely a rain fed river with very high discharge of water during rainy season and extremely low discharge
during summers.
y The dam was constructed in 1957 for irrigation purpose.
Pair 3 is incorrectly matched: It is one of the largest dams in South India. It is built on Kaveri river. It was built in 1934 at the
meeting point of Cauvery to the plains irrigating Namakkal, Karur, Thiruchirapalli, Tanjore districts and also parts of Salem
and Erode amounting to over 2 lakh acres of farmland.
Source: https://downtoearth.org.in/news/governance/pong-dam-eco-sensitive-zone-awaiting-rehabilitation-for-over-50-
years-not-consulted-for-policy-decision-say-locals-93814

141.
Answer: B
Context: NISAR, the radar satellite set to be launched this year, will measure some key Earth vital signs.
Explanation:
Statement 1 is incorrect: It is a collaborative programme of Indian Space Research Organisation and NASA.
Statement 2 is correct: Named the NASA-ISRO Synthetic Aperture Radar mission (NISAR). This joint effort aims to capture
crucial Earth data, measuring the health of wetlands, ground deformation caused by volcanoes, and the dynamics of land
and sea ice.
y It is designed to observe nearly all of the planet›s land and ice surfaces twice every 12 days, offering an unprece-
dented frequency of data collection.
y The satellite›s capabilities will be enhanced through the use of two radars: an L-band system with a 10-inch
(25-centimeter) wavelength and an S-band system with a 4-inch (10-centimeter) wavelength.
y NISAR›s orbit orientation will allow it to collect data from Antarctica›s far interior, close to the South Pole.
y The satellite›s measurements will enable scientists to closely examine the complex interactions at the intersection
of ice and ocean.
Source: https://ddnews.gov.in/sci-tech/nisar-isro-nasa-satellite-unravel-earths-frozen-mysteries

142.
Answer: B
Context: In a groundbreaking archaeological revelation, researchers have utilized LiDAR (Light Detection and Ranging) tech-
nology to unveil a hidden network of ancient civilizations in the Amazon rainforest.
Explanation:
Statement 1 is correct: There are three primary components of a LiDAR instrument — the scanner, laser and GPS receiver.
Statement 2 is incorrect.
Types of lidar:
y Topographic lidar: It typically uses a near-infrared laser to map the land.
y Bathymetric lidar: It uses water-penetrating green light to also measure seafloor and riverbed elevations.

CSE 2024 January 2024 117


DTRS Expalanations
Additional information:
LiDAR Technology:
y LiDAR is a remote sensing method that uses light in the form of a pulsed laser to measure ranges (variable distanc-
es) to the Earth.
y These light pulses—combined with other data recorded by the airborne system — generate precise, three-dimen-
sional information about the shape of the Earth and its surface characteristics.
y There are three primary components of a LiDAR instrument — the scanner, laser and GPS receiver.
y Airplanes and helicopters are the most commonly used platforms for acquiring lidar data over broad areas.
LiDAR Technology Unveils Lost History
y Context of Lost Amazonian History:
¾ The Amazon, renowned for its biodiversity, was traditionally perceived as devoid of large ancient civilizations.
¾ However, LiDAR technology has emerged as a game-changer.
¾ It allowed researchers to peer through the thick jungle canopy and uncover the hidden historical treasures
beneath.
y Discovery of Sangay:
¾ Covering 115 square miles with LIDAR, scientists from the French National Center for Scientific Research
identified the ruins of Sangay, an ancient city nestled in the Upano Valley.
¾ This settlement, rivaling the complexity of civilizations in Mexico and Central America, challenges previous
notions of the Amazon›s historical landscape.
Future Exploration and Implications
y Ongoing Exploration:
¾ While the discovery of Sangay provides a glimpse into Amazonian history, much remains unexplored.
¾ With approximately 90% of Amazonian history yet to be uncovered, future expeditions hold the promise of
revealing more about these ancient civilizations.
y Technological Advancements:
¾ The use of advanced technologies like LiDAR demonstrates the potential to unravel hidden histories globally.
¾ As researchers continue to employ innovative methods, our understanding of ancient cultures and their
impact on diverse ecosystems is likely to evolve.
Source: https://www.popularmechanics.com/science/archaeology/a46458939/lost-city-amazon-discovered/

143.
Answer: D
Context: Scientists have been experimenting with the In-Vitro Fertilization method to save the highly threatened White
Rhino.
Explanation:
The name ‘White rhinoceros’ is taken from the Afrikaans word describing its mouth: “wyd”, meaning “wide”. Early English
settlers misinterpreted the “wyd” for “white”. It is also sometimes called the square-lipped rhinoceros.
Statement 1 is incorrect: The white rhino lives in Africa, in long and short-grass savannahs. Just five countries hold 99% of
Africa’s white rhinos – the vast majority of those in South Africa.
CSE 2024 January 2024 118
DTRS Expalanations

Statement 2 is incorrect: White rhinos and Sumatran Rhinos are among the rhino species which have two horns. The larger
front horn measures 37 – 79 inches (94 – 201 cm). The rear horn measures up to 22 inches (55 cm) long.
Note that Indian Rhino and Javan Rhino possess only one horn.
Statement 3 is incorrect: IUCN classifies the White Rhino as ‘Near Threatened’ in its Red List, it must be noted that the
subspecies of white rhino have the following IUCN status:
Northern White Rhino: Critically Endangered
Southern White Rhino: Near Threatened
Additional information:
y White rhinos are grazers. Its wide, square upper lip is adapted for feeding on grasses.
y White rhinos can live to be 35-40 years of age. Gestation lasts approximately 16 months, and mothers give birth to
one calf every 2-3 years.
y White rhinos are semi-social and territorial. Females and subadults generally are social, but bulls are often solitary.
Source: Test tube rhinos: Why rebuilding doomed species is a desperate race against time | Explained News - The Indian
Express; White Rhino | International Rhino FoundationInternational Rhino Foundation (rhinos.org)

144.
Answer: A
Context: Ministry of Environment, Forest and Climate Change has released the first-ever Status Report of Snow Leopards
in India
Explanation:
Statement 1 is correct:
About Snow Leopard Population Assessment in India (SPAI):
y India has ratified and played an important role in the Global Snow Leopard and Ecosystem Protection Program
(GSLEP), a high level inter-governmental alliance of all the 12 snow leopard range countries.
y These Governments have jointly initiated an effort to conduct Population Assessment of the World’s Snow Leop-
ards, or PAWS.

CSE 2024 January 2024 119


DTRS Expalanations
y Recognizing the importance of understanding snow leopard occurrence and status for conservation planning, the
Government of India has launched India’s PAWS effort, referred to here as the ‘Snow Leopard Population Assess-
ment in India (SPAI)’, which is expected to lead to scientifically robust national and state-wise population esti-
mates of this endangered and elusive cat across its high altitude habitat, both inside and outside protected areas.
y The Snow Leopard Population Assessment in India (SPAI) Program, 2023 is the first-ever scientific exercise in
India.
y SPAI exercise was conducted from 2019 to 2023 and assessment has been released now.
Statement 2 is incorrect:
y The Wildlife Institute of India (WII) is the National Coordinator for this exercise that was carried out with the
support of all snow leopard range states and two conservation partners, the Nature Conservation Foundation,
Mysuru and WWF-India.
Statement 3 is incorrect:
y Based on data analysis, the estimated population in different States/UTs are as follows: Ladakh (477), Uttarakhand
(124), Himachal Pradesh (51), Arunachal Pradesh (36), Sikkim (21), and Jammu and Kashmir (9)

Sources:
1. https://pib.gov.in/PressReleasePage.aspx?PRID=2000545
2. https://images.assettype.com/ncfindia/2021-04/20ba641b-992f-4b4c-9368-e8d4d77168da/SPAI_compressed.pdf

CSE 2024 January 2024 120


DTRS Expalanations
145.
Answer: B
Explanation:
Context: Scientists recently discovered a new kangaroo lizard species from Western Ghats.
Statement 1 is correct: It is endemic to Western Ghats, where it is known from Sivagiri Hills (type locality), Agasthyamalai,
Cardamom Hills and Travancore hills in Tamil Nadu and Kerala states.
Statement 2 is incorrect: It is a tiny species of lizards and lives in moist leaf litter on the forest floor of both evergreen and
deciduous forests.
Statement 3 is correct: A reduced fifth toe makes these reptiles poor climbers and hence do not climb trees like other liz-
ards. Instead, they are mostly terrestrial and found in areas with dense leaf litter cover. While they feed on small insects,
this variety of kangaroo lizards run fast and hide within dry leaves to evade predators.

Source: https://www.thehindu.com/sci-tech/energy-and-environment/scientists-discover-new-kangaroo-lizard-spe-
cies-from-western-ghats/article67792494.ece

146.
Answer: B
Context: Japan›s Smart Lander for Investigating Moon (SLIM) spacecraft successfully touched down on the lunar surface on
Friday, making Japan the fifth nation to achieve a soft landing on the Moon. The Japan Aerospace Exploration Agency (JAXA)
has confirmed the landing of the spacecraft equipped with a variety of instruments, including a camera, a laser altimeter,
and a magnetometer, which will be used to study the Moon›s geology, environment, and resources.
Explanation:
Statement 1 is incorrect: It was launched by Japan›s space agency Japan Aerospace Exploration Agency (JAXA) on January
19, 2024, on a Mitsubishi Heavy H2A rocket from Tanegashima Space Center on Tanegashima Island, Japan.
Statement 2 is correct: SLIM is a lightweight spacecraft the size of a passenger vehicle. It has been nicknamed ‹The Moon
Sniper›. It has two instruments, LEV-1 and LEV-2, which are small autonomous testing instruments.
y LEV-1 is tasked with recording SLIM›s landing.

CSE 2024 January 2024 121


DTRS Expalanations
y Whereas LEV-2 is a ball-shaped rover equipped with 2 cameras.
Statement 3 is correct: It aims to land near Xioli Crater, an area covered in volcanic rock, using a «pinpoint landing» tech-
nique providing greater control than a Moon landing.
y "Pinpoint Landing" Testing New Landing Technology That Will Get Moon Missions 'Where We Want to Land'
y Landing there will allow finding clues about the Moon›s origin and analyzing minerals with a special camera.
Source: https://www.thehindu.com/sci-tech/science/japans-slim-spacecraft-lands-on-moon-a-first-for-the-country/arti-
cle67757174.ece

147.
Answer: A
Context: 38 Meitei legislators under the chief of an armed militia took a six-point public oath-taking at the Kangla Fort,
Imphal recently.
Explanation:
y Kangla Fort is one of the most important historic and archaeological site of Manipur located in the heart of the
capital city Imphal.
y This fort was built in 33 AD and spread over 200 plus acres acted as the centre of power of the Meitei kings and
the site of its many rituals and festivals.
y It had served as tradition seat of past Meetei rulers of Manipur till 1891.
y The old Govindajee Temple is the largest Hindu, Vaishnav temple in Imphal city in Manipur.
y It is located next to Kangla Fort, palace of the former rulers of the then Manipur Kingdom.
y Its outer and inner moat and other relics are perfect reflections of the rich art and architectural heritage of Mani-
pur.
Additional information:
y Shivneri Fort was the birthplace of the great Maratha warrior. Although Shivaji never ruled the fort, he did try to
capture it on two occasions without any success.
y Sinhagad Fort is an ancient hill fortress located at around 49 km southwest of the city of Pune, India. Previously
known as Kondhana, the fort had been the site of many battles, most notably the Battle of Sinhagad in 1670. It is
located in the Pune district in Maharashtra.
y Lachit Grah Fort, located in Jorhat, is a historical fort built by the Ahom King, Swargadeo Pratap Singha, in the 17th
century. The fort was strategically constructed to serve as a military base and protection against enemy invasions.
Source: https://indianexpress.com/article/political-pulse/why-meitei-outfit-chose-imphals-kangla-fort-to-summon-legisla-
tors-9127314/
https://www.thehindu.com/opinion/lead/a-dark-message-to-delhi-from-kangla-fort/article67793708.ece

148.
Answer: D
Context: The Reserve Bank of India (RBI) has issued a directive to prohibit Paytm Payments Bank Limited (PPBL) from ac-
cepting deposits or top-ups in any customer account, including wallets and FASTags, effective from February 29 onwards.
Statement 1 is incorrect: The formation of Payment Banks was recommended by the Nachiket Mor committee. It was
formed in September 2013 by the Reserve Bank of India to study ‹Comprehensive financial services for small businesses

CSE 2024 January 2024 122


DTRS Expalanations
and low income households›. The objective of the committee was to propose measures for achieving financial inclusion and
increased access to financial services.
Statement 2 is incorrect: A payments bank is like any other bank, but operating on a smaller scale without involving any
credit risk.
y It can carry out most banking operations but can’t advance loans or issue credit cards. It can accept demand de-
posits.
y It cannot accept time deposits or NRI deposits.
y It can accept demand deposits in the form of savings and current accounts. They can take deposits up to Rs.
2,00,000.
y It can issue debit cards.
Source: https://www.indiatoday.in/business/story/rbi-bars-paytm-payments-bank-from-accepting-deposits-in-custom-
er-accounts-wallets-after-february-29-2495898-2024-01-31

149.
Answer: B
Context: Various scientific studies have highlighted the sensitivity of lichens to environmental shifts, making them valuable
instruments for monitoring air quality and ecological health.
Statement 1 is correct: Lichen are non-destructive, living organisms composed of a fungus and algae living in a symbiotic
relationship.
Statement 2 is incorrect: The algae derive nutrients through photosynthesis, and the fungi protect the algae from drying
out. Lichens can live and grow in extremely barren areas.
y Unlike plants, lichens lack cuticles for protection or roots to draw nutrients from deep within the soil.

Statement is correct: Lichens are bioindicators and biomonitors providing invaluable clues about the quality and quantity
of pollution in the environment.
y Lichens are known for their resilience in the face of pollution, a trait that has positioned them as biomonitors and
bioindicators.
y Bioindicators are organisms that furnish information about the quality of the environmental parameters studied,
while biomonitors provide clues on the quantitative aspects of the environmental quality.
y Different lichen species react differently to different pollutants.
Source: https://india.mongabay.com/2024/01/explainer-what-can-lichens-tell-us-about-atmospheric-pollution/

150.
Answer: B
Context: India has been ranked 93 out of 180 countries in corruption perceptions index 2023.
Explanation:
y Corruption Perceptions Index (CPI), 2023 is released by Transparency International
y It ranks countries and territories by their perceived levels of public sector corruption, on a scale of 0 (highly cor-
rupt) to 100 (very clean).
y In 2023, Denmark ranked first, followed by Finland, New Zealand, and Norway.
CSE 2024 January 2024 123
DTRS Expalanations
y The CPI is based on surveys of domestic and international business executives, financial journalists, and risk ana-
lysts. Therefore, it reflects the perceptions of experts and business elites, not of the general public.
y The CPI, being based on evaluations, is a valuable alternative source of information about the degree of illegal
practices among civil servants and politicians in a given country.
India›s Corruption Perception Index score

y Both Pakistan (133) and Sri Lanka (115) grapple with their respective debt burdens and ensuing political instability.
y But the two countries have strong judicial oversight, which is helping to keep the government in check.
y Although Bangladesh (149) emerges from the least developed country (LDC) status, the flow of information on the
public sector is still hindered amidst an ongoing crackdown against the press.
y China (76) has made headlines with its aggressive anti-corruption crackdown by punishing more than 3.7 million
public officials for corruption over the last decade.
Source: https://www.thehindu.com/news/national/india-ranks-93-out-of-180-countries-in-corruption-perceptions-in-
dex-2023/article67793578.ece#:~:text=Corruption%20Perception%20Index%202023&text=A%20Flourish%20map-,In%20
2023%2C%20India›s%20overall%20score%20was%2039%20while%20in%202022,drawn%20on%20any%20significant%20
change.

151.
Answer: A
Context: There won›t be an Economic Survey this year ahead of the interim budget.
Explanation:
Statement 1 is correct: The Economic Survey is a detailed annual report of the state of the national economy in the financial
year which is coming to a close.
CSE 2024 January 2024 124
DTRS Expalanations
What is the Economic Survey?
y The document is published annually and prepared by the Economic Division of the Ministry of Finance in India.
y It serves as a companion document to the Union Budget, presented a day later.
What does it tell us?
y Detailed Analysis: It goes beyond broad statements and provides an in-depth look at various sectors of the econ-
omy like agriculture, industry, services, infrastructure, and social development.
y Performance Assessment: It analyzes the performance of the economy compared to the previous year, highlight-
ing key trends, growth rates, and changes in various indicators.
y Challenges and Issues: It identifies key challenges faced by the economy, such as unemployment, inflation, or fiscal
deficit.
y Policy Recommendations: It proposes policy recommendations for the government to address the identified chal-
lenges and promote economic growth and development.
Why is it important?
y Independent Perspective: Unlike the Budget, which focuses on government spending plans, the Survey offers an
independent and critical analysis of the economy.
y Data-Driven Insights: It relies on extensive data and research to back its arguments and provide a credible over-
view of the economic situation.
y Future Outlook: It often discusses potential future scenarios and suggests strategies for navigating them.
y Public Discourse: It stimulates public debate and informed discussions on economic issues and policies.
y Statement 2 is incorrect: The Economic Survey›s recommendations are not binding on the Union Budget. The
Economic Survey, despite its detailed analysis and policy suggestions, does not dictate or bind the Union Budget.
Separate Purposes:
y Economic Survey: Analyzes the previous financial year, assesses its performance, and suggests broad, long-term
economic policies.
y Union Budget: Outlines the government›s financial roadmap for the coming year, specifying revenue and expen-
diture allocations.
Different Approaches:
y Survey: Emphasizes economic analysis, data-driven insights, and independent perspectives.
y Budget: Considers political, social, and fiscal realities alongside economic factors. It aligns with the government›s
broader agenda and resource constraints.
y Independent Processes:
y Survey: Prepared by the Economic Division of the Ministry of Finance.
y Budget: Formulated by the Finance Ministry in consultation with various stakeholders.
Benefits of Flexibility:
y Open Dialogue: Creates a space for diverse economic viewpoints and fosters debate on policy options.
y Adaptability to Changing Circumstances: Allows the government to adjust based on current economic situations
and unforeseen events.
y Consideration of Political and Social Realities: Enables balancing economic goals with other priorities like social
welfare and political feasibility.
Source: https://www.cnbctv18.com/economy/explained-heres-why-there-wont-be-an-economic-survey-this-
year-18913041.htm

CSE 2024 January 2024 125


DTRS Expalanations
Additional Information:
y 2024 is an election year in India. Presenting the usual Economic Survey, a document analysing the past year’s per-
formance and future outlook, could get politicised due to the potential change in government after the elections.
This could disrupt the smooth functioning of the regular Budget process, which typically follows the survey pre-
sentation. Finance Minister will present an interim budget, not a full budget. Therefore, there will be no Economic
Survey

152.
Answer: D
Context: The National Institute of Oceanography (NIO) has unveiled a cutting-edge autonomous underwater vehicle (AUV)
named «C-bot» for comprehensive coral reef monitoring.
Explanation:
y It was launched by the director-general of the Council for Scientific and Industrial Research (CSIR) in Goa.
y This technological marvel marks a significant stride in understanding and safeguarding the delicate ecosystems of
coral reefs.
C-Bot›s Capabilities:
y C-bot boasts the ability to reach depths of up to 200 meters, making it a pioneering step in the development of
underwater vehicles capable of scanning the profound depths of the Indian Ocean.
y Director-general of the Council for Scientific and Industrial Research (CSIR) emphasized that this is just the initial
phase, with plans to enhance the AUV›s capabilities for even deeper exploration, potentially reaching thousands
of meters beneath the ocean surface.
y The primary objective of C-bot is to address the critical issue of coral bleaching caused by climate change.
y NIO Director highlighted the AUV›s sophisticated array of sensors and cameras that measure various parameters,
aiming to elucidate the reasons behind coral degradation.
y By providing real-time data, C-bot becomes an invaluable tool for scientists studying the impact of climate change
on coral reefs.
Applications in Navy and Bathymetry Studies:
y Beyond coral reef monitoring, C-bot serves multiple purposes, including assisting the Navy in bathymetry studies.
y The vehicle aids in mapping navigation channels and exploring hydrothermal vents.
y These vents, where geothermally heated water rises from the ocean floor, are crucial to understanding the diverse
ecosystems thriving in extreme conditions.
y C-bot will capture photographs and samples in real-time, shedding light on the active biology occurring in such
extreme environments.
Contributions to Environmental Research:
y C-bot›s launch signifies a leap forward in oceanographic research capabilities.
y Its ability to navigate great depths and capture real-time data opens avenues for a more profound understanding
of ocean ecosystems.
y As climate change poses an increasing threat to marine environments, this underwater vehicle stands as a testa-
ment to scientific innovation in addressing environmental challenges.
Source: https://www.hindustantimes.com/india-news/nio-in-goa-launches-underwater-vehicle-c-bot-to-monitor-coral-
reefs-101706546049939.html

CSE 2024 January 2024 126


DTRS Expalanations

CSE 2024 January 2024 127


DTRS Expalanations
153.
Answer: D
Context: Indigenous Mobile Hospital (BHISHM) Deployed in Ayodhya for the ‹Pran Pratishtha› ceremony.
Explanation:
Project BHISHM (Bharat Health Initiative for Sahyog, Hita and Maitri)
Developed by: Ministry of Health and Family Welfare
About BHISHMA:
y The portable hospital was developed as part of an ambitious ‘Project BHISHMA’ (Bharat Health Initiative for
Sahyog Hita and Maitri) and consists of a fully equipped operation theatre, mini-ICUs, ventilators, blood-testing
equipment, an X-ray machine, and even a cooking station.

y It also includes a shelter facility and power generator, making it entirely self-sufficient during emergencies.
y The life-saving equipments are packed in a set of “mini-cubes” and a combination of these mini-cubes is used to
set up a mobile hospital capable of performing life-saving surgery.
y The mini-cubes are carried in an “Aarogya Maitri Cube Cage”, which has three frames, each accommodating 12
mini-cubes. An Aarogya Maitri Cube Cage can fit 36 mini-cubes overall.
y Two such ‘Aarogya Maitri’ mother cubes combine to form a “brick” to complete the kit which is also water and
corrosion-proof.
y The kit also comes with a unique locking and tamper-proof system which is easy to locate, draw-out and use.
y It can be dismantled in various combinations based on availability of space.
Source:
1. https://pib.gov.in/PressReleasePage.aspx?PRID=1998368
2. https://government.economictimes.indiatimes.com/news/healthcare/project-bhishm-mobile-hospital-integrat-
ed-with-ai-data-analytics-deployed-in-ayodhya/107031817
3. https://www.thehindu.com/news/national/india-all-terrain-portable-disaster-hospital-ready-to-be-shared-with-
the-world/article67392626.ece

CSE 2024 January 2024 128


DTRS Expalanations
154.
Answer: A
Context: The water levels in major dams on the Himachal rivers have declined to half of their capacity, owing to dry winters.
Himachal Pradesh has been reeling under a 3-month long dry spell, thus, aggravating the problem of water scarcity in the
region.
Explanation:
Only pair 1 is correctly matched above.
Bhakra Nangal dam with its reservoir Gobind Sagar, is built on a gorge carved by Satluj river in Una and Bilaspur districts
of Himachal Pradesh. In terms of storage of water, it is the third largest reservoir in India, the first being Indira Sagar dam in
Madhya Pradesh with capacity of 12.22 billion cubic meters and the second being Nagarjunasagar Dam in Telangana.

Pair 2 is incorrectly matched:


Pong dam along with its reservoir Maharana Pratap Sagar is situated on river Beas in the Kangra district of HimachaL
Pradesh. Pong reservoir is the most important fish reservoir in the foothills of the Himalayas in Himachal Pradesh. In this
reservoir, majestic fish are found in excess. An international Ramsar site, the wetlands of Pong Dam Lake are considered a
‘wetlands of international importance’, thanks to the large congregation of the ‘Bar Headed Goose’.

CSE 2024 January 2024 129


DTRS Expalanations
Pair 3 is incorrectly matched:
The Thein or Ranjit Sagar dam is built on the river Ravi in Punjab’s Pathankot district.

Source- Dry winter: Water levels in dams on Himachal rivers decline to half of their capacity; experts warn of thirsty summer
(downtoearth.org.in)

155.
Answer: B
Context: Growing violence and tensions in the Levant region, for instance, the recent drone attack on a US military base in
Jordan and the ongoing tussle between Israel and Palestine.
Explanation:
Statement 1 is incorrect: The Levant denotes a vast geographical region situated in the Eastern Mediterranean. The area
termed as the Levant does not have fixed boundaries and it changes over time. The countries which comprise the Levant
are characterized by similar linguistic, cultural, and religious traits. These countries and regions are Iraq, Syria, Lebanon,
Cyprus, Turkey (Hatay Province), Israel, Jordan, and Palestine. A broader definition of the Levant roughly refers to the
territory from Greece to Egypt.

CSE 2024 January 2024 130


DTRS Expalanations
Statement 2 is correct:The Levant region can be said to constitute the countries lying to the North-West of the Arabian
tectonic plate.
Additional information:
The ancient history of the Levant includes what was occupied by the Akkadian Empire, Amorite kingdom, Hittite Kingdom,
Assyrians, Persians, Greeks, Seleucids, Byzantines, and Arabs. The term «Levant» was first used by the French first appeared
in the English language in the late 15th century to describe the French mandate of Syria. The term Levante was used by
Italians while referring to maritime commerce in East Mediterranean. The Levant eventually came to refer to the Muslim
nations of Egypt and Syria- Palestine. England established the Levant Company to trade with the Ottoman Empire in 1581.
Source- Drone that killed US troops in attack slipped past defenses because there was confusion on whether or not it was
American: reports (msn.com); Where is the Levant? - WorldAtlas

156.
Answer: D
Context: Three military-led West African nations have announced their immediate withdrawal from regional bloc ECOWAS,
accusing the body of becoming a threat to its members.
Explanation:
y In May 1975, 15 West African Countries established the Economic Community of West African States (ECOWAS) by
signing the Lagos treaty.
y Its headquarter is located in Nigeria.
y At the signing of the treaty, its members were: Benin, Burkina Faso, Côte d’Ivoire, The Gambia, Ghana, Guinea,
Guinea Bissau, Liberia, Mali, Mauritania, Niger, Nigeria, Sierra Leone, Sénégal and Togo.
y The aim of the Community is to promote cooperation and integration, leading to the establishment of an economic
union in West Africa in order to raise the living standards of its peoples, and to maintain and enhance economic
stability, foster relations-among Member States and contribute to the progress and development of the African
continent.
y The establishment of a common market is one of the objectives of the bloc. It aims to achieve it through liber-
alization of trade by the abolition of customs duties levied on imports and exports, and the abolition of non-tariff
barriers in order to establish a free trade area at the Community level.
y The bloc is working towards establishing a common currency, known as the Eco. The process has been ongoing for
several years. Hence, it is yet to adopt a common currency.
Source: https://www.aljazeera.com/news/2024/1/28/niger-mali-burkina-faso-announce-withdrawal-from-ecowas

157.
Answer: B
Context: Increasing amounts of black carbon in the atmosphere are leading to extreme rainfall in Asian countries such as
India and China, suggested a policy brief from the Center for Study of Science, Technology and Policy (CSTEP) and Clean Air
Fund.
Explanation:
Statement 1 is correct: Black carbon, commonly known as soot, is a component of fine particulate air pollution (PM2.5). It
is formed by the incomplete combustion of wood and fossil fuels.
CSE 2024 January 2024 131
DTRS Expalanations
Statement 2 is correct: Black carbon is a dual threat as it pollutes the air and warms the atmosphere, exacerbating climate
change. The incomplete combustion also creates carbon dioxide (CO2), carbon monoxide, and volatile organic compounds.
Statement 3 is incorrect: Black carbon lasts only days to weeks in the atmosphere but has significant direct and indirect
impacts on the climate, snow and ice, agriculture, and human health.
y It has a warming impact up to 1,500 times stronger than CO2 per unit of mass.
y Household energy accounts for nearly half of global black carbon emissions globally.
Source: https://india.mongabay.com/2024/01/dual-role-as-air-pollutant-and-warming-agent-makes-black-carbon-a-ne-
glected-player-in-policy/

158.
Answer: D
Context: The Indian government in January 2024 appointed four members of the 16th Finance Commission that will recom-
mend how federal taxes will be shared with states.
Explanation:
On December 31, 2023, the government appointed the Sixteenth Union Finance Commission, with Arvind Panagariya as
its chairman, in accordance with the constitutional mandate outlined in Article 280(1), which serves as the primary link
between the Union and state financial relations.
The main duties of Union Finance Commission are specified in Article 280(3): vertical devolution-sharing of net taxes and
other funds between the Union and states; horizontal devolution-sharing the financial resources among the states; grants-
in-aid under Article 275; devolution of funds to the rural and urban local governments; and any other related fiscal issues.
Source: https://theprint.in/india/indian-government-appoints-members-for-new-finance-panel/1946175/

159.
Answer: D
Explanation:
Context: The 54th annual meeting of the World Economic Forum held in Davos, Switzerland on January 19, 2024.
Energy Transition Index (ETI):
y Published by the World Economic Forum (WEF).
y Tracks the progress of countries towards transitioning to low-carbon energy systems.
y Ranks 115 countries based on their performance in 40 indicators across five dimensions: enabling environment,
infrastructure and innovation, regulatory and institutional framework, human capital, and market stability.
Global Competitiveness Report (GCR):
y Also published by WEF.
y Assesses the competitiveness of economies based on 12 pillars, including institutional environment, infrastructure,
technological innovation, and business sophistication.
y Ranks 141 countries based on their ability to sustain long-term economic growth and prosperity.
Global IT Report:
y Produced by WEF in collaboration with INSEAD and Cornell University.
y Examines the use of information technology to drive economic growth and development.

CSE 2024 January 2024 132


DTRS Expalanations
y Ranks 132 countries based on their IT infrastructure, human capital, and innovation ecosystem.
Global Gender Gap Report (GGGR):
y Another WEF publication.
y Measures the gap between genders in economic participation, educational attainment, health and survival, and
political empowerment.
y Ranks 156 countries based on their progress towards closing the gender gap.
Global Travel and Tourism Report (GTTR):
y Published by WEF in partnership with the World Travel and Tourism Council (WTTC).
y Analyzes the competitiveness of the travel and tourism industry in 117 countries.
y Ranks countries based on their enabling environment, infrastructure, human capital, and market demand.
Hence, All reports are published by WEF, so answers will be (d).
Source: https://indianexpress.com/article/explained/everyday-explainers/davos-meeting-2024-5-key-takeaways-9120670/

160.
Ans. C
Context: Explainer article in the Hindu on the topic Acid Rain.
Explanation:
Statement 1 is correct: Acid rain is rain that is acidic. When fossil fuels that contain sulphur are combusted, their emissions
include sulphur dioxide (SO2). When such combustion happens at a higher temperature, like inside the engine of a car, the
combustion products also include nitrogen oxide and nitrogen dioxide (collectively called NOx). Both SO2 and NOx are also
produced naturally, such as when volcanoes erupt or when lightning passes through the atmosphere, but in and around
cities, their principal source is the use of fossil fuels for transport and power generation. Once SO2 and NOx rise into the air,
they react with water and oxygen molecules to produce sulphuric acid (H2SO4) and nitric acid (HNO3), both of which are
corrosive. When these molecules dissolve in water droplets and the droplets precipitate, we have acid rain, acid snow, and
even acid fog. The typical pH of acid rain is around 4.2-4.4.
Statement 2 is correct: When acid rain flows into rivers and lakes, it can render the water inhospitable to some species; in
soil, it destroys some bacteria. However, it causes the fungi soil microbes to increase tremendously. These effects can in
turn adversely affect forests and other large ecosystems in complex ways.
Statement 3 is incorrect: Dead or dying trees are a common sight in areas affected by acid rain. Acid rain leaches Alumin-
ium from the soil.
Statement 4 is correct: To mitigate these effects, coal power plants have been able to reduce the SO, content in their atmo-
spheric emissions by more than 90% using flue-gas desulphurisation. Many governments around the world have also been
working together to minimize acid rain; an example in Asia is the Acid Deposition Monitoring Network in East Asia (EANET).
Source- PressReader.com : Digital Newspaper & Magazine Subscriptions; Effects of Acid Rain | US EPA

161.
Answer: B
Explanation:
Context: Recently, two new mammalian species - Binturong or the bearcat and the small-clawed otter - have been record-
ed in the Kaziranga National Park and Tiger Reserve.

CSE 2024 January 2024 133


DTRS Expalanations

Statement 1 is incorrect: Binturongs are found in the tropical forests of India, Nepal, Bhutan, Bangladesh, China, Cambodia,
Laos, Malaysia, Thailand, Vietnam, Myanmar, Philippines, and Indonesia. It is omnivorous and feeds on small mammals,
birds, fish, earthworms, insects, and fruits. They also prey on rodents, fish, and earthworms. Figs, however, are a major
component of their diet.
Statement 2 is correct: It is the largest civet in India and listed under Schedule I of the Wildlife Protection Act, 1972.
Statement 3 is incorrect: It is uncommon in much of its range, and has been assessed as Vulnerable on the IUCN Red List
because of a declining population. It is estimated to have declined at least 30% since the mid-1980s.
Additional Information:

Appearance y Long, shaggy black fur with white or gray highlights


y Prehensile tail, almost as long as its body, for swinging through trees
y Large eyes and small, rounded ears
y Sharp claws for climbing and catching prey

Habitat y Rainforests of Southeast Asia, from India to Indonesia


y Spends most of its time in the canopy, rarely coming down to the ground
Behavior y Solitary except during mating season
y Nocturnal, coming out at night to forage
y Moves slowly and deliberately through the trees
y Known for its strong, musky odor, sometimes compared to popcorn or butter

Interesting facts y One of only two carnivores with a prehensile tail (the other is the kinkajou)
y Has a pouch-like fold of skin under its tail, possibly used for storing fat or carrying its young
y Females give birth to 1-3 pups, which they carry on their backs
Source: https://www.deccanherald.com/environment/two-new-mammalian-species-recorded-in-kaziranga-nation-
al-park-2868110

CSE 2024 January 2024 134


DTRS Expalanations
162.
Answer: A
Explanation:
Context: Union Minister launched the National Geoscience Data Repository Portal.
Statement 1 is correct. It is a comprehensive online platform for accessing, sharing, and analyzing geospatial information
across India.
Statement 2 is incorrect. The NGDR initiative will be spearheaded by the Geological Survey of India (GSI) and Bhaskara-
chaya Institute of Space Applications and Geoinformatics (BISAG-N).
It represents a significant leap forward in democratizing critical geoscience data, empowering stakeholders across industries
and academia with unprecedented access to invaluable resources.
Geological Survey of India
y The primary goal of the Geological Survey of India (GSI), founded in 1851, was to locate coal reserves for the rail-
ways.
y As time has gone on, GSI has not only expanded into a national repository for geoscience data needed in a variety
of sectors, but it has also gained recognition as a reputable geoscientific organisation worldwide.
y primary responsibilities include mineral resource evaluation and the creation and upgrading of national geoscien-
tific data.
y The main responsibility of GSI is to provide current, unbiased, and objective geological expertise as well as geosci-
entific information of all types, with an emphasis on the demands of business, society, and policymakers.

Source:
1. https://pib.gov.in/PressReleseDetailm.aspx?PRID=1988603
2. https://indianexpress.com/article/cities/chandigarh/full-blown-late-blight-attack-damages-potato-crop-pun-
jab-90738

163.
Answer: D
Context: Recently, Ola group›s AI firm Krutrim has raised $50 million at a valuation of $1 billion in a funding round led by
Matrix Partners, the company. This makes Krutrim the first artificial intelligence unicorn from India.
Explanation:
y Ola’s Krutrim is the first AI unicorn in India.

CSE 2024 January 2024 135


DTRS Expalanations
y Krutrim is a family of LLMs, including Krutrim Base and Krutrim Pro which will have multimodal, larger knowledge
capabilities, and many other technical advancements for inference.
y Krutrim has its base Large Language Model (LLM) with the largest representation of Indian data used for its train-
ing. Krutrim claims to power generative AI applications for all Indian languages.
y Trained by a team of leading computer scientists based in Bangalore and San Francisco, the model will also power
Krutrim›s conversational AI assistant that claims to understand and speak multiple Indian languages fluently.
y Krutrim announced that it will run an early access programme until January 2024 with a simple sign up page on
the website. Additionally, Krutrim›s open APIs (tool to embed it with apps) will open to all developers by February
2024.
Source: https://www.thehindu.com/business/Economy/olas-krutrim-becomes-indias-first-ai-firm-to-turn-unicorn-after-
50-mn-fundraise/article67780438.ece

164.
Answer: B
Context: Directorate of Revenue Intelligence (DRI) seizes alloy containing 16.67 kg gold and 39.73 kg silver worth over Rs.
10 crore, at FPO Delhi in Operation Black Gold.

Explanation:
y Operation Black Gold has been started by the Directorate of Revenue Intelligence (DRI).
About Directorate of Revenue Intelligence (DRI):
y The Directorate of Revenue Intelligence is the apex anti-smuggling agency of India.
y It works under the Central Board of Indirect Taxes & Customs (CBITC)
y Nodal Ministry: Ministry of Finance, Government of India.
y It came into existence on 4th December 1957.
y Headquarter: The DRI with its Headquarters at New Delhi. It has 12 Zonal Units, 35
regional Units and 15 Sub-regional units, with a working strength of about 800 officials.
y Aim: It is tasked with detecting and curbing smuggling of contraband, including drug
trafficking and illicit international trade in wildlife and environmentally sensitive items,
as well as combating commercial frauds related to international trade and evasion of
Customs duty.

CSE 2024 January 2024 136


DTRS Expalanations
Source:
1. https://pib.gov.in/PressReleasePage.aspx?PRID=2000257
2. https://pib.gov.in/PressReleseDetail.aspx?PRID=1880764
3. https://dri.nic.in/main/aboutus

165.
Answer: D
Explanation:
Context: The Indian Space Research Organisation (ISRO) has marked a significant milestone in space exploration with the
announcement of the INSAT-3DS satellite›s journey to the launch port at the Satish Dhawan Space Centre in Sriharikota.
Statement 1 is correct:
y It is an exclusive meteorological satellite realized by the ISRO.
y The satellite is designed for enhanced meteorological observations and monitoring of land and ocean surfaces for
weather forecasting and disaster warning.
y Its primary objective is to provide continuity of services to the existing in-orbit INSAT-3D and 3DR satellites.
Statement 2 is incorrect:
y The satellite was flagged off to SDSC-SHAR launch port on January 25, 2024 for the launch onboard GSLV F14.
y The satellite is a user-funded project with the Ministry of Earth Science (MoES), configured around ISRO›s well
proven I-2k bus platform with a Lift-Off Mass of 2275 kg.
y Indian Industries have significantly contributed to the making of the Satellite.
Additional information:
INSAT-3DS:
y The Satellite had successfully completed Satellite Assembly, Integration & Testing activities at U R Rao Satellite
Centre, Bengaluru.
y Pre-Shipment Review (PSR) was held with the participation of members from the user community on January 25,
2024.

CSE 2024 January 2024 137


DTRS Expalanations
DESIGN AND ADVANCED PAYLOADS:
y The satellite is designed for enhanced meteorological observations and monitoring of land and ocean surfaces
for weather forecasting and disaster warning, with state-of-the-art payloads viz., 6 channel Imager & 19 channel
Sounder meteorology payloads, communication payloads viz., The Data Relay Transponder (DRT) and Satellite aid-
ed Search and Rescue (SAS&R) transponder.
Source: https://www.thehindu.com/sci-tech/science/insat-3ds-satellite-flagged-off-to-sriharikota/article67785551.ece/
amp/

166.
Answer: C
Context: RBI›s invoice discounting platform (TReDS), meant for easy access to credit for small businesses, has seen in-
creased usage in the past year.
Explanation:
Statement 1 is correct: TReDS is an electronic platform for facilitating the financing / discounting of trade receivables of
Micro, Small and Medium Enterprises (MSMEs) through multiple financiers. These receivables can be due from corporates
and other buyers, including government departments and Public Sector Undertakings (PSUs).
y The RBI had introduced TReDS in 2014.
Statement 2 is correct: In 2018, the government made it mandatory for all companies with a turnover greater than Rs.500
crores to register with TReDS.
y The RBI recently allowed more participants on the TReDS platform to improve access to funds for MSMEs. It per-
mitted all institutions that undertake factoring business to participate as financiers in TReDS.
y The regulator also allowed taking insurance cover for invoicing facilities on the TReDS platform which would aid
financiers to hedge default risks.
Source: https://economictimes.indiatimes.com/tech/technology/rbi-backed-treds-logs-quick-growth-after-slow-takeoff/
articleshow/107209463.cms

167.
Answer: A
Context:Mt. Marapi in Indonesia has erupted twice since the month of December, with the most recent eruption seen on
13th January, 2024. Meanwhile, there have been several volcanic eruptions witnessed in the recent past which have been
in the news.
Pair 1 is correct: Ulawun volcanic mountain is located in Papua New Guinea. It is the most active volcano in the region and
has a history of repeated eruptions since the 1700s. The latest eruption happened in December.

CSE 2024 January 2024 138


DTRS Expalanations
Pair 2 is incorrectly matched: Sangay is an active stratovolcano in central Ecuador in the northern volcanic zone of the
Andes. It has been erupting virtually continuously since at least 1934. The recent eruption has been going on since 2019.

Pair 3 is incorrectly matched: Lewotobi Laki-Laki is a volcanic mountain located in Indonesia. The archipelago sits on the
Pacific Ring of Fire, which is an area of intense volcanic and seismic activity.

Source: https://ddnews.gov.in/international/india-extends-1-million-emergency-relief-papua-new-guinea

168.
Answer: C
Context: The Election Commission has notified elections to 56 Rajya Sabha seats to be held in February 2024.
Explanation:
Statement 1 is correct: Rajya Sabha members serve a six-year term and one-third of the members retire every two years
to ensure continuity. Each member of the Rajya Sabha serves a six-year term. This provides stability and experience within
the house. However, the entire house doesn›t dissolve at the end of six years. Instead, only one-third of the members re-
tire every two years. This system ensures continuity within the Rajya Sabha, as two-thirds of the members remain serving
throughout a six-year cycle.
Initially, members are drawn by lots to decide their respective six-year terms. After every two years, one-third of the mem-
bers complete their terms and retire from the house. New members are then elected or nominated to fill the vacant seats.
Statement 2 is correct: Rajya Sabha members are elected indirectly, by members of state legislative assemblies, through a
proportional representation by means of single transferable vote. There are 12 other members who are nominated by the
President.

CSE 2024 January 2024 139


DTRS Expalanations
What is Proportional Representation?
y Instead of the «first-past-the-post» system used in Lok Sabha elections, the Rajya Sabha uses a proportional repre-
sentation (PR) system. This means seats are allocated to parties based on their share of the vote, ensuring a more
proportionate representation of the political landscape in each state.
Statement 3 is correct: According to the Representation of People Act, 1951, specifically Section 154(2), a member chosen
to fill a casual vacancy in the Rajya Sabha will serve for the remaining term of their predecessor. This means that if a Rajya
Sabha member resigns, passes away, or is disqualified mid-term, their seat doesn›t remain vacant until the next sched-
uled election. Instead, a new member is elected through a by-election to fill the remaining period of their predecessor›s
term.
Source: https://www.thehindu.com/news/national/elections-for-56-rajya-sabha-seats-to-be-held-on-february-27-eci/arti-
cle67788646.ece

169.
Answer: A
Context: Union govt. makes it clear that an Aadhaar card is not proof of citizenship, birth date. New Aadhaar cards now
come with a more prominent disclaimer that Aadhaar is a proof of identity, not citizenship or date of birth.
Explanation:
New Aadhaar cards and PDF versions of the identity document have started including a more explicit disclaimer that they
are “a proof of identity, not of citizenship or date of birth”, signaling to government departments and other organisations
not to use it for those purposes.
Aadhaar card has never been proof of citizenship as foreign nationals are also eligible to obtain one if they have been living
in India for 180 days.
However, different government departments accept it for purposes reserved for citizens or adults. For instance, the Election
Commission of India explicitly accepts Aadhaar as a proof of date of birth for enrolling people to vote. These new clarifica-
tions, prominently printed on the document, may challenge such allowances.
Source: https://www.thehindu.com/news/national/government-makes-citizenship-disclaimer-on-new-aad-
haar-cards-more-prominent/article67780076.ece

170.
Answer: B
Context: On 26th November 1949, the Constituent Assembly of India adopted the Constitution of India, which came into
effect from 26th January 1950. The Indian Constitution is not just the lengthiest written Constitution in the world but also
the most richly illustrated, with all 22 parts carrying hand-painted images and its pages adorned with elaborate borders.
Explanation:
Statement 1 is incorrect: Constitution was handwritten by calligrapher Prem Behari Narain Raizada, the paint-
ings were conceived and implemented in Santiniketan by artist-pedagogue Nandalal Bose and his team.
Statement 2 is correct: The only female figure illustrated prominently in the Constitution, Rani Lakshmibai of Jhansi, is
sketched in her armour as she shares the page with Tipu Sultan, the king of Mysore, in Part XVI of the Constitution.
Additional information:
y The Preamble page has intricate patterns sketched by Beohar Rammanohar Sinha and bears his signature, while
Dinanath Bhargava sketched the National Emblem, the Lion Capital of Ashoka.
y The Bull Seal excavated from the Indus Valley region, is the first pictorial representation in the Constitution appear-
ing in ‘Part I: The Union and its Territory’.
CSE 2024 January 2024 140
DTRS Expalanations
y ‘Part II: Citizenship’ features a hermitage scene with male ascetic figures offering prayers in a meditative environ-
ment. In another scene of hermitage that appears in Part V, Buddha is the central figure, surrounded by disciples,
animals, and birds in a serene setting.
y Out of the select representations in colour is an image in Part VI of Mahavir, the 24th Jain Tirthankara, seated
crossed-legged in meditation.
y In Part XIII, we see sculptures from Mahabalipuram and the descent of Ganga to Earth.
y Part IV on Directive Principles of State Policy begins with a scene from the Mahabharata, with the discussion be-
tween Arjun and Krishna before the onset of the war. For Part III on Fundamental Rights, the artists turned to the
Ramayana, drawing a sketch of Ram, Lakshman and Sita returning home after the battle in Lanka.
y While Emperor Ashoka is seen seated on an elephant, propagating Buddhism, in Part VII of the Constitution, Part
IX has a scene from King Vikramaditya’s court with musicians and dancers, representing him as a patron of art.
y Chhatrapati Shivaji and Guru Gobind Singh are featured in Part XV.
y “Portraits of Rana Pratap and Ranjit Singh were also meant to be there but were not included probably due to
space constraints,” Siva Kumar said.
y Gandhi appears twice, leading the Dandi March and visiting riot-hit Noakhali in Bangladesh. He is being welcomed
by women with an aarti thali and Muslim peasants wearing kufi caps.
y In Part XIX, Subhas Chandra Bose is seen against a mountainous backdrop, saluting the flag, with members of Azad
Hind Fauj marching ahead.
Source: https://indianexpress.com/article/explained/explained-culture/illustrations-constitution-who-painted-9127795/

171.
Answer: D
Explanation:
Context: Prime Minister recently launched the ‹Pradhan Mantri Suryodaya Yojana›, a pivotal government initiative aimed at
installing rooftop solar power systems in one crore households.
Statement 1 is correct. It comes under the ambit of the Ministry of New & Renewable Energy (MNRE).
Statement 2 is correct.
Pradhanmantri Suryoday Yojana Eligibility:
y Applicants are required to be permanent residents of India.
y The annual income of applicants should not surpass Rs 1 or 1.5 lakh.
y All necessary documents must be submitted accurately.
y Applicants should not be engaged in government service.
Benefits:
y Approximately one crore households will receive solar rooftop systems installed on their residences.
y The scheme specifically targets assisting those who are economically disadvantaged, including citizens below the
poverty line, with their electricity expenses and other issues related to lighting.
y Participants in the scheme will have solar-powered lights installed in their homes, ensuring a continuous 24-hour
supply of electricity.
y It is envisioned as a lasting solution, offering citizens a permanent and reliable source of electricity.
Additional information:
Pradhanmantri Suryodaya Yojana (PSY):
y This initiative comes in the backdrop of the existing Rooftop Solar Programme.

CSE 2024 January 2024 141


DTRS Expalanations
¾ It was launched in 2014 with the ambitious goal of achieving 40 GW of cumulative installed capacity by 2022.
¾ It was later extended to 2026.
y The primary objective of this scheme is to install solar power systems on residential rooftops, benefiting approxi-
mately one crore households.
y This initiative not only aims to reduce electricity bills for the poor and middle class but also aligns with India›s
broader goal of achieving self-reliance in the energy sector.
Source: https://indianexpress.com/article/explained/modi-launches-new-rooftop-solar-scheme-pradhan-mantri-suryo-
daya-yojana-9122514/

172.
Answer: C
Explanation:
Context: Recently, the possibility of life originating in a “warm little pond” with a suitable mix of chemicals and energy. New
research conducted by the University of Washington and published in Communications Earth & Environment highlights a
shallow “soda lake” in western Canada as a potential match for these conditions. The findings provide new support that life
could have emerged from lakes on the early Earth, roughly 4 billion years ago.
Statement 1 is correct: Soda lakes having high levels of dissolved sodium and carbonate, similar to dissolved baking soda.
Statement 2 is correct: This occurs from the reactions between water and volcanic rocks beneath.
Statement 3 is correct: Soda lakes can have high levels of dissolved phosphate.
Source: https://scitechdaily.com/soda-lakes-the-missing-link-in-the-origin-of-life/

173.
Answer: C
Context: Lake Victoria, the largest freshwater lake in Africa and world’s second-largest, is facing numerous environmental
challenges that demand collective efforts for restoration and conservation.
Explanation: The lake is shared by Kenya (6% by area), Uganda (43%) and Tanzania (51%).
y Among the freshwater lakes of the world, it is exceeded in size only by Lake Superior in North America.
y Lake Victoria has more than 200 species of fish, of which the Tilapia is the most economically important.
y The White Nile begins at Lake Victoria in Tanzania and flows north until it reaches Khartoum.

Source: https://www.downtoearth.org.in/news/africa/restoring-lake-victoria-cse-tanzanian-authorities-hold-multi-na-
tion-stakeholder-consultation-94060
CSE 2024 January 2024 142
DTRS Expalanations
174.
Answer: B
Context: The Central Pollution Control Board (CPCB) recently proposed a new draft of classification criteria for industries
into the red, orange, green and white categories.
Explanation:
Statement 1 is correct: The Central Pollution Control Board, in the year 2016, developed methodology for classifying indus-
trial sectors and other polluting activities as Red, Orange, Green and White, primarily to facilitate uniformity and objectivity
in streamlining enforcement mechanism. The scoring methodology is based on the Pollution Index (PI), which is a function
of water pollution, air pollution, hazardous waste generation, fuel consumption and amount of waste water generation.
Statement 2 is correct: White category includes “practically non-polluting industries”. State Pollution Control Boards were
also empowered to categorize any new/left-out sector at their own level, following the methodology prescribed by the
CPCB.
Statement 3 is incorrect: Semi-conductor fabs manufacturing is classified under red category.

Source:
1. https://www.downtoearth.org.in/blog/pollution/placing-stone-crushers-in-green-category-will-create-a-huge-
environmental-mess-93605
2. https://cpcb.nic.in/openpdffile.php?id=TGF0ZXN0RmlsZS8zODdfMTY5Mzk5MTA1Nl9tZWRpYXBob3RvMjI0Nz-
cucGRm

175.
Answer: B
Context: On the occasion of the Republic Day in New Delhi, a tableau by CSIR was showcased in the annual parade at
Kartavya Path. The visually enchanting tableau of CSIR was based on the famed Purple Revolution ushered through
scientific interventions of CSIR for bringing in the phenomenal increase in Lavender cultivation in several regions of
Jammu & Kashmir.

CSE 2024 January 2024 143


DTRS Expalanations
Explanation: Option 1 and 3 are correct
Statement 1 is correct:
y The Purple Revolution is related to the cultivation of lavender.

About Lavender:
y Lavandula (common name lavender) is a genus of 47 known species of flowering plants in the mint family, Lami-
aceae.
y The flowers may be blue, violet or lilac in the wild species, occasionally blackish purple or yellowish.
y Lavender has been used over centuries in traditional medicine and cosmetics.
y These flowers grow in temperate areas and are drought-resistant crops.
y A single Lavender plant bears flowers for 15 years, needs low maintenance and can be used from the second year
of plantation.
Statement 2 is incorrect:
AROMA Mission: By Council of Scientific & Industrial Research (CSIR)
y CSIR-AROMA Mission, under the Ministry of Science & Technology aims to develop and disseminate the aro-
ma-related science and technology to reach the end user/clients of CSIR: Farmers, industry and society.
y CSIR developed an elite variety of lavender suitable for cultivation in the temperate regions of J&K and provided
free quality planting materials and end-to-end agro-technologies to farmers and also installed distillation units for
essential oil extraction.
Statement 3 is correct:
Bhaderwah (Doda district, Jammu and Kashmir):
y Bhaderwah, which is also known as ‘Chhota Kashmir’, has a mild cold climate during summers that is ideal for
Lavender.
CSE 2024 January 2024 144
DTRS Expalanations
y Bhaderwah is the birthplace of India›s Purple Revolution.
y India’s first National Institute of High Altitude Medicine (NIHAM) is also being built in Bhaderwah.
y Bhaderwah (also Bhadarwah Valley) is a town, tehsil and sub-division in the district Doda of Jammu.
Source:
1. https://pib.gov.in/PressReleasePage.aspx?PRID=1999988
2. https://pib.gov.in/PressReleasePage.aspx?PRID=1799889
3. https://pib.gov.in/PressReleseDetail.aspx?PRID=1875493

176.
Answer: B
Context: A new plant species has been discovered in Theni district of Tamil Nadu in the Western Ghats by a research student
from Wanaparthy. This plant, named ‹Andrographis theniensis› after the place (Theni),
Explanation:
It is a plant which was named ‹Andrographis theniensis› after the place (Theni). This plant resembles Andrographis me-
gamaliana.
y It has shiny leaves and stems, a nine-veined lower lip, pale yellow sparse hairs on the middle lobe of the lower lip,
and yellow anthers.
y Andrographis is a tropical Asian species native to Bangladesh, India, Myanmar, Nepal, Sri Lanka and the western
Himalayas. However, most Andrographis taxa are distributed in southern India and Sri Lanka, especially in the
Western and Eastern Ghats.
Hence option b is the correct answer.
Source: https://timesofindia.indiatimes.com/city/hyderabad/new-plant-species-discovered-in-western-ghats-by-t-re-
searcher/articleshow/107022167.cms

177.
Answer: C
Context: The Union Cabinet approves a ₹8,500 crore viability gap funding scheme for coal gasification projects. The fund-
ing will be given in 3 categories, including grants for government-owned projects and bids for public and private companies.
The scheme aims to produce synthetic natural gas, dimethyl ether, and ammonium nitrate from coal.
Explanation: Options 1, 2, 3 and 5 are correct.
About 80% coal is used in thermal power plants. With environment concerns and development of renewable energy, di-
versification of coal for its sustainable use is inevitable. Coal gasification is considered as cleaner option as compared to
burning of coal and has diversified use of coal in other form of energy.
In the coal gasification process, coal is partially oxidised by air, oxygen, steam or carbon dioxide under controlled condi-
tions to produce chemical constituents.
The desired resultant products can be synthetic natural gas, dimethyl ether, ammonium nitrate and methanol, among
others.
Synthetic natural gas can be used as a substitute for LPG and for electricity generation, while dimethyl ether is an alterna-
tive to be used in diesel engines. Ammonium nitrate can be used for explosives.

CSE 2024 January 2024 145


DTRS Expalanations
Syngas (comprising of Carbon monoxide and Hydrogen gas with smaller amounts of CO2 and CH4) can be used to produce
Gaseous Fuels such as Hydrogen, Substitute Natural Gas (SNG or Methane), Di-Methyl Ether (DME), Liquid Fuels such as
Methanol, Ethanol, Synthetic diesel and Chemical and Petrochemicals like Methanol derivatives, Olefins, Propylene, Mo-
no-Ethylene Glycol (MEG), nitrogenous fertilizers including Ammonia, Industrial Chemicals along with Power Generation.

Source: Cabinet okays Rs 8500 crore for coal gasification - The Economic Times (indiatimes.com); ncgm21-09-21.pdf (coal.
gov.in)

178.
Answer: C
Context: Contentions regarding India’s rice exports due to subsidies like MSP and the upcoming Ministerial of Conference of
the WTO (to be held at Abu Dhabi). Finding a permanent solution to issues of public stockholding would be one of the core
agendas for discussion and negotiation at this meeting. In this context, from the UPSC perspective, it becomes important to
know about the Agreement on Agriculture.
Explanation:
Statement 1 is correct: The Agreement on Agriculture (AoA) is a World Trade Organisation treaty that focuses on reducing
the agricultural support and subsidies given to domestic producers by countries. was negotiated during the Uruguay Round
of the General Agreement on Tariffs and Trade (GATT) and formally ratified in 1994 at Marrakesh, Morocco. The AoA came
into effect in 1995.
y According to its provisions, developing countries were to complete their reduction commitments by 2000 and
developing countries by 2004.
y The Least Developed Countries were not required to make any reductions.
y The Agreement covers products that are normally considered part of agriculture but excludes forestry and fishery
products and also rubber, sisal, jute, coir and abaca.
y The Agreement on Agriculture consists of three pillars — domestic support, market access, and export subsidies.
Statement 2 is correct: The focus of the AoA is the elimination of amber box subsidies which are considered “trade distort-
ing” agricultural subsidies. The AoA classifies the agricultural subsidies into the following types:
CSE 2024 January 2024 146
DTRS Expalanations

Statement 3 is correct: The Agreement on Agriculture (AoA) restricts the developing countries to provide food security
subsidies for programmes such as the MSP beyond 10% of production value. This limit is 5% of the agricultural production
for the developed countries.
Source- India could promise it won’t export from public stocks in return for permanent solution at WTO: GTRI - The Hindu
BusinessLine

179.
Answer: B
Context: 60 Parachute Field Hospital, Uttar Pradesh selected in the Institutional Category for Subhash Chandra Bose Aapda
Prabandhan Puraskar-2024.
Explanation: Statements 2 and 3 are correct
About Subhas Chandra Bose Aapda Prabandhan Puraskar:
y The Government of India instituted Subhash Chandra Bose Aapda Prabandhan Puraskaar to recognise the excel-
lent work done by the individuals and institutions in the field of disaster management.
Statement 1 is incorrect:
Eligibility:
y Only Indian nationals and Indian institutions are eligible to apply for the award.
y For institutions, voluntary organisations, corporate entities, academic/research institutions, response/ uniformed
forces or any other institution may apply as an institution for the award.
y The Individual/Institution for the award must have worked in the area of Disaster Management like Prevention,
Mitigation, Preparedness, Rescue, Response, Relief, Rehabilitation, Research/ Innovations or Early Warning related
work in India.
y The application must be accompanied by details of the work done in disaster management.
Statement 2 is correct:
Decoration of the Award:
y In case the winner is an institution, it shall receive a certificate and a cash prize of Rs.51 lakhs. This cash prize shall
be utilized by the winning institution for Disaster Management related activities only.
y In case of the winner being an individual, the winner shall receive a certificate and a cash prize of Rs. 5.00 lakhs.

CSE 2024 January 2024 147


DTRS Expalanations
Statement 3 is correct:
Who can nominate?
y Any individual and institution can nominate an individual or institution for the award. The application for the same
can be filed between 1st July to 31st August each year.
Ceremony details:
y The award shalle be announced on 23rd January each year, on the birth anniversary of Netaji Subhash Chandra
Bose which is now celebrated as the Parakram Divas.
Source:
1. https://pib.gov.in/PressReleasePage.aspx?PRID=1998709
2. https://ndma.gov.in/subhas-chandra-bose-aapda-prabandhan-puraskar

180.
Answer: B
Context: The recent satellite pictures showed that the Antarctic iceberg called A23a was moving beyond the northern tip
of the Antarctic Peninsula.
Explanation:
y The A23a refers to the name of the iceberg.
y The tooth-shaped iceberg named A23a is nearly 4,000 sq. km across, making it more than twice the size of Greater
London.
y After three decades stuck to the Antarctic ocean floor, A23a is now heading north on what could be its final journey.
y It contains an estimated one trillion tonnes of freshwater that is likely to melt off into the ocean along the way.
y The iceberg, which is up to 400 metres thick in places, is currently drifting between Elephant Island and the South
Orkney islands.
y A23a first broke off the Antarctic coast in 1986, making it the world’s oldest iceberg as well as its largest. But it
quickly became stuck to the ocean floor, where it languished for decades.
y Then in 2023, A23a broke free from its icy shackles and started venturing north.
Source: https://m.economictimes.com/news/international/world-news/worlds-biggest-iceberg-battered-by-waves-as-it-
heads-north/articleshow/107007105.cms

181.
Answer: B
Context: Assam›s Batadrava shrine denies entry to Rahul Gandhi.
Explanation:
Statement 1 is incorrect:
Sankardeva›s philosophy:
y Philosophy propounded by Srimanta Sankardeva is the concept of ‹Ek Saran Naam Dharma›.
y The Ek Saran Naam Dharma focussed on worship in the form of bhakti (devotion) to Lord Krishna, through singing
and congregational listening of His name and deeds.
y Sankardeva espoused a society based on equality and fraternity, free from caste differences, orthodox Brahmanical
rituals and sacrifices. His teaching focused on prayer and chanting (naam) instead of idol worship.

CSE 2024 January 2024 148


DTRS Expalanations
y His dharma was based on the four components of deva (god), naam (prayers), bhakats (devotees), and guru
(teacher).
Achintya Bheda Abheda:
y Chaitanya Mahaprabhu (established Goudiya Vaishnava) propounded his philosophy based on the concept of
Achintya Bheda Abheda, which translates to «inconceivable oneness and difference» in the context of the soul›s
relationship with Krishna, and also Krishna›s relationship with his other energies.
Statement 2 is correct:
Batadrava Than:
y Located in Nagaon district, the Batadrava Than, or Bordowa Than, is one of the most sacred sites for Assamese
Vaishnavites.
y The Than is situated at the birthplace of revered Vaishnavite reformer-saint Srimanta Sankardeva (1449-1568).
y Sankardeva founded the first-ever Kirtan Ghar at Bordowa in 1494 AD to practise and preach the neo Vaishnavite
faith during the fifteenth century in Assam, and propagated the Ek Saran Naam Dharma.”
Source: https://indianexpress.com/article/explained/explained-politics/rahul-gandhi-prevented-batadrava-than-signifi-
cance-9122417/

182.
Answer: B
Context: In a significant political development, the Indian government has announced the posthumous conferment of
the Bharat Ratna, the highest civilian award, on the late Karpoori Thakur, a prominent socialist icon and former Chief
Minister of Bihar.
Explanation:
Statement 1 is incorrect:
y Bharat Ratna is not a title under Article 18.
y Clause 1 of article 18 covers exceptions like academic or military titles. Awards like Padma Vibhushan, Padma
Shri etc. can be given to the citizens for exceptional and distinguished services rendered in art, literature, science
and other fields.
y In the case of Balaji Raghavan v. Union of India, AIR 1996, the court said that National awards aren’t titles un-
der clause 1 of Article 18. The court asked for setting up guidelines while giving these awards as they are highly
respected.
Statement 2 is correct: It is India’s highest civilian award which was instituted in 1954.
Statement 3 is correct. Recently, the Government has decided to confer Bharat Ratna to former Bihar CM Karpoori Thakur,
posthumously.
y Government officials underscore that the Bharat Ratna serves as a tribute to Thakur›s lifelong dedication to uplift-
ing the underprivileged.
y Affectionately known as ‹Jan Nayak› (People’s Leader), his simplicity in personal conduct and monumental contri-
butions to Indian politics make him a fitting recipient of this prestigious award.
y By honoring Thakur, the government not only acknowledges him as a symbol of democracy and social justice but
also recognizes his deep impact as a motivating figure for marginalized sections.
y The award aligns with the values of the Indian Constitution – equality, fraternity, and justice for all.
y Thakur›s life and work, embodying these principles, serve as an inspiration for future generations.

CSE 2024 January 2024 149


DTRS Expalanations
Additional information:
y It is awarded in recognition of exceptional service/performance of highest order in any field of human endeavour.
y It is recommended by the Prime Minister to the President of India.
y The number of Bharat Ratna Awards is restricted to a maximum of three in a particular year.
y Recipient receives a Sanad (certificate) signed by the President and a medallion. Does not carry any monetary
grant.
Source: https://www.thehindubusinessline.com/news/late-karpoori-thakur-to-be-bestowed-bharat-ratna/arti-
cle67769735.ece

183.
Answer: B
Context: India recently submitted its Third National Communication (TNC) and Initial Adaptation Communication to the
United Nation’s Framework Convention on Climate Change (UNFCCC). The National Communication is a report prepared by
Parties in line with articles 4 and 12 of the convention.
Statement 1 is incorrect: Emissions intensity is the total GHG emission per unit of a country’s Gross Domestic Product
(GDP).
As per the current inventory, the emission intensity has reduced by 33% between 2005 and 2019, indicating decoupling
of India’s GDP from GHG emissions, and achievement of India’s previous emissions intensity target, states the submission.
Statement 2 is correct: The energy sector has remained the highest contributor in India for over two decades, accounting
for three fourths of total GHG emissions in 2019.
The National Communication includes:
y An inventory of anthropogenic greenhouse gas (GHG) emissions of the country by sources and removals by sinks
y Materials relevant for calculation of global emissions trends
y Any information pertaining to the steps taken by a country for implementing the convention
Other highlights:
y Carbon dioxide continues to remain the most emitted GHG, comprising 79 per cent of total emissions in 2019.
y All sectors have shown growth in emissions between the two inventories of 2016 and 2019.
y Agricultural activities primarily contributing to GHG emissions are animal husbandry and crop production.

Source: https://www.downtoearth.org.in/blog/pollution/5-rise-in-india-s-ghg-emissions-since-2016-driven-by-energy-in-
dustrial-sectors-94076
CSE 2024 January 2024 150
DTRS Expalanations

CSE 2024 January 2024 151


DTRS Expalanations
184.
Answer: D
Context: Four Indians drowned off an unpatrolled beach at Phillip Island in Australia›s Victoria.
Explanation:
Statement 1 is incorrect: Phillip Island is an island on the south coast of Victoria in Australia.
y It is the site of a koala bear sanctuary; seal, muttonbird, and little blue (fairy) penguin rookeries.

Statement 2 is incorrect: Reunion is an island of the Mascarene Islands that is a French overseas department and overseas
region.
y It is of volcanic origin and consists mostly of rugged mountains in an advanced state of dissection by short torren-
tial rivers.
y Recently, a tropical cyclone, Belal, hit the French island of Reunion in the Indian Ocean.

Statement 3 is also incorrect: Maafushi is one of the most popular and biggest islands in the Maldives. It is located nearly
30 km away from Male in Kaafu Atoll.
y The Maldives is a popular tourist destination, which was caught in a diplomatic row with India recently.
y The Maldives is a chain of 1,190 small islands stretching across the equator to the south west of Sri Lanka.
Source: https://www.hindustantimes.com/world-news/4-indians-drown-at-unpatrolled-beach-in-australias-victo-
ria-101706148274679.html

CSE 2024 January 2024 152


DTRS Expalanations
185.
Answer: D
Explanation:
Context: Recently, Sweden’s attempt to join NATO cleared a major hurdle after Turkey’s parliament supported its member-
ship. For a new country to join the North Atlantic Treaty Organization (NATO), all the existing members have to approve it.
Turkey and Hungary had been opposing Sweden’s entry for almost the past two years.
y NATO stands for North Atlantic Treaty Organization and is made up of 28 member countries in North America and
Europe.
y NATO aims to safeguard its members through political and military means. NATO›s headquarters are in Brussels,
Belgium.
y The genesis of NATO can be traced backed to cold war era. In 1949, the prospect of further Communist expansion
prompted the United States and 11 other Western nations to form the North Atlantic Treaty Organization (NATO).
The Soviet Union and its affiliated Communist nations in Eastern Europe founded a rival alliance, the Warsaw Pact,
in 1955.
Statement 1 is correct: It is an intergovernmental military alliance based on the North Atlantic Treaty.
Statement 2 is correct: It works on the principle of collective defence i.e. an attack against one ally is considered as an attack
against all allies.
Statement 3 is correct: Turkey is a member coutry of North Atlantic Treaty Organization (NATO). Recently, it was supported
by its allies against its war on terror.
The NATO countries are: Albania, Belgium, Bulgaria, Canada, Croatia, Czech Republic, Denmark, Estonia, France, Germany,
Greece, Hungary, Iceland, Italy, Latvia, Lithuania, Luxembourg, Netherland, Norway, Poland, Portugal, Romania, Slovakia,
Slovenia, Spain, Turkey, United Kingdom, United States.
Source: https://indianexpress.com/article/explained/everyday-explainers/turkey-swedens-nato-why-it-changed-its-
stance-9126127/

186.
Answer: A
Context: The premature release of the 11 convicts by the Gujarat government in the Bilkis Bano case raised certain serious
legal and moral questions. However, The Supreme Court in January 2024 set aside the remission of 11 convicts sentenced
to life imprisonment.
Explanation:
Statement 1 is incorrect: Article 72 and 161 of the Constitution provide powers to the President and Governor respectively
to grant pardon, commutation, remission, respite or reprieve to a convict.
The powers vested in the heads of the Union and State executive are to be exercised on the advice of the council of minis-
ters.
Statement 2 is correct: Apart from this, the appropriate State government under Section 432 of the Criminal Procedure
Code, 1973 (CrPC) may remit the whole or part of the punishment to which a convict has been sentenced. In case of life
imprisonment convicts, this remission can be done only after a period of 14 years in jail as per Section 433A of the CrPC.
Statement 3 is incorrect: The appropriate State government for considering the remission application is the one where the
offense was committed or jail term was being served. Also, the law requires the opinion of the presiding judge of the con-
victing court to be obtained before considering the remission petition.

CSE 2024 January 2024 153


DTRS Expalanations
The Supreme Court in Laxman Naskar versus Union of India (2000) had laid down five grounds on which remission is to be
considered. The first of these is whether the offense is an individual act of crime that does not affect society. It would be
preposterous to surmise that a heinous crime does not impact the conscience of a civilized society.
The Supreme Court in Sangeet versus State of Haryana (2012) had held that a convict serving life imprisonment does not
have a right to be prematurely released on completion of 14 years in jail and that remission should be considered only on
a case-by-case basis.
Source: https://www.thehindu.com/news/national/the-laws-around-remission-policy-explained/article67728274.ece
Additional Information:
y Article 72 empowers the President the power to grant pardons, reprieves, respites or remissions of punishment or
to suspend, remit or commute the sentence of any person convicted of any offence. The meaning of these terms
is as follows:
y Pardon: It removes both the sentence and the conviction and completely absolves the convict from all sentences,
punishments, and disqualifications.
y Commutation: It denotes the substitution of one form of punishment with a lighter form of punishment. For ex-
ample, a death sentence may be commuted to rigorous imprisonment.
y Remission: It implies reducing the period of the sentence without changing its character. For example, a sentence
of rigorous imprisonment for five years may be remitted to rigorous imprisonment for one year.
y Respite: It denotes awarding a lesser sentence in place of one originally awarded due to some special fact, such
as the physical disability of a convict or the pregnancy of a woman offender.
y Reprieve: It implies a stay of the execution of a sentence (especially that of death) for a temporary period. Its
purpose is to enable the convict to have time to seek pardon or commutation from the President.
Article 161 provides that the Governor of a State shall have the power to grant pardons, reprieves, respites or remissions of
punishment or to suspend, remit or commute the sentence of any person convicted of any offense against any law relating
to a matter to which the executive power of the State extends.
Difference between Pardoning Powers of President and Governor:
The scope of the pardoning power of the President under Article 72 is wider than the pardoning power of the Governor
under Article 161 which differs in the following two ways:
1. The power of the President to grant pardon extends in cases where the punishment or sentence is by a Court
Martial but Article 161 does not provide any such power to the Governor.
2. The President can grant pardon in all cases where the sentence given is the sentence of death but the pardoning
power of the Governor does not extend to death sentence cases.

187.
Answer: B
Explanation:
Context: India's nascent space sector has sought a liberal FDI policy on par with the defence industry and production-linked
incentive in the interim Budget 2024.
y The role and functions of Department for Promotion of Industry and Internal Trade (DPIIT) includes Formulation of
Foreign Direct Investment (FDI) Policy and promotion, approval and facilitation of FDI.
y The Foreign Investment Promotion Board (FIPB), housed in the Department of Economic Affairs, Ministry of Fi-
nance, is an inter-ministerial body, responsible for processing of FDI proposals and making recommendations for
Government approval.

CSE 2024 January 2024 154


DTRS Expalanations
y In the process of making recommendations, the FIPB provides significant inputs for FDI policy-making. But it
doesn›t formulate the policy.
Therefore, while DPIIT takes the lead in formulating the overarching FDI policy framework, FIPB acts as a collaborative and
advisory body providing valuable inputs and recommendations on specific investment proposals. This two-pronged ap-
proach ensures both comprehensive policy development and efficient evaluation of individual investments.
y The Minister of Finance who is in-charge of FIPB would consider the recommendations of FIPB on proposals with
total foreign equity inflow of and below Rs. 3000 crore. The recommendations of FIPB on proposals with total
foreign equity inflow of more than Rs. 3000 crore would be placed for consideration of Cabinet Committee on
Economic Affairs (CCEA).
Hence, Option (b) is the correct answer.
Source: https://www.livemint.com/science/news/interim-budget-2024-space-industry-seeks-liberal-fdi-policy-pli-
scheme-11706092973011.html

188.
Answer: C
Explanation:
Context: Remembering father of India›s nuclear programme on his death anniversary.
y Homi Jehangir Bhabha is known as the father of India›s nuclear programme.
y He laid the foundation of India›s nuclear power and his visions also made the country a responsible nuclear power
country.
y Homi Jehangir Bhabha, died on 24 January 1966 in a mysterious plane crash.
He was a physicist and later became a very prominent atomic expert in the world.
y Bhabha was born in a wealthy family in Mumbai in 1927. He went to England at Cambridge University to study
engineering.
y His father was a lawyer, who wanted him to pursue mechanical engineering at Cambridge University so he could
join Tata Steel Mills in Jamshedpur. But Mr. Bhabha chose physics for his career.
y Before World War II, Mr. Bhabha returned to India and joined the Indian Institute of Science under Nobel laureate
CV Raman.
y In 1944, He started the Cosmic Ray Research Unit in Indian Institute of Science and started researching in the nu-
clear field.
y With the help of TATA, Mr. Bhabha established the Tata Institute of Fundamental Research in 1945 and later the
Trombay Atomic Energy Establishment (TAEE) in 1954 which played a major role in India›s nuclear program in fu-
ture.
y He was among very few people in India who realized the significance of atomic energy very early.
Source: https://www.freepressjournal.in/india/homi-bhabha-death-anniversary-remembering-father-of-indias-nucle-
ar-programme

189.
Answer: D
Context: The Indian Patent Office has granted a «record» 75,000 patents in the last 10 months, according to Commerce
Minister Piyush goyal.

CSE 2024 January 2024 155


DTRS Expalanations
Explanation:
Statement 1 is incorrect: The Department for Promotion of Industry and Internal Trade, under the Commerce Ministry,
is recognised as the nodal department to coordinate, guide and oversee implementation and future development of IPRs
in India.
Statement 2 is incorrect: According to the World Intellectual Property Indicators Report, China, the US, Japan, Republic of
Korea and Germany were the countries with the highest numbers of patent filings in 2022.
y India accounts for the sixth largest number of patent applications globally.
y However, patent filings by Indians surged by 31.6% in 2022, the sharpest increase globally, reflecting the country’s
growing innovation ecosystem.
y There has been a gradual increase in the filing and granting of patents in India. The number of patents filed in India
has gone up from 39,400 in 2010-11 to 45,444 in 2016-17 to 66,440 in 2021-22 and the patents granted in India
has gone up from 7,509 to 9,847 to 30,074 during the same time period.
Source: https://economictimes.indiatimes.com/news/india/as-many-as-75000-patents-granted-in-last-10-months-piyush-
goyal/articleshow/107171955.cms?from=mdr

190.
Answer: A
Context: The Supreme Court recently in January 2024 issued a non-bailable warrant against a litigant who had refused to
deposit costs earlier imposed on him and then went on to call the notice of contempt proceedings "useless" and "disre-
spectful."
Explanation:
Statement 1 is correct: The Constitution of India empowers both the Supreme Court and the High Courts to punish for con-
tempt of themselves (Articles 121 and 226, respectively). This power is crucial for maintaining the dignity and authority of
the courts, ensuring smooth administration of justice, and protecting the rule of law.
Types of Contempt:
Contempt can be classified into two categories:
y Civil Contempt: This involves willful disobedience of a court order or decree. For example, if a party in a lawsuit
refuses to pay damages awarded by the court, it could be held in civil contempt.
y Criminal Contempt: This involves acts or publications that scandalize or interfere with the court›s administration
of justice. This can include:
y Abusive language or behavior towards judges or court officers.
y Publishing material that prejudices a pending case.
y Obstructing the course of justice, such as tampering with evidence or intimidating witnesses.
Statement 2 is incorrect: Both Civil Contempt and Criminal Contempt are not explicitly mentioned in the Constitution of
India.
Article 121 for the Supreme Court and Article 226 for High Courts empower them to punish for contempt "for the purpose
of protecting the majesty and dignity of the Court and ensuring the due administration of justice."
The Constitution doesn›t explicitly define Criminal Contempt but the power stems from Articles 120 and 227, enabling both
courts to punish for any act that "scandalizes or tends to scandalize the court, or otherwise interferes or tends to interfere
with the due course of judicial proceedings."

CSE 2024 January 2024 156


DTRS Expalanations
Differences in Nature:
y Civil Contempt: Focuses on disobeying court orders or decrees. It aims to enforce compliance and vindicate the
court›s authority. For example, refusing to hand over property as ordered by a court can be civil contempt.
y Criminal Contempt: Addresses behavior that interferes with the court›s functioning, even without disobeying an
order. It protects the court from external disruption and maintains public confidence in its impartiality. Examples
include publishing scandalous articles about a pending case or threatening a witness.
Statement 3 is incorrect: While the Supreme Court in India holds immense power regarding contempt of court, it doesn›t
have the authority to make laws on the matter. This critical distinction rests on the separation of powers within the Indian
government.
Supreme Court›s Role in Contempt of Court:
y Defining the scope of contempt: While the Contempt of Courts Act outlines broad categories of contempt, the Su-
preme Court, through its judgments, has further clarified and interpreted these categories. This helps in applying
the law consistently and addressing emerging forms of contemptuous behavior.
y Punishment for contempt: The Court has the power to punish individuals and entities found guilty of contempt
through fines or imprisonment. However, this power must be exercised with utmost caution and due process, ad-
hering to principles of natural justice.
y Setting precedents: Supreme Court rulings on contempt cases create legal precedents that lower courts follow.
This helps maintain uniformity in the application of the law across the country.
Source: https://www.barandbench.com/news/supreme-court-non-bailable-warrant-litigant-who-said-contempt-no-
tice-useless-disrespectful

191.
Answer: C
Context: Fires in Ukhiya, Cox Bazaar located in Bangladesh have gutted about 800 refugee camps, rendering about 7000
Rohingya refugees homeless.

Explanation: Cox Bazar is located in the South-western part of Bangladesh. It houses one of the largest Rohingya refugee
settlements. However, it is highly prone to natural disasters such as tropical cyclones and forest fires. This raises humanitar-
ian concerns over the plight of these Rohingya refugees who have now been leaving Bangladeshi refugee camps to migrate
to Indonesia. Apparently, there have been news of missing Rohingya migrants in the Indian Ocean.
CSE 2024 January 2024 157
DTRS Expalanations

ABOUT ROHINGYAS: The Rohingya people are an ethnic group from Myanmar, once called Burma. Most live in Rakhine
State on Myanmar’s western coast.
Myanmar is a majority-Buddhist state, but the Rohingya people are primarily Muslim, though a small number are Hindu.
The ethnic minority is considered “the most persecuted minority in the world” by the United Nations.
As non-citizens, Rohingya people lack basic rights within Myanmar and are considered stateless. Though Myanmar recog-
nizes 135 distinct ethnic groups, the Rohingya are not one of them. Myanmar refuses to recognize the term as one that
refers to the region’s Muslim minority.
When Myanmar became a military state in 1962, the Rohingya became victims of state-sponsored persecution. During
“Operation King Dragon,” Burmese military forces targeted the Rohingya people, and were accused of human rights abuses
including rape, destruction of houses and villages, and mass arrests. Rohingya people began fleeing to nearby Bangladesh
in huge numbers. Another targeted campaign, “Operation Clean and Beautiful Nation,” pushed another 200,000 people
out of the country.
Today, the Rohingya are considered illegal immigrants by Myanmar, and are not recognized under the law.
Source: UNHCR: Nearly 7,000 Rohingya refugees homeless as fire guts Cox’s Bazar camp (dhakatribune.com); The Rohingya
people—facts and information (nationalgeographic.com)

192.
Answer: B
Explanation:
Context: The Finance Ministry confirmed the appointment of Arvind Panagariya, former Niti Aayog vice chairman and Co-
lumbia University professor, as the Chairman of the Sixteenth Finance Commission.
Statement 1 is incorrect:
y The Finance Commission is constituted by the President under Article 280 of the Constitution.

CSE 2024 January 2024 158


DTRS Expalanations
y As per the provisions contained in the Finance Commission [Miscellaneous Provisions] Act, 1951 and The Finance
Commission (Salaries & Allowances) Rules, 1951, the Chairman of the Commission is selected from among persons
who have had experience in public affairs, and the four other members are selected from among persons who--
(a) are, or have been, or are qualified to be appointed as Judges of a High Court; or
(b) have special knowledge of the finances and accounts of Government; or
(c) have had wide experience in financial matters and in administration; or
(d) have special knowledge of economics
Statement 2 is correct:
y The First Finance Commission was constituted vide Presidential Order dated 22.11.1951 under the chairmanship
of Shri K.C. Neogy on 6th April, 1952.
y Fifteenth Finance Commissions have been Constituted so far at intervals of every five years.
The recommendations of the Finance Commission are implemented as under:-
y Those to be implemented by an order of the President:
¾ The recommendations relating to distribution of Union Taxes and Duties and Grants-in-aid fall in this category.
y Those to be implemented by executive orders:
¾ Other recommendations to be made by the Finance Commission, as per its Terms of Reference.
Statement 3 is correct:
y It gives its recommendations on distribution of tax revenues between the Union and the States and amongst the
States themselves.
y Two distinctive features of the Commission’s work involve redressing the vertical imbalances between the taxation
powers and expenditure responsibilities of the centre and the States respectively and equalization of all public
services across the States.
y It is the duty of the Commission to make recommendations to the President as to—
y the distribution between the Union and the States of the net proceeds of taxes which are to be, or may be, divided
between them and the allocation between the States of the respective shares of such proceeds;
y the principles which should govern the grants-in-aid of the revenues of the States out of the Consolidated Fund of
India;
y the measures needed to augment the Consolidated Fund of a State to supplement the resources of the Panchayats
in the State on the basis of the recommendations made by the Finance Commission of the State;
y the measures needed to augment the Consolidated Fund of a State to supplement the resources of the Municipal-
ities in the State on the basis of the recommendations made by the Finance Commission of the State;
y any other matter referred to the Commission by the President in the interests of sound finance.
Source: https://www.thehindu.com/news/national/government-appoints-arvind-panagariya-as-sixteenth-finance-com-
mission-chief/article67692524.ece

193.
Answer: C
Explanation:
Context: The Ministry of Social Justice & Empowerment recently highlighted the Scheme for Residential Education for Stu-
dents in High Schools in Targeted Areas (SHRESHTA).
CSE 2024 January 2024 159
DTRS Expalanations
Scheme for Residential Education for Students in High Schools in Targeted Areas (SHRESHTA) is a scheme launched by the
Indian government in 2022 to improve the educational outcomes of Scheduled Castes (SC) students in specific/targeted
areas . The scheme aims to bridge the gap in educational opportunities for SC students by providing them with access to
high-quality residential education. Hence, option (c) is the correct answer.

The Ministry of Social Justice & Empowerment is the nodal ministry for the scheme.
SC students studying in class 9th and 10th in the current academic year are eligible under the scheme.
The scheme will cover the tuition fee, the hostel fee, and the mess fees for qualifying students.
Students will also be allowed to choose from a range of schools based on their merit.
The scheme will collaborate with voluntary organizations to provide an environment that can help create conditions for
socio-economic upliftment and the overall development of the SC communities.
Source: https://www.educationtimes.com/article/newsroom/99734595/the-cbse-issues-guidelines-for-private-residential-
schools-availing-shreshtha

194.
Answer: C
Explanation:
Context: On January 12, 2024 Papua New Guinea (PNG) Prime Minister declared a 14-day state of emergency, in response
to violent riots. The move raises critical questions about the balance between maintaining public order and upholding civil
liberties.
CSE 2024 January 2024 160
DTRS Expalanations
y Orang Asli is a tribe in Malaysia. The Bedouin are a tribe in the Arabian Peninsula. Hausa is a tribe in western Africa.
So, in the travel journey from east to west he/she will first meet Orang-Asli then Bedouin, and then Hausa. Orang
Asli is a collective term for some 18 ethnic groups of less than 150,000 in total who are widely regarded as com-
prising peninsular Malaysia’s original inhabitants.
y The Bedouin, Beduin or Bedu are nomadic Arab tribes who have historically in habitated the desert regions in the
Arabian Peninsula, North Africa, the Levant and Mesopotamia. However, the Arabian Peninsula is the historic and
original homeland of the Bedouin Arabs.
y The Hausa, numbering more than 20 million, are the largest ethnic group in west Africa.

Hence, Option (c) is the correct answer.


Source: https://thediplomat.com/2024/01/the-state-of-emergency-in-papua-new-guinea-balancing-order-and-rights/
https://www.downtoearth.org.in/interviews/environment/international-day-of-the-world-s-indigenous-peoples-2023-the-
orang-asli-use-traditional-knowledge-to-save-malayan-tigers-91087

195.
Answer: A
Context: The latest edition of the Annual Status of Education Report (ASER) has revealed that more than half of 14- to
18-year-old children in rural India cannot solve a simple three-digit division problem that’s usually taught in Class 3-4.
Explanation:
Statement 1 is correct: The statement reflects the core tenet of the Right of Children to Free and Compulsory Education
Act, 2009 (RTE Act). Article 21A of the Indian Constitution also makes education a fundamental right for children in this age
group. The RTE Act guarantees admission to a nearby school regardless of socio-economic background, gender, caste, or
religion.
Statement 2 is incorrect: While the RTE Act guarantees free education in government and government-aided schools, pri-
vately-run schools can charge fees.
Source: https://indianexpress.com/article/opinion/columns/aser-2023-report-on-education-lets-listen-to-the-teenag-
ers-9116242/
CSE 2024 January 2024 161
DTRS Expalanations
196.
Answer: B
Context: In June 2022, a faction of the Shiv Sena headed by Eknath Shinde moved with 37 of the 55 MLAs and claimed to be
the real Shiv Sena. It appointed Bharat Gogawale as its whip. However, the UBT faction claimed that they were the original
political party and that Sunil Prabhu of its faction will continue to be the whip. The Speaker has now recognised the Eknath
Shinde faction as the real Shiv Sena and held the appointment of whip by this group as valid. The Speaker has also refused
to disqualify 40 MLAs of the Eknath Shinde faction after recognising it as the real Shiv Sena.
Explanation:
Statement 1 is incorrect: The law provides exemption from disqualification to a member of a House on the ground of a
MERGER between political parties.
Statement 2 is incorrect: The law is silent on who can file the disqualification petition with respect to a member of a House.
Statement 3 is correct: The law bars a nominated member of a House from joining a political party after six months of his/
her date of taking the seat in the House. i.e. the nominated member can join any political party within six months of taking
a seat in the House without inviting disqualification.
Source: https://www.thehindu.com/news/national/understanding-the-tenth-schedule-explained/article67746169.
ece#:~:text=This%20Schedule%20provides%20that%20a,for%20disqualification%20from%20said%20House.

197.
Answer: D
Context: Despite being a leader in shrimp culture in India, Andhra
Pradesh has made to the headlines recently due to the environmental
damage caused by shrimp culture in the state.
The shrimp culture done in the brackish waters in Andhra Pradesh
has caused the ammonia levels in groundwater in the state to rise.
This has been adversely affecting the health of humans (leading to
skin allergies) and agricultural production in the Western districts of
the state.
Explanation:
The above statements characterize the state of Andhra Pradesh.
The commercial farming of this shrimp started from the year 2009-10
and is the largest cultured shrimp in terms of production and produc-
tivity. Andhra Pradesh tops in area under shrimp culture and produc-
tion followed by Gujarat way behind.
Kerala, Karnataka, Tamil Nadu, Gujarat, Maharashtra and Puducherry
also engage in shrimp culture in India.
The region of Konaseema, now a district, was known for its coconut trees. This district was carved out of the East Godavari
district of Andhra Pradesh.
Source: A catch in Andhra Pradesh’s aquaculture success story - The Hindu; State-wise Aquaculture Production – MPEDA

198.
Answer: C
Context: The Iran government has claimed that it has successfully launched three satellites into space.

CSE 2024 January 2024 162


DTRS Expalanations
Explanation:
These satellites were launched using the Simorgh rocket.
y The Simorgh is a part of Iran’s civilian space program. It is a two-stage, liquid-fueled rocket designed to place sat-
ellites into a low Earth orbit.
y The Mahda is a research satellite, while the Kayhan and the Hatef are nanosatellites focused on global positioning
and communication respectively.
y The United States has been apprehensive of Iran’s Simorgh program.
y The US intelligence community’s 2023 worldwide threat assessment report cites the Simorgh as a possible du-
al-use rocket.
Source: https://www.hindustantimes.com/world-news/iran-launches-3-satellites-into-space-amid-ballistic-missile-con-
cerns-101706430054020.html

199.
Answer: B
Context: The government has decided to extend benefits under the Remissions of Duties and Taxes on Export Products
(RoDTEP) scheme for shipments sent abroad via courier or e-commerce channels from sectors like gems and jewellery,
pharma, leather, textiles and handicrafts,
Explanation:
Statement 1 is correct: The scheme was announced as a replacement of the Merchandise Exports from India Scheme
(MEIS). The RoDTEP scheme can be said to be a combination of the MEIS and the Rebate of State and Central Taxes and
Levies (RoSCTL).
Statement 2 is correct: Under the scheme, exports will get a refund of underlying central, state and local duties or taxes that
were not yet exempted or refunded.
y Under the scheme, exporters receive refunds on the embedded taxes and duties previously non-recoverable.
y The chief aim of the scheme is to boost the export of

CSE 2024 January 2024 163


DTRS Expalanations
Statement 3 is incorrect: Remission of Duties and Taxes on Exported Products (RoDTEP) is It was announced in 2019 and
launched on 1 January 2021 by the Ministry of Commerce and Industry.
Source: https://www.thehindu.com/business/govt-makes-a-push-for-e-commerce-exports/article67684380.ece#:~:tex-
t=The%20government%20has%20decided%20to,Minister%20Piyush%20Goyal%20announced%20on

200.
Answer: C
Context: Encroachers have cleared about 70 hectares of forest in the buffer area of Udanti Sitanadi Tiger Reserve located
in Gariaband district of Chhattisgarh.
The Forest Department will remove these encroachers from the buffer area along the Odisha border in January 2024, after
the Chhattisgarh High Court ruled in Favour of the department.
Explanation: The Udanti Sitanadi Tiger Reserve, which was recently in the news due to issues related to encroachment, is
located in the state of Chhattisgarh.
About Udanti Sitanadi Tiger Reserve:
y The Udanti Sitanadi Tiger Reserve is a protected area in the Indian state of Chhattisgarh. It was formed in 2009 by
combining the Udanti and Sitanadi Wildlife Sanctuaries. The reserve is located in the Gariaband district, about 150
km from the state capital, Raipur.

y The Udanti Sitanadi Tiger Reserve has a total area of 4143.55 sq km. The core area of the reserve is 1557.94 sq km,
while the buffer area is 2585.61 sq km. The core area is made up of dense Sal forests, while the buffer area is a mix
of forests, grasslands, and agricultural land.
y The reserve is home to a variety of wild animals, including tigers, leopards, sloth bears, wild boars, chital deer, and
sambar deer. The reserve is also an important bird area, with over 250 species of birds recorded.
y The Udanti Sitanadi Tiger Reserve is a popular tourist destination. The reserve offers a variety of activities for
visitors, including jeep safaris, boat safaris, and treks. There are also a number of forest lodges and camps where
visitors can stay.
y The Udanti Sitanadi Tiger Reserve is an important part of the conservation efforts in India. The reserve is playing
a vital role in protecting the endangered tiger population. The reserve is also working to conserve other wildlife
species and to protect the natural habitat of the region.

CSE 2024 January 2024 164


DTRS Expalanations
The interesting facts about the Udanti Sitanadi Tiger Reserve:
y The reserve is home to one of the largest populations of Asian wild buffaloes in India.
y The reserve is also home to a number of rare and endangered species, such as the Indian pangolin and the dhole.
y The reserve is working to create a corridor between the Udanti Sitanadi Tiger Reserve and the Kanha National Park
in Madhya Pradesh. This would create a much larger protected area for tigers and other wildlife.
Source: https://timesofindia.indiatimes.com/india/odisha-residents-among-30-encroachers-cleared-from-udanti-sitana-
di-tiger-reserve-buffer-zone/articleshow/106605826.cms?from=mdr

CSE 2024 January 2024 165


DTRS Expalanations

CSE 2024 January 2024 166


DTRS Expalanations

CSE 2024 January 2024 167

You might also like